Vous êtes sur la page 1sur 103

Universidad La Salle.

Facultad Mexicana de Medicina.


Curso de extensin universitaria para la preparacin del Examen Nacional para
Aspirantes a Residencias Mdicas.
Examen Mdulo Medicina Interna.
Modalidad a distancia.
1.- Paciente femenino de 28 aos de edad, que acude a consulta por presentar dolor en el
recto, sin encontrarse una causa orgnica despus de mltiples exploraciones mdicas.
Adems refiere que desde hace 4 aos ha presentado vmitos, dolor abdominal
generalizado, palpitaciones, mareos, disfagia, visin borrosa, dolor en los miembros
inferiores, dismenorrea y dispareunia. Se observa triste y ansiosa. Cul es el diagnstico
ms probable?

a) Trastorno de ansiedad crnico


b) Depresin crnica con somatizacin
c) Trastorno de somatizacin
d) Trastorno de conversin

Referencia:
Lpez- Ibor J J, Valds M M. Manual diagnstico y estadstico de los trastornos
mentales (DSM- IV). Masson 2005. 545 574.

El Trastorno de somatizacin se caracteriza por la presencia de muchos sntomas


somticos que no pueden explicarse por los hallazgos fsicos o de laboratorio. Comienza
antes de los 30 aos, puede perdurar durante aos, es crnico y va asociado a malestar
psicolgico, a un deterioro del funcionamiento social y laboral y a la bsqueda excesiva de
ayuda mdica. Para hacer el diagnstico deben presentarse dolor en cuatro zonas del
cuerpo, dos sntomas gastrointestinales, un sntoma sexual y un sntoma pseudoneurolgico.
La ansiedad y la depresin son las patologas psiquitricas ms prevalentes.

2.- Una mujer de 25 aos de edad presenta fiebre y artritis inflamatoria que afecta a las
articulaciones metacarpofalngicas e interfalngicas proximales Cul de las siguientes
manifestaciones es muy sugestiva de lupus eritematoso diseminado y no de artritis
reumatoide?
a) Sedimento urinario activo (eritrocitos, leucocitos, cilindros celulares, sin bacterias).
b) Artritis inflamatoria de articulaciones metacarpofalngicas e interfalngicas.
c) Derrame pleural de la radiografa de trax.
d) Pruebas de funcionamiento heptico anormales.

Referencia:
Allen R. M. MMS Medicina Interna. 5. Edicin. National Medical Series. Mc. Graw Hill.
2006. (captulo 10IIG;VIIG l;cuadro 10-10).

Un sedimento urinario activo sugiere glomerulonefritis, dato comn en pacientes con lupus
eritematoso diseminado, pero no en quienes tienen artritis reumatoide. Pueden encontrarse
artritis de articulaciones metacarpofalngicas e interfalngicas proximales derrames
pleurales y anemia en cualquiera de las dos enfermedades. Las anomalas de la funcin
heptica son atpicas en el lupus eritematoso sistmico, pero comnmente son resultado de
disfuncin heptica relacionada con frmacos (p. ej., por antiinflamatorios no esteroideos
en cualquier enfermedad.

3.- Paciente de 52 aos de edad con diagnstico de neumona adquirida en la comunidad,


quien recibi tratamiento antibitico a base de Ceftriaxona. No tuvo respuesta adecuada, y
en el estudio diagnstico para determinar la causa, se encontr un derrame pleural del
60%, el cual se puncion. En el estudio citoqumico y bacteriolgico de este lquido, se
reportan cocos grampositivos en la tincin de Gram, y un pH de 7. Con esto, usted
considera como indispensable:

a)
b)
c)
d)

Iniciar cobertura con vancomicina.


Colocacin de sonda endopleural.
Intubacin orotraqueal y ventilacin con volmenes altos.
Ingreso a Unidad de Terapia Intensiva.

Los hallazgos del citoqumico y tincin son caractersticos de empiema. El drenaje del
mismo es la maniobra teraputica base para el tratamiento y resolucin del mismo.
Kasper DL, Braunwald E, Fauci AS, Hauser SL, Longo DL, Jameson JL. Harrisons Principles
of Internal Medicine. McGraw Hill. 16 Ed. 1536 p.

4.- Ante un paciente con diagnstico de trastorno obsesivo-compulsivo, usted decide


indicar el siguiente frmaco por ser el principal tratamiento de eleccin:

a) Antipsicticos.
b) Antidepresivos inhibidores de la recaptacin de noradrenalina.
c) Antidepresivos inhibidores de la recaptacin de serotonina.
d) Benzodiacepinas.

El abordaje farmacolgico del TOC se basa en los inhibidores selectivos de la recaptura de


serotonina (ISRS), medicamentos que han resultado ser efectivos y seguros. Como grupo,
los ISRS son igual de eficaces que la clorimipramina, pero producen menos efectos
secundarios y, por lo tanto, mejor tolerancia y mejor apego al tratamiento. La efectividad
antiobsesiva parece ser independiente de su actividad antidepresiva.
Entre los factores predictores de respuesta al tratameinto, se ha propuesto que los altos
puntajes en las compulsiones predicen una mala respuesta al tratamiento. Para los casos
refractarios y resistentes o cuando hay sntomas de comorbilidad, se han utilizado, con
relativo xito, combinaciones con diferentes IRS, con benzodiacepinas, o bien, con
potenciadores como el litio, o antipsicticos, como la risperidona y el haloperidol. El uso del
carbonato de litio es controvertido, aunque parece ser til como potenciador a largo plazo
entre 15 y 30% de los pacientes. La combinacin de ISRS con antipsicticos comenz a
utilizarse en los pacientes con sntomas psicticos, aunque ahora se combinan tambin en
los pacientes resistentes.
Tratamiento farmacolgico del TOC
Cristina Lyzaga*
Humberto Nicolini*

*Divisin de Investigaciones Clnicas. Instituto Nacional de Psiquiatra.


Ramn de la Fuente. Calzada Mxico-Xochimilco, 101, San Lorenzo
Huipulco, 14370, Mxico D.F.
Primera versin: 14 de septiembre de 2000.
Aceptado: 26 de septiembre de 2000.

5.- Un paciente con carcinoma pulmonar presenta nuseas, vmitos y letargo y se encuentra
que tiene una concentracin srica de calcio de 13.4 mg/100 ml. Cul de los siguientes
agentes debe ser el primer paso para el tratamiento?
a) Etidronato intravenoso
b) Mitramicina intravenosa
c) Glucocorticoides intravenosos
d) Solucin salina y furosemida intravenosos

Allen R. M. MMS Medicina Interna. 5. Edicin. National Medical Series. Mc. Graw Hill.
2006. (captulo 9 IIIA 6 a, b, d, 7 c (1), (3)). La hipercalciemia causada por otras
enfermedades aparte del hiperparatiroidismo puede tratarse con solucin salina y
furosemida intravenosos. El reemplazo de lquidos con solucin salina intravenosa, seguido
por diuresis forzada con solucin salina y furosemida intravenosas, es una forma rpida y
segura de disminuir el calcio srico y debe intentarse primero. Pueden aadirse
pamidronato, mitramicina o calcitonina si se necesita disminuir an ms la concentracin de
calcio. Los glucocorticoides son eficaces para tratar la hipercalciemia causada por exceso

de vitamina D, sarcoidosis y algunas neoplasias malignas hemticas, pero no disminuyen el


calcio srico en la mayor parte de los casos de hipercalciemia relacionada con tumores
slidos.

6.- Una mujer de 20 aos de edad presenta trombosis venosa profunda de nueva aparicin
en la extremidad inferior izquierda. Tiene antecedentes de trombocitopenia leve y dos
abortos; fue tratada para sfilis hace dos aos debido a una prueba de reagina rpida del
plasma positiva, aunque el anticuerpo antitreponmico fue negativo. Cul de los siguientes
trastornos es ms probable que represente el conjunto de su historia clnica?
a)
b)
c)
d)

Sndrome de anticuerpos antifosfolpido


Lupus eritematoso diseminado (SLE)
Sndrome de anticuerpo Ro
Arteritis de Takayasu

Allen R. M. MMS Medicina Interna. 5. Edicin. National Medical Series. Mc. Graw Hill.
2006. (captulo 10VIIFl,2c, G3b). Esta paciente tiene varias manifestaciones sutiles de
sndrome de anticuerpo antifosfolpido, que en conjunto hacen de ste un diagnstico
probable. La prueba de reagina plasmtica rpida (RPR) positiva, con resultados negativos
de la prueba de treponemas, probablemente refleja anticuerpos que producen reaccin
cruzada a la cardiolipina o a componentes fosfolpidos de antgenos treponmicos. La
trombocitopenia es frecuente en esta situacin, debido a las interacciones de plaquetas y
clulas endoteliales y a la coagulacin inducida por los anticuerpos. Los abortos pueden
deberse a coagulacin en vasos placentarios pequeos. Las trombosis venosas profundas o
incluso la coagulacin en arterias mayores pueden ser causadas por hipercoagulabilidad
consecuente. No hay otras pruebas de lupus eritematoso diseminado, aunque pueden
encontrarse anticuerpos antifosfolpido en alrededor de 33% de los pacientes.

Tampoco se describen datos de sndrome de anticuerpo Ro o enfermedad indiferenciada


del tejido conjuntivo. La arteritis de Takayasu se presenta en mujeres asiticas jvenes,
pero no hay datos de isquemia de grandes vasos arteriales caracterstica de esta
enfermedad.

7.- Masculino de 56 con derrame pleural. Se realiza una toracocentesis y se analiza el


fluido revelando que tiene una diferencia de protenas con el suero de 0.9 y diferencia de
DHL con suero de 1. Este fluido es un:
a) Trasudado
b) Exudado
c) Derrame
d) Infiltrado

Toracentesis y anlisis del lquido pleural. La toracentesis diagnstica requiere


menos de 30 ml de lquido. En los derrames enquistados es til la ecografa para localizar
con precisin el lquido, y con ello hacer la toracentesis ms fcil y sin riesgos (Ver gua
para drenaje y succin pleural).
Los derrames pleurales se clasifican en trasudados y exudados. Un trasudado es un filtrado
de plasma que resulta del aumento de la presin hidrostlica o de la alteracin de la
permeabilidad capilar. Los trasudados se asocian con insuficiencia cardiaca congestiva,
sindrome nefrtico, cirrosis y condiciones de sobrecarga de volumen.
El exudado es un lquido rico en protenas resultante de una inflamacin local o por una falla
en la eliminacin de protenas por los linfticos o ambos mecanismos. Los exudados se
producen en infecciones colagenopatas y neoplasias.
Existen criterios para la diferenciacin entre trasudados y exudados (los exudados deben
cumplir al menos uno de los siguientes criterios):

1.
2.
3.
4.

Relacin de protena pleural/srica > 0.5


Relacin de LDH pleural/srica > 0.6
LDH pleural > 200 Ul
Estos criterios tiene significancia diagnstica con una sensibilidad del 98% y
especificidad de 77%.

Otros criterios tiles para el diagnstico son:


a. Colesterol total en lquido pleural mayor de 60 mg/dl
b. Relacin de colesterol pleural/srico mayor 0.4

Otro anlisis del lquido pleural de importancia es la concentracin de glucosa. Un resultado


bajo (< 60 mg/dl) en el lquido pleural es sugestivo de empiema, neoplasia, TBC, LES o
pleuresa reumtica.
El pH normal es de 7.60 encontrndose menor a 7.30 en las mismas entidades patolgicas
descritas para la glucosa, y en la ruptura esofgica; en los derrames por neoplasia un pH
bajo se relaciona con menor sobrevida y menor respuesta a la pleurodesis qumica.
La medicin de triglicridos es til ante la sospecha de quilotrax (> 110 mg/dl).
La medicin de adenosina deaminasa permite la diferenciacin de TBC pleural y neoplasia
cuando es mayor de 45 UI.

El recuento y la diferenciacin celular ayudan al diagnstico etiolgico del derrame. Los


trasudados en general tienen menos de 1.000 leucocitos/ml; recuentos mayores a
10.000/ml se ven en derrames paraneumnicos, mayores de 50.000/ml en empiema. Los
derrames crnicos (TBC, neoplasia) tienen menos de 5.000/ml. La linfocitosis es indicativa
de TBC, neoplasia, linfoma, sarcoidosis, pleuresa reumtica. Se encuentra predominio
neutroflico en neumona, embola y pancreatitis.
Ante la sospecha de neoplasia, se debe solicitar una citologa del lquido pleural, la cual
tiene una sensibilidad importante.

LECTURAS RECOMENDADAS
American College of Physicians. Diagnostic thoracentesis and pleural biopsy in pleural
effusions Ann Inter Med 103:799, 1985.
Camacho Durn F, Restrepo Molina J. Enfermedades de la pleura. En: Fundamentos de
Medicina. Neumologa. Tercera Edicin.
Corporacin para Investigaciones Biolgicas. Medelln, 1986.
Des Jardins T. Enfermedades pleurales En: Enfermedades Respiratorias. Terry Des
Jardins Editor.

Editorial El Manual Moderno SA Mxico DF, 1993.


Light RW, Mac Gregor M. The diagnostic separation of trasudates and exudates. Ann
Intern Med 77: 507, 1972.
Pacheco PM. Estudio del derrame pleural En: Enfermedades del Trax. Fidel Camacho,
Jaime Pez, Carlos Awad Editores. Ediciones Mdicas Zambn. Santaf de Bogot, 1992.
Patio JF, Arroyo de S. Gua prctica de toracentesis y de toracostoma cerrada (insercin
de tubo de trax).Trib Mdica 89:161, 1994.
Patio JF, Arroyo de S. Gua para drenaje y succin pleural. Manejo del drenaje pleural.

8.- Se trata de paciente de 30 aos que refiere datos de ansiedad, enojo e inseguridad
refiriendo que existe una constante sensacin de que en su trabajo no realiza
correctamente las funciones que se encomiendan. Esto hace que necesite permanentemente
revisarlas una y otra vez, lo que le supone prdida de tiempo y eficacia. Esta sensacin es
tan dominante en su psiquismo que le conduce a la idea de fracaso y a la prdida de
autoestima. El diagnstico inicial corresponde a trastorno:

a) Psictico paranoide
b) Por ansiedad fbica
c) De personalidad evitativo-fbica
d) Obsesivo-compulsivo

CRITERIOS DEL DSM-IV PARA EL DIAGNSTICO DEL TRASTORNO OBSESIVOCOMPULSIVO. Fuente AMERICAN PSYCHIATRIC ASSOCIATION.

Criterios para el diagnstico de F42.8 Trastorno obsesivo-compulsivo (300.3)


A. Se cumple para las obsesiones y las compulsiones:
Las obsesiones se definen por 1, 2, 3 y 4:
1. Pensamientos, impulsos o imgenes recurrentes y persistentes que se experimentan
en algn momento del trastorno como intrusos e inapropiados, y causan ansiedad o
malestar significativos.

2. Los pensamientos, impulsos o imgenes no se reducen a simples preocupaciones


excesivas sobre problemas de la vida real.

3. La persona intenta ignorar o suprimir estos pensamientos, impulsos o imgenes, o


bien intenta neutralizarlos mediante otros pensamientos o actos.

4. La persona reconoce que estos pensamientos, impulsos o imgenes obsesivos son el


producto de su mente (y no vienen impuestos como en la insercin del pensamiento)

Las compulsiones se definen por 1 y 2:


1. Comportamientos (p. ej., lavado de manos, puesta en orden de objetos,
comprobaciones) o actos mentales (p. ej., rezar, contar o repetir palabras en
silencio) de carcter repetitivo, que el individuo se ve obligado a realizar en
respuesta a una obsesin o con arreglo a ciertas reglas que debe seguir
estrictamente.

2. El objetivo de estos comportamientos u operaciones mentales es la prevencin o


reduccin del malestar o la prevencin de algn acontecimiento o situacin negativos;
sin embargo, estos comportamientos u operaciones mentales o bien no estn conectados
de forma realista con aquello que pretenden neutralizar o prevenir o bien resultan
claramente excesivos.

B. En algn momento del curso del trastorno la persona ha reconocido que estas obsesiones
o compulsiones resultan excesivas o irracionales.
Nota: Este punto no es aplicable en los nios.
C. Las obsesiones o compulsiones provocan un malestar clnico significativo, representan
una prdida de tiempo (suponen ms de 1 hora al da) o interfieren marcadamente con la
rutina diaria del individuo, sus relaciones laborales (o acadmicas) o su vida social.
D. Si hay otro trastorno, el contenido de las obsesiones o compulsiones no se limita a l (p.
ej., preocupaciones por la comida en un trastorno alimentario, arranque de cabellos en la
tricotilomana, inquietud por la propia apariencia en el trastorno dismrfico corporal,
preocupacin por las drogas en un trastorno por consumo de sustancias, preocupacin por
estar padeciendo una grave enfermedad en la hipocondra, preocupacin por las
necesidades o fantasas sexuales en una parafilia o sentimientos repetitivos de culpabilidad
en el trastorno depresivo mayor).
E. El trastorno no se debe a los efectos fisiolgicos directos de una sustancia (p. ej.,
drogas, frmacos) o de una enfermedad mdica.

Especificar si:

Con poca conciencia de enfermedad: Si, durante la mayor parte del tiempo del episodio
actual, el individuo no reconoce que las obsesiones o compulsiones son excesivas o
irracionales.

9.- Se trata de paciente femenino de 43 aos de edad con diagnstico de polimiosistis al


realizar la exploracin fsica, la disminucin de la fuerza muscular se caracteriza por:
a)
b)
c)
d)

Ser de predominio proximal


Ser de predominio distal
Acompaarse de dolor intenso
Afectar nicamente a la musculatura distal

La polimiositis (PD) y dematomiositis (DM), son enfermedades inflamatorias en las cuales el


compromiso principal es la debilidad muscular, generalmente proximal y simtrica con
atrofia subsecuente, generalmente indolora.

Su etiologa y patogenia siguen siendo desconocidas, siendo considerada dentro de las


enfermedades difusas del tejido conectivo.
Numerosos estudios sugieren que una exposicin previa a diversos virus como Influenzae A
y B, Coxsackie virus tipo B y Picornavirus pueden desencadenar PM-DM; adems se cree
que los fenmenos inmunolgicos juegan un papel preponderante, por la presencia de
autoanticuerpos, depsito de inmunoglobulinas y complemento en las paredes vasculares.
A nivel anatomo-patolgico las principales alteraciones se encuentran en los msculos
esquelticos y en los vasos sanguneos. Siendo la vasculitis un signo comn en la
dermatomiositis infantil y su severidad indicar una peor evolucin. Clnicamente los
sntomas iniciales son la debilidad muscular proximal, el rash tpico, frecuente el edema en
cara y a veces en miembros, fiebre moderada, posteriormente dificultad para la deglucin y
debilidad en la voz; asimismo manifestaciones viscerales que no se presentan siempre, pero
son de gran importancia en cuanto al pronstico, asimismo transtornos ventilatorios
(pulmonares): por afectacin primaria (neumona intersticial) o secundaria a la disfuncin
farngea (aspiracin) o a la debilidad de los msculos respiratorios (insuficiencia
ventilatoria), o como complicacin del tratamiento (infecciones oportunistas).

Wortmann RL. Idiopathic inflammatory myopathies. A. Clinical features. In: Primer on the
rheumatic diseases. New York: Springer-Arthritis Foundation; 2008. p. 363-7.

10.- Un trabajador limpia vidrios con antecedente de fractura vertebral de C6 y fractura


del extremo distal del radio derecho tras cada de un tercer piso se presenta a consulta,
refiere dolor, limitacin de la movilidad y hormigueo en los dedos 2 y 3 de la mano
derecha, de predominio nocturno, que mejora durante el da. Cul de los siguientes
diagnsticos es el ms probable?:
a) Sndrome del tnel del carpo.
b) Radiculalgia C5-C6.
c) Distrofia simptico refleja.
d) Artrosis postraumtica del carpo.

El Sndrome del Tnel Carpiano (STC) es una patologa que afecta a la mano, provocada
por una presin sobre el nervio mediano a nivel de la mueca. Esto provoca sntomas como
adormecimiento y hormigueos en la mano (especialmente en los dedos pulgar, ndice,
corazn y mitad del anular). Puede existir dolor, que puede estar limitado a la mano y
mueca, pero que en algunas ocasiones se irradia hacia el antebrazo. El STC con frecuencia
despierta al paciente por la noche, y los sntomas pueden aparecer con actividades como
conducir un vehculo, escribir, u otros ejercicios que suponen una utilizacin significativa de
la mano. En el sndrome del tnel del carpo avanzado, puede producirse una prdida de
fuerza y una disminucin de la masa muscular en la base del pulgar.

CAUSA
El tnel del carpo es un canal formado por los huesos de la mueca y un ligamento (el
ligamento transverso del carpo) situado en la cara palmar de la mueca. Por este tnel
transcurren todos los tendones que flexionan la mueca y los dedos, y el nervio (nervio
mediano) que recoge la sensibilidad del pulgar, ndice, corazn y parte del anular y moviliza
los msculos de la base del pulgar. Algunas personas nacen con tneles estrechos y por
tanto estn predispuestas a problemas de presin sobre el nervio. La utilizacin vigorosa de
la mano, que conduce a una tendinitis de los tendones que flexionan el pulgar y los dems
dedos, tambin puede conducir a un sndrome del tnel del carpo a travs del
engrosamiento de las vainas tendinosas. Las vainas engrosadas "rellenan" el tnel
presionando sobre el nervio. Las personas con artritis reumatoide, hipotiroidismo, diabetes,
amiloidosis, insuficiencia renal y algunos otros problemas mdicos estn ms predispuestas
a padecer este sndrome.

1.

Arthroshi I, Gummenson C, Johonsson R, Ornstein E, Ranstam J, Rossen I.


Prevalence of carpal tunnel syndrome in a general population. JAMA. 1999;282:1538 Peridico El Pblico, 17 de octubre del 2007,pag 26.
2. Durkan, JA. The carpal compression test: an instrumental device for diagnostic
carpal tunnel syndrome. Lancet. 1990;335:393-5.
3. Marshall S, Tardif G. Injection local of steroids in the carpal syndrome. Cochrane
Data Base of Systematic Reviews. 2005. Issue 5.
4. Verdugo RJ, Salinas RS, Castillo J, Cea JG. Tratamiento quirrgico versus
tratamiento no quirrgico para el sndrome del tnel carpiano. Cochrane Data Base.
2005.

11.- Masculino de 50 aos con dolor precordial relacionado al esfuerzo, de corta duracin,
de 4 meses de evolucin y con ECG en reposo normal, el siguiente estudio de eleccin es:

a) Prueba de esfuerzo con protocolo de Bruce


b) Ecocardiograma de reposo
c) Prueba de Talio- Dipiridamol
d) Ecocardiograma con estrs farmacolgico

El propsito fundamental de la prueba es el de demostrar la existencia de isquemia


miocrdica en los subgrupos de poblacin con mayor prevalencia de cardiopata isqumica o
bien en los subgrupos en donde la prueba se efecta a manera de evaluacin del
tratamiento en pacientes ya conocidos con cardiopata isqumica. Existe otro subgrupo de
poblacin en quienes este estudio ayuda a determinar la clase funcional en la que se
encuentran e incluye a pacientes con valvulopatas o con insuficiencia cardiaca de cualquier
etiologa.

Protocolos de esfuerzo.
El protocolo ms empleado es el de Bruce sobre treadmill, aunque existen otros protocolos
y su eleccin depender de las condiciones del individuo.
Los protocolos discontinuos son los que alternan periodos de esfuerzo que se intercalan con
periodos de reposo de duracin similar, se emplean en escasas circunstancias. Los
protocolos continuos son los que no interrumpen el esfuerzo una vez iniciado hasta
finalizada la prueba, permiten mejor adaptacin fsica y psicolgica y es posible adaptar la
intensidad de forma individualizada para que la prueba tenga una duracin de 6 a 12
minutos.

Los protocolos mximos son los que se suspenden debido a la sintomatologa del paciente, a
los signos registrados durante la prueba o se alcanzan valores mximos de FC y VO2. Los
protocolos submximos son los que se suspenden cuando el sujeto alcanza un nivel
determinado de carga, habitualmente el 85% de la FC mxima terica (que se encuentra
entorno a los 170 lpm).
En la prctica diaria, el nivel de carga (VO2) se expresa en forma de trabajo externo (MET
equivalentes metablicos) que corresponden a 3,5ml/kg/min de VO2, lo que permite
comparar protocolos entre s (cada protocolo dispone de frmulas para realizar el clculo
de los METS), el error que cometen en el clculo de los METS es mayor en protocolos
discontinuos.
Emplear la FC como nico criterio para determinar el esfuerzo mximo es errneo, por lo
que deberan tenerse en cuenta otros criterios, como es la percepcin subjetiva por parte
del paciente mediante la escala de Borg (tabla I). Esta dificultad en la prediccin del
esfuerzo mximo es lo que limita la realizacin de pruebas submximas a la determinacin
de la condicin fsica de sujetos aparentemente sanos.
Tabla I. Escala de Percepcin del esfuerzo de Borg.
Escala de 15 grados

Escala de 10 grados

Valor

Percepcin

Valor

Percepcin

No se siente nada

Nada

0,5

Muy muy leve

Muy leve

Muy leve

Leve

Considerablemente leve

Moderada

Algo fuerte

Fuerte o intensa

Muy muy leve

8
9
10
11
12
13

Moderadamente dura

14

Dura

15
16

Muy dura

17
18

Muy muy dura

19
20

Muy fuerte

7
8
9
10

Muy muy fuerte


(submxima)

Esfuerzo mximo

* A la izquierda la escala original de esfuerzo percibido en 15 grados (de 6 a 20) y a


la derecha la ms nueva de 10 categoras.
Tabla II. Indicaciones clsicas de la ergometra.
I. Fines diagnsticos
A. Pacientes sintomticos.
1. Dolor torcico:
a) Tpico
b) Atpico
2. Clnica de equivalentes isqumicos.
A. Pacientes asintomticos.
1. Con alteraciones en el ECG sugestivas de isquemia.
2. Con alta probabilidad de padecer Cardiopata Isqumica (paciente con
mltiples factores de riesgo)
3. Cuando convenga descartar con cierta seguridad CI.
4. Con sospecha de CI silente.
5. Sedentarios que inician programa de actividad fsica.
6. Para estudio funcional de ciertas arritmias.
II. Con fines valorativos y pronsticos.
1. Seguimiento de paciente con CI conocida.
2. Tras IAM.
3. En exmenes prelaborales o laborales.
4. De la eficacia del tratamiento:

Mdico.
Cateterismo y angioplastia.
Quirrgico.
5. Respuesta de la Tensin Arterial.
6. En valvulopatas o miocardiopatas.
7. Estudio de arritmias y trastornos de la conduccin aurculo-ventricular.
8. En cardiopatas congnitas.
Bibliografa
1.

2.
3.

4.

5.

6.

7.
8.

9.
10.
11.
12.

13.

Gibbons RJ (Edit.). ACC/AHA 2002 Guideline Update of Exercise Testing. 2002


American College of Cardiology Foundation and American Heart Association
ACC/AHA; 2002 [Acceso 1-4-06]. Disponible en:
Guidelines for cardiac exercise testing. ESC Working Group on Exercise
Physiology, Physiopathology and Electrocardiography Eur Heart J 1993; 14: 969-988.
Fernando Ars Aros F, Boraita A, Alegria E, Alonso AM, Bardaji A, Lamiel R el al.
Guas de prctica clnica de la Sociedad Espaola de Cardiologa en pruebas de
esfuerzo. Rev Esp Cardiol 2000; 53 (8): 1063-94.
Chaitman B. Las pruebas de esfuerzo. En: Braunwald E, editor. Tratado de
Cardiologa. Medicina Cardiovascular. 4 ed. Madrid Mc-Graw-Hill-Interamericana de
Espaa; 1993. p. 177-197.
Schlant RC, Friesinger GC 2nd, Leonard JJ. Clinical competence in exercise testing:
a statement for physicians from the ACP/ACC/AHA Task Force on Clinical Privileges
in Cardiology. J Am Coll Cardiol 1990; 16: 1061-5.

Reyes Lopez de los M, Iiguez Romo A, Goicolea de Oro A, Funes Lopez B, Castro
Beiras A. El consentimiento informado en cardiologa. Rev Esp Cardiol 1998; 51: 782796.
Fletcher GF, Flipse T, Malouf J, Kligfield P. Current status of ECG stress testing.
Curr Probl Cardiol. 1998 Jul; 23(7): 353-423.
Alegra Ezquerra E, Alijarde Guimer M, Cordo Mollar JC, Chorro Gasc FJ,
Pajarn Lpez A. Utilidad de la prueba de esfuerzo y de otros mtodos basados en el
electrocardiograma en la cardiopata isqumica crnica. Rev Esp Cardiol 1997; 50: 6-14
Wasserman K, Hansen JE, Sue DY, Whipp BJ, Casaburi R. Principles of exercise
testing and interpretation . 2 ed. Philadelphia: Lea & Febiger; 1994. p. 95-111.
American college of Sports Medicine. Guideliness for exercise testing and
prescription. 5 ed. Baltimore: Williams & Wilkins; 1995.
Borg GA. Psychophysical bases of perceived exertion. Med Sci Sports Exerc 1982;
14: 377-381
Froelicher VF, Umann TM. Exercise testing: clinical applications. En: Pollock ML,
Schmidt DH, editors. Heart disease and rehabilitation . 3 ed. Champaign, IL: Human
Kinetics, 1995; p.57-79.
Myers J, Froelicher VF. Exercise testing. Procedures and implementation. Cardiol
Clin. 1993; 11(2): 199-213.

14. Weiner DA, McCabe C, Hueter DC, Ryan TJ, Hood WB Jr. The predictive value of
anginal chest pain as an indicator of coronary disease during exercise testing. Am
Heart J 1978; 96: 458-462.

12.- Una mujer de 40 aos de edad en buena salud general experimenta dolor retroesternal
sbito con fiebre y falta de aire. Es fumadora y no toma medicamentos excepto
anticonceptivos orales. En la exploracin fsica se encuentran taquipnea y temperatura de
38C. Los datos de auscultacin, percusin y radiogrficos del trax son normales. Cul de
los siguientes diagnsticos es ms probable?

a) Traqueobronquitis
b) Neumona atpica
c) Embolia pulmonar
d) Neumona bacteriana

Allen R. M. MMS Medicina Interna. 5. Edicin. National Medical Series. Mc. Graw Hill.
2006. (captulo 2 VIII E 1, 2 a; captulo 8 V C 3, 4). El diagnstico ms probable es embolia
pulmonar. El inicio agudo descarta neumona atpica y hace poco probable el cncer
pulmonar. Sin pruebas de tos productiva es poco probable que haya traqueobronquitis,
trastorno tambin subagudo. La neumona bacteriana es muy improbable junto con la
radiografa de trax. El tabaquismo y el uso de anticonceptivos orales predisponen a
trombosis venosa profunda y embolias pulmonares.

13.- Durante su consulta en la unidad de Medicina Familiar recibe a un paciente de 60 aos


de edad que asegura ser hipertenso de ms de cinco aos de evolucin, sin agudizaciones
adems de ser portador de una fibrilacin auricular, motivo por los cuales recibe
propranolol en dosis de 100 MG cada maana. Al revisar su electrocardiograma usted
espera encontrar:

a) Ausencia de onda P, complejo QRS normal e intervalos R-R diferentes.

b) Ausencia de onda P, complejo QRS ensanchado, diferencia constante de los intervalos RR.
c) Presencia de onda P, complejo QRS ensanchado y eje rotado a la derecha.
d) Ritmo nodal con disociacin A-V

Guadalajara J. Cardiologa. Sexta Edicin 151 152 La fibrilacin auricular es la


arritmia cardiaca ms frecuente. La despolarizacin catica y desordenada de las aurculas
por mltiples ondas en simultnea, trae como consecuencia que se pierda la funcin
mecnica de la contraccin auricular. Estos dipolos de activacin mltiple y desordenados
alcanzan al nodo A-V y penetran en l, algunos pasan hacia el Haz de His mientras que otros
no lo alcanzan, debido a la penetracin parcial del nodo por conduccin decreciente
completa, esto se conoce como, conduccin oculta; as, la rpida penetracin de los
estmulos auriculares favorece la aparicin de conduccin oculta, la cual afecta en forma
impredecible el periodo refractario del nodo. El trazo electrocardiogrfico tpico es la
ausencia de onda P, complejo QRS normal e intervalos R-R diferentes.

14.- Femenino de 59 aos, refiere vivir sola, como antecedentes refiere HTA y artritis
reumatoide, con buen control farmacolgico. Presenta alucinaciones auditivas y
cenestsicas, con ideas delirantes de perjuicio con los vecinos y de contenido msticoreligioso de 4 meses de evolucin. Cul sera el diagnstico ms probable?

a) Sndrome confusional agudo.


b) Depresin delirante.
c) Esquizofrenia de inicio tardo.
d) Psicosis psicgena.

LOS LTIMOS AOS se ha incrementado el inters por el estudio de los estados


psicticos de inicio tardo y su relacin con el deterioro cognoscitivo y los procesos
demenciales ya instalados.
Las dificultades en la organizacin de un cuadro clnico especfico de psicosis tarda que
responda a un estado nosolgico, con caractersticas clnicas propias y a una etiologa clara,
que le de cuerpo como una entidad, ha generado confusin entre los diversos investigadores
clnicos y epidemiolgicos.

Esquizofrenia tarda: se caracteriza por la presencia de delirios, con alucinaciones y con


desorganizacin de la personalidad, alteracin afectiva y conductas bizarras. Hay un menor
grado o ausencia de deterioro cognoscitivo, con una edad de inicio entre los 45 y 60 aos.
No hay antecedentes de trastornos psicticos, afectivos o demenciales.
. Esquizofrenia de inicio muy tardo en el anciano: son cuadros clnicos de tipo psictico en
ancianos-ancianos (mayores de 60 aos) que presentan sintomatologa delirante poco
estructurada, con alucinaciones, con ausencia de deterioro cognoscitivo generalizado y
progresivo, sin compromiso afectivo significativo.

Existen otros estados psicticos no esquizofrnicos de aparicin en edades avanzadas,


como: el trastorno delirante de ideas persistentes, la paranoia, el trastorno delirante de
perjuicio, los delirios hipocondracos de enfermedad y los delirios erotomanacos.

Cuadro clnico

Diversos autores han intentado caracterizar un cuadro clnico de las psicosis de inicio
tardo en donde predominan los delirios, las alucinaciones, interpretaciones paranoides
(errores de percepcin) y otros sntomas Scheneiderianos. En ancianos con psicosis tardas
el cuadro clnico puede estar conformado por:
Fenmenos delirantes, especialmente paranoides de tipo persecutorio o de referencia.
Pueden presentar un tipo especial de delirios denominados de tabique, los cuales se
caracterizan por la creencia de que detrs de las paredes de su habitacin operan personas
con el propsito de hacerles dao o conspirar contra sus intereses e interferir contra su
vida.
. Fenmeno del eco del pensamiento: manifiestan que sus pensamientos son ledos o
robados.
. Presencia de alucinaciones especialmente auditivas, con contenidos de referencia,
persecutorios o erticos. Tambin pueden presentar alucinaciones visuales, tctiles y
olfativas.

. Ausencia de trastornos cognoscitivos generalizados y progresivos.


. Los trastornos afectivos no son significativos, de tal modo que se sospeche de cuadros de
trastorno esquizoafectivo; pero estos ancianos psicticos pueden presentar cuadros
depresivos moderados, sin que lleguen a constituirse en una depresin mayor de tipo
psictico.
. Edad de inicio por encima de 60 aos.
. Co-morbilidad frecuente con personalidad premrbida esquizoide o paranoide.
. Alteraciones del comportamiento se presentan ms en el perodo de estado de la crisis
psictica, pero en general hay menos desorganizacin y conductas bizarras que en aquellos
con cuadros crnicos de esquizofrenia.
. Falta de insight o introspeccin.
. Signos neurolgicos blandos como reflejo glabelar, temblor, movimientos anormales,
discinesia tarda, hipoacusia, rigidez.

ALARCN R.

Alteraciones Psiquitricas en la Demencias, en Arango LJC, Fernndez GS y Ardila A, Las

Demencias: Aspectos Clnicos, Neuropsicolgicos y Tratamiento, Ed. Manual Moderno,


Mxico,Mxico, 2003.
ALMEIDA OP, HOWARD R, LEVY R, DAVID AS. Psychotic states arising in late life (late
paraphrenia). The role of risk factors. Br J Psychiatry, 1995; 166: 215-228, AMORES GF.
Funciones Cognitivas Superiores, Taller Master de Psicogeriatra, Universidad Autnoma de
Barcelona, Barcelona, Febrero, 2004, CASTLE D, MURRAY RM.
The epidemiology of late onset schizophrenia. Schizophr Bull, 1993; 19: 691-700
CHRISTENSON R, BLAZER DG.
Epidemiology of persecutory ideation in an elderly population in the community. Am J
Psychiatry, 984; 141: 1088-1091.

15.- Mujer de 42 aos. Acude a consulta por cefalea y acfenos. Tiene antecedente de DM
en padre. EF: peso 78 kg, estatura 1.62, permetro abdominal 108 cm, TA: 140/100.
Laboratorio: glucosa de ayuno 116, prueba de tolerancia a la glucosa, resultado a las 2 horas
de 189.
Los diagnsticos que se establecen en la paciente son:
a)
b)
c)
d)

Obesidad y diabetes mellitus


Sndrome metablico con intolerancia a la glucosa
Obesidad e intolerancia a la glucosa
Sndrome metablico y diabetes mellitus

GLUCOSA:
<100 mg/dl = normal
100 y < 126 = glucosa de ayuno
alterada (GAA)
126 mg/dl = diabetes mellitas
Glucosa a las 2 horas postcarga:

140 mg/dl = normal


140 a 199 mg/dl = intolerancia a la glucosa
200 mg/dl = diabetes mellitus

El sndrome metablico es la agrupacin de diferentes factores de riesgo asociados con el


sndrome de resistencia a la insulina. En la literatura existen variaciones importantes en la
prevalencia de este sndrome, dependiendo de los criterios o definicin que se empleen
para su diagnstico. En la actualidad hay ms de cinco definiciones del sndrome
metablico; la descrita en 2001 y su actualizacin en 2005 por el Programa Nacional de
Educacin para el Colesterol, Panel de Tratamiento del Adulto III (NCEP-ATPIII), es la
ms empleada en la prctica clnica. Esta definicin establece que se requiere la
presencia de tres o ms de los siguientes cinco criterios: glucosa en ayuno 100
mg/dl, triglicridos 150 mg/dl, colesterol HDL bajo (< 40 mg/dl en el hombre o <
50 mg/dl en la mujer), tensin arterial 130/85 mm Hg o en tratamiento para la
hipertensin y obesidad abdominal detectada mediante medicin del permetro
abdominal (> 102 cm en hombres y > 88 cm en mujeres).1 La obesidad ha alcanzado
proporciones epidmicas en los pases occidentales y, por lo tanto, es un importante
problema de salud. De hecho, Mxico ocupa el segundo lugar a nivel mundial en
sobrepeso y obesidad. La resistencia a la insulina que se promueve por la obesidad
abdominal o fenotipo de obesidad androide se considera la responsable de algunos
factores de riesgo que se agrupan en el sndrome metablico.2
Estudios recientes indican que la prevalencia del sndrome metablico en la poblacin
general en Mxico es de aproximadamente 26.6 %, similar a los resultados informados por
la Tercera Encuesta de Evaluacin Nacional en Salud y Nutricin en poblacin de Estados
Unidos, que fue de 26.7 %.3 Sin embargo, la prevalencia aumenta conforme avanza la edad,
llegando a ser hasta de 44 % en individuos mayores de 50 aos.4

El sndrome metablico no slo incluye algunos factores de riesgo cardiovascular


tradicionales, sino otros componentes que representan aspectos involucrados en el
desarrollo y progresin de enfermedades cardiovasculares, como la disfuncin endotelial,
marcadores de inflamacin y alteraciones en la coagulacin. La sinergia de la combinacin
de estos factores se considera causante del aumento en el riesgo de morbilidad y
mortalidad asociada con enfermedades cardiovasculares en este grupo de pacientes. Los
pacientes que completan criterios para sndrome metablico tienen tres veces ms riesgo
de padecer enfermedad arterial coronaria y cerebrovascular.5 Ms an, en pacientes con
enfermedad vascular sintomtica se encontr que el sndrome metablico se asociaba con la
extensin del dao vascular.6,7
El sndrome metablico es uno de los principales problemas de salud pblica del siglo XXI.
El diagnstico es sencillo, principalmente con base en datos clnicos y bioqumicos
sistemticos, los cuales estn al alcance de cualquier sistema de salud.

Sndrome metablico, impacto clnico y angiogrfico en pacientes con sndrome coronario


agudo
Volumen 78, No. 2, Marzo-Abril 2010

Alejandra Madrid-Miller,* Antonio Alcaraz-Ruiz,* Gabriela Borrayo-Snchez,**


Eduardo Almeida-Gutirrez,* Rosa Mara Vargas-Guzmn,* Ricardo Juregui-Aguilar***

16. - Male, 25, is brought to ER after a solvent factory accident 30 minutes ago, he shows
burn injuries on both legs, genitals, anterior and posterior surface of lower trunk. Select
the estimation of body surface area in burns according to Rule of nines.
a)
b)
c)
d)

55%
60%
25%
46%

Esta

ilustracin

muestra

la

Regla

de

los

Nueve

para

reas

corporales.

Divide el cuerpo en secciones que representan el nueve por ciento del total del rea de la
superficie del cuerpo (TBSA). Puede ser utilizado conjuntamente con los pacientes adultos
quemados para determinar el TBSA quemado. Las secciones incluyen la cabeza y cuello,
brazos, torso (pecho, abdomen anterior, regin dorsal y regin lumbar), perineo y piernas.
A mayor extensin, mayor gravedad. Las quemaduras en cara, genitales, manos o pies por si
mismas ya son graves. En cualquier caso, sea cual sea el rea quemada, si supera el 15%
tambin se considerar grave.

Se estima que el reparto en porcentaje sera el siguiente:


Cabeza y cuello: 9%
Tronco anterior: 18% (Trax 9 y abdomen 9)
Tronco posterior: 18% (parte alta espalda 9 y baja 9)
miembros inferiores (cada uno): 18%
miembros superiores (cada uno): 9%
perin: 1%

Libro: Urgencias Medicas De Flint de Cain Editorial: MC GRAW-HILL EDUCATION ...

17.- Mujer de 26 aos, con tumor anexial de 6 cm lquido, dolor abdominal, fiebre,
leucorrea, con historia de cervicovaginitis de repeticin y dispareunia crnica, ltima
menstruacin hace una semana. El diagnstico ms probable ser:
a)
b)
c)
d)

Cistadenoma
Embarazo ectpico
Quiste de ovario
Enfermedad plvica inflamatoria

La EIP puede cursar con los siguientes sntomas:

Dolor abdominal bajo (incluyendo dolor anexial, dispareunia). Es el sntoma ms


frecuente (95%)
Aumento del flujo vaginal, flujo de caractersticas anormales (74%)
Sangrado anormal (intermestrual, poscoital) (45%)
Sntomas urinarios (35%)
Vmitos (14%)
Es posible la ausencia de sntomas

Y en ella podemos encontrar estos signos:

Dolor a la movilizacin del cuello, dolor anexial en la exploracin vaginal bimanual


(99%)
En el examen con espculo observamos cervicitis y descarga endocervical purulenta
(74%)
Fiebre (> 38 C) (menos del 47%).
Masa plvica: sugiere abceso tuboovrico (ATO)
Peritonitis

CRITERIOS CLINICOS PARA EL DIAGNOSTICO DE SALPINGITIS


a. Dolor abdominal con o sin rebote.
b. Sensibilidad a la movilizacin del crvix.
c. Sensibilidad anexial.
Los tres criterios anteriores son necesarios para establecer el diagnstico, con uno o ms
de los siguientes:
a. Extendido de Gram de endocrvix positivo, para diplococos gram negativos
intracelulares

b. Temperatura mayor de 38C


c. Leucocitosis (mayor de 10.000 por c.c.)
d. Material purulento (positivo para leucocitos) en la cavidad peritoneal
obtenido por culdocentesis o laparoscopia.

Establecido el diagnstico clnico de EPI, se debe hacer la definicin del estado clnico y
anatmico de la patologa plvica:
a) No complicada (limitada a trompas u ovarios)
1) Sin peritonitis plvica
2) Con peritonitis plvica
b) Complicada (masa inflamatoria o absceso que compromete trompa (s) u ovario (s)
1) Sin peritonitis plvica
2) Con peritonitis plvica

Referencia:
Beigi RH, Wiesenfeld HC. Pelvic inflammatory disease: new diagnostic criteria and
treatment. Obstet Gynecol Clin Norh Am. 2003; 30 (4): 777 93
Center for Disease Control. Guidelines for treatment of sexually transmited diseases.
MMWR Recomm Rep. 2002 May 10;51(RR-6):1-78
Center for Disease Control. Guidelines for prevention and management (MMWR. 40: 1 - 25
1991) Pelvic inflammatory disease: guidelines for prevention and management.
MMWR Recomm Rep. 1991 Apr 26;40(RR-5):1-25.
Hager WD, Eschenbach DA, Spence MR, Sweet RL. Criteria for diagnosis and grading of
salpingitis. Obstet Gynecol. 1983 Jan;61(1):113-4.
Prodigy Guidance. Pelvic inflammatory disease. [Internet]. UK : NHS, Department of
Health; 2003. [Acceso 18 de Junio de 2005]. Disponible en:

Ross J. Pelvic inflammatory disease. Clin Evid. 2004 Dec;(12):2259-65.


Royal College of Obstetricians and Gynaecologists. Pelvic Inflammatory Disease. Guideline
n 32. [Internet]. RCOG; Mayo 2003. [Acceso 18 de Junio de 2005].

18.- En el cunero, se observa que un neonato presenta asimetra de pliegues glteos. A la EF


la cadera, una de ellas puede ser fcilmente luxada posteriormente con un click y
regresada a su posicin normal con un sonido parecido. La familia se encuentra preocupada
porque el primer hijo tuvo el mismo problema. Cul es el diagnstico ms probable?

a)
b)
c)
d)

Displasia de cadera
Enfermedad de Legg-Perthes
Artritis sptica
Parto traumtico

La falta de relacin normal en las estructuras que forman una articulacin se conoce como
luxacin. En el caso de la cadera, la cabeza del fmur no encaja debidamente en su cavidad
(acetbulo).
El desarrollo del acetbulo ser normal siempre que la cabeza del fmur permanezca en
posicin correcta y el esfuerzo muscular sea adecuado.
Siempre que esto no se cumpla, se producir una alteracin entre el ctilo y la cabeza
femoral, dando lugar a una alteracin con el resultado de luxacin de la cadera.
DIAGNSTICO: EXPLORACIN FSICA
a) Maniobras de Ortolani y Barlow - click audible-.
Ortolani: el click se produce cuando la fvea de la cabeza del fmur encuentra la cresta
cotiloidea caminando sobre ella. La limitacin a la abduccin desaparece, esto es signo de
reduccin.
Barlow: se realiza la aduccin de la cadera, que tras una ligera presin longitudinal sobre el
fmur, produce una luxacin.
La prueba de Ortolani traduce una luxacin y la de Barlow una cadera luxable.
b) Asimetra de pliegues -no valorable en luxaciones bilaterales.
c) Limitacin a la abduccin.
d) Dismetra.

Referencia:
Garca Portabella, M.
Luxacin congnita de cadera antes de los tres meses de edad. 2001.
Garcia-Siso Pardo, J. M.
Displasia del desarrollo de la cadera.
(Parte I). Peditra Rural y Extrahospitalaria.
Vol. 32. N 304 Pgs. 481-491. 2002.
Graf, R.

Classification of hip joint dysplasia by means of sonography.


Arch Orthop Trauma Surg 102:248-255, 1984.
Edeiken, J.
Luxacin congnita de cadera.
Diagnstico Radiolgico de las Enfermedades de los Huesos.
Ed. Mdica Panamericana. Pgs. 388-392. 1977.

19.- Femenino de 18 aos originario de Tabasco, quien inicio con fiebre elevada, cefaleas,
mialgias generalizadas y artralgias, se diagnostica dengue clsico, el tratamiento de
eleccin en esta patologa es:
a)
b)
c)
d)

Aciclovir
Cristaloides y plasma
Antivirales e hidratacin con cristaloides.
Sintomtico

DENGUE CLSICO
Las primeras manifestaciones clnicas son de inicio abrupto tras 2-7 das de incubacin.
Se caracterizan por fiebre elevada (39-40C), cefaleas, mialgias intensas generalizadas y
artralgias con dolor cervical y lumbar, anorexia, gran astenia, nuseas, vmitos y dolor
abdominal. Los sntomas respiratorios (tos, rinitis, faringitis) son frecuentes. Se puede
presentar una erupcin cutnea mculo-papular, que aparece al comienzo de la fiebre o
coincide con un segundo pico febril a los 3-5 das. Pueden observarse poliadenopatas,
granulocitopenia, linfocitosis relativa y trombopenia.
Algunos de los aspectos clnicos dependen fundamentalmente de la edad del paciente.
El dolor abdominal generalizado ha sido observado ms frecuentemente en nios.
En adultos, al final del perodo febril se pueden presentar manifestaciones hemorrgicas
de poca entidad, como epstaxis, petequias, gingivorragias, y en casos ms raros
hematemesis, melenas o hematurias. Si bien el Dengue Clsico es usualmente benigno y
autolimitado, se asocia con gran debilidad fsica y algunas veces con una convalescencia
prolongada, pudiendo estar presentes las manifestaciones hemorrgicas, que no son
exclusivas de la entidad clnica llamada Fiebre Hemorrgica de Dengue.
La enfermedad cursa con viremia precoz y breve (desde un da antes de los sntomas hasta
3-5 das despus aproximadamente), lesiones de engrosamiento endotelial, edema e
infiltracin mononuclear en torno a los pequeos vasos.

Tratamiento

El tratamiento en el dengue clsico es sintomtico, no deben usarse saliclicos.


La prevencin y vigilancia

Hidratacin oral
Tratamiento del Shock EN CASO NECESARIO.
Rehidratacin parenteral si se presenta: intolerancia a la va oral, deshidratacin moderada
o grave, hematocrito en aumento o derrames cavitarios.
Los corticoides no demostraron ser efectivos
Antibiticos no estn indicados

20.- Un nio de 12 aos, varn, acude al centro de urgencias 30 mins. Despus de haber
ingerido comprimidos de sulfato ferroso de 325 mg. La madre indica que faltan 20
comprimidos del frasco. Cada comprimido tiene 65 mg de hierro elemental. El nio ha
vomitado una vez y su comportamiento parece totalmente normal. Los hallazgos de la
exploracin fsica no arrojan nada importante. Pesa 13 kg. Cul de los siguientes
enunciados sobre el estado del nio es el correcto?
a)
b)
c)
d)

La dosis de hierro no debera causar secuelas clnicas.


Ese episodio nico de vmito probablemente no esta relacionado con la ingestin.
Aunque ha vomitado una vez debera administrarse ipecacuana.
Los catrticos son ineficaces en intoxicaciones.

En situaciones en las que no hay testigos de la ingestin debe presuponerse inicialmente el


peor escenario posible para estimar la toxicidad potencial. La dosis mxima de hierro
ingerida por este nio es de 65 mg de hierro elemental x 20 pldoras / 13 kg = 100 mg /kg
que es una dosis potencialmente grave. Entre los sntomas causados por la ingestin grave
de hierro estn los vmitos por tanto en este caso el vmito debe considerarse relacionado
con la ingestin. Ello implica un riesgo potencial de secuelas graves. En conjunto la ingestin
debe valorarse como clnicamente importante.
Como un solo episodio de vmitos no vaca el estmago de manera suficiente debe
administrarse ipecacuana o alternativamente proceder a un lavado gstrico. El carbn
activado no adsorbe el hierro y no es necesario. Los catrticos sirven de ayuda una vez que
e ha vaciado el estmago. Los niveles de hierro srico y de capacidad de enlace del hierro
deben estimarse segn las circunstancias. Tambin es til obtener una radiografa de
abdomen por cuanto las pldoras son radioopacas y puede deducirse as algn indicio de su
permanencia en el tracto GI despus del tratamiento inicial.

1.- Montoya-Cabrera MA. Intoxicaciones y envenenamientos en nios. Mxico,


Intersistemas , 2000.
2.-Montoya CMA. Toxicologa clnica. 2. Ed, Mxico, Mndez Editores, 1997

21.- Mujer de 37 aos. Acude a consulta por presentar cefalea, cansancio e irregularidades
menstruales con ritmo de 36 a 50 x 2-3 das. No se ha podido embarazar despus de 18
meses de actividad sexual regular. No tiene antecedentes importantes. EF: Campos visuales
normales, tiroides aumentada de tamao una vez y aumentada de consistencia, no tiene
galactorrea. Resto normal. Laboratorio: qumica sangunea, Bh y electrolitos normales.
Prolactina 47 ng/dL (< 25), perfil tiroideo: TSH 18 mUI/ml, T4t: 50 nmol/L (57.9 a 154.4),
T4L: 7.7 pmol/L (9 a 24), T3T: 1.06 nmol/L (1.2 a 2.9), T3L: 1.96 pmol/L (3 a 6.31)
El diagnstico es:

a)
b)
c)
d)

Hiperprolactinemia
Hipertiroidismo
Sndrome de ovarios poliqusticos
Hipotiroidismo primario

DIAGNOSTICO
Inicialmente el hipotiroidismo se diagnosticaba mediante la cuantificacin por tcnicas de
Radio Inmuno Anlisis (RIA) de las hormonas circulantes triyodotironina y tiroxina; el
proceso era lento y sometido a muchos factores de error que hacan su sensibilidad y
especificidad poco confiables. Posteriormente, se desarrollaron tcnicas para la medicin
de la TSH hipofisiaria igualmente mediante el RIA lo que mejor en forma importante la
sensibilidad para el diagnstico de esta enfermedad; sin embargo, los niveles de deteccin
de la prueba se encontraban en el orden de 1 IU/ml lo que haca que la prueba no fuera
sensible para valores menores de 1 IU/ml. Debido a esto se crearon tcnicas de segunda
generacin mediante la cuantificacin de TSH por anticuerpos monoclonales y RIA, el
IRMA (Immuno Radiometric with Monoclonal Antibodies) que permiti detectar valores de
TSH en rangos de 0.1 IU/ml; posibilitando desde entonces diagnosticar pacientes con
hipertiroidismo primario; pero con la limitante de que para esta tcnica era imposible
detectar valores de TSH menores de 0.1 IU/ml por lo que se creo la medicin de TSH
mediante quimioluminiscencia o mtodos enzimticos, es decir las tcnicas de tercera
generacin, las cuales pueden detectar valores de TSH de 0.01 IU/ml; con lo que se logra
el espectro ideal para una prueba de laboratorio que tiene la capacidad de diagnosticar
tanto la hipofuncin como la hiperfuncin(20).

Adems el avance no slo fue en la medicin de TSH sino tambin en las hormonas tiroideas
que han evolucionado simultneamente con la TSH y ya se miden incluso las fracciones
libres de hormonas y las fracciones totales, lo que ha facilitado el manejo de estos
pacientes. Gracias a esta evolucin en tcnicas de laboratorio, el diagnstico de
hipotiroidismo primario es bastante sencillo. Niveles de TSH superiores al valor mximo de
la tcnica seran diagnsticos de la disfuncin; pero no es tan fcil. Cuando tenemos un
paciente con toda la sintomatologa del hipotiroidismo y la TSH se encuentra elevada el
diagnstico es obvio; pero podemos tener pacientes con sntomas muy inespecficos como
depresin y con examen fsico normal a quienes se les encuentran valores de TSH por

encima del lmite superior y con hormonas tiroideas normales. Se trata de un hipotiroidismo
o es un valor ligeramente elevado ocasional de una persona sana(21). Igualmente tenemos
otra circunstancia que ha sido descrita con mayor frecuencia: pacientes con valores de
TSH en el lmite superior normal y con dislipidemia a quienes se les da tratamiento con
hormonas tiroideas y su dislipidemia se corrige manteniendo valores de TSH en rangos
normales. Todas las circunstancias anteriores han hecho que aparezca en el hipotiroidismo
primario la expresin de hipotiroidismo subclinico, que ha sido objeto de reuniones y
congresos dedicados exclusivamente a este tema. La sociedad Europea de Tiroides hace
algunas recomendaciones para el manejo de esta situacin que se consideran tiles como
gua (Tabla ).

Tabla. Enfoque del paciente con disfuncin tiroidea de acuerdo a los niveles de hormona
estimulante de la tiroides (TSH).

Si TSH < 0.4m U/L

Si
TSH
0.4 a 2.0
mU/L
Si TSH 2.01 a 5.0mU/L

Medir T3 y T4 totales o
libres para diagnostico
de hipertiroidismo.

Normal,
Repetir
cada cinco
aos

Medir T4 libre y anticuerpos


antitiroideos

Si TSH
mU/L

>

5.0

Dar tratamiento
para
hipotiroidismo

1. Si AAT (-) y T4 libre es


normal repetir screening cada
ao. Si TSH es > 4.0mU/l en dos
ocasiones dar tratamiento
2. Si AAT (+) y/o T4 libre esta
baja o normal baja tratar si
TSH es mayor de 3.0 mU/l y
observar a los otros
Tomado de Koutras DA. Subclinical hypothyroidism. En G. Hennemann, E.P. Krenning,
Thyroid International Merck KGaA, Darmstadt 1999 (3), 6-9

22.- Hombre de 54 aos, acude a consulta por descontrol glucmico. Tiene antecedente de
DM tipo 2 de 13 aos de evolucin controlada con diferentes hipoglucemiantes orales.
Desde hace un mes est bajando de peso y tiene polidipsia y poliuria a pesar de tomar su
tratamiento con metformn 850 mg 3 veces al da y glibenclamida tab 5 mg, 4 tabletas
diarias, adems de la dieta. EF: peso 68 kg, estatura 1.70, TA 140/80, FC 96x. Glucosa:
289 mg, hemoglobina glucosilada de 11%. La conducta teraputica ms apropiada es:
a)
b)
c)
d)

Aumentar dosis de glibenclamida


Iniciar insulina de accin intermedia o prolongada
Iniciar insulina rpida por requerimientos
Aadir un tercer hipoglucemiante

SELECCIN DEL TRATAMIENTO


DIETA + EJERCICIO
Metas no
obeso
BIGUANIDA

Metas s
continuar

delgado
SULFONIUREA

Metas s

Metas no

SELECCIN DEL TRATAMIENTO


MEZCLAS SU + BG

Metas no
Metas s
TX COMBINADO
HO da
Insulina noche

Continuar

Metas no
INSULINA

Medical Management of Hyperglycemia in Type 2 Diabetes: A Consensus Algorithm for


the Initiation and Adjustment of Therapy.

Dieta, ejercicio, educacin y automonitoreo

HbA1c > 9%

2 hipoglucemiantes
Biguanida + secretagogo
Biguanida + Tiazolidinediona
HO + insulina

Insulina basal
o preprandial o
ambas

Cheng YY A. CMAJ 2005; 172(2):213-26.

23.- Una mujer de 43 aos inicia recientemente con fatiga, somnolencia, piel seca,
estreimiento y aumento de peso de 5 kg. Su tiroides est firme y tiene el doble del
tamao normal. Cul de las siguientes pruebas de laboratorio confirma el presunto
diagnstico de hipotiroidismo?

a)
b)
c)
d)

Tiroxina srica (T4)


Triyodotironina srica (T3)
Captacin de resina T3
Hormona estimulante de tiroides (TSH) en suero

Hipotiroidismo
La instauracin es habitualmente lenta y progresiva. Los sntomas se relacionan con una
disminucin en la actividad funcional de todos los sistemas del organismo. Los ms clsicos
son cansancio, intolerancia al fro (carcter muy friolero), apata e indiferencia, depresin,
disminucin de memoria y de la capacidad de concentracin mental, piel seca, cabello seco y
quebradizo, fragilidad de uas, palidez de piel, aumento de peso, estreimiento pertinaz y
somnolencia excesiva. En situaciones extremas puede evolucionar hacia la insuficiencia
cardiaca, la hinchazn generalizada (mixedema), insuficiencia respiratoria y abocar al coma
mixedematoso con prdida de conocimiento que conlleva un alto grado de mortalidad.
Al igual que el resto de enfermedades del tiroides, el hipotiroidismo es ms frecuente en
el sexo femenino. Es a partir de los 40-50 aos cuando las mujeres tienden a desarrollar
con ms frecuencia hipotiroidismo de causa autoinmune (tiroiditis de Hashimoto). El
periodo postparto es igualmente propenso a la aparicin de este problema. La ciruga de
tiroides y la aplicacin de yodo radioactivo representan situaciones de riesgo para el

desarrollo de hipotiroidismo, lo que obliga a controlar evolutivamente la funcin tiroidea en


estos casos.Los recin nacidos de madres hipertiroideas, hayan recibido o no tratamiento

antitiroideo durante la gestacin, deben ser evaluados en este sentido. Las personas en las
que se detectan anticuerpos antitiroideos (antimicrosomales, antitiroglobulina) tienden a
desarrollar con el tiempo alteraciones de la funcin tiroidea, por lo que deben ser evaluados
crnicamente de forma peridica.
La determinacin de TSH es el parmetro ms sensible para el diagnstico del
hipotiroidismo. Su elevacin es indicativa de que la funcin del tiroides es insuficiente.
Este fenmeno se produce antes de que comiencen a descender en la sangre las
concentraciones de hormonas tiroideas. Generalmente, en el hipotiroidismo establecido,
adems de la elevacin de TSH, se produce un descenso de T4. El nivel de T3 con
frecuencia se encuentra dentro de la normalidad. As pues, cuando aparecen sntomas
sugestivos, el mdico solicitar una determinacin de TSH que es el mejor mtodo para
descartar que exista hipotiroidismo. Puede acompaarse de una determinacin de T4 y de
anticuerpos antitiroideos si se desea conocer si la causa se debe a fenmenos de
autoinmunidad. En los casos de hipotiroidismo secundario debido a disminucin de la
secrecin de TSH por parte de la hipfisis, el diagnstico se basa en confirmar
concentraciones disminuidas de T4 y TSH en la sangre. Cuando la elevacin de TSH se
acompaa de niveles normales de T4 la condicin es conocida con el nombre de
hipotiroidismo subclnico. Si existe bocio puede ser conveniente realizar una ecografa
tiroidea. Cuando existe sospecha de alteraciones en el desarrollo de la glndula o de
deficiencia enzimtica, puede ser til obtener una gammagrafa tiroidea. Si se confirma un
diagnstico de hipotiroidismo de causa autoinmune, es habitual evaluar la asociacin de
alteraciones en otras glndulas como las suprarrenales, paratiroides o gnadas.
REFERENCIAS:
(1.) Anderson R, Harnes J. 1975. Thyroid hormones secretion rates in growing and mature
goats. J Anim Sci 40: 11301135.

(2.) Anke M, Henning A, Grun M, Partschefeld M, Groppel B. 1977. Der einluss des mangan,
zink, kupfer, jod, selen, molybdan und nickelmangels aauf die fortpflanzuggsleistung des
wiederkauers. Mathem Natur Reihe (Leipzig) 26: 283-292.

(3.) Balbuena O. 2003. Nutricin Mineral del Ganado. Sitio Argentino de Produccin Animal:
1-5, www.produccionanimal. com.ar.

(4.) Beckett GJ, Beddows SE, Morrice PC, Nicol F, Arthur JR. 1987. Inhibition of hepatic
deiodination of thyroxine is caused by selenium deficiency in rats. Biochem J 248: 443447.

(5.) Brem JJ, Pochon DO, Roux JP, Trulls H. 1998. Exploracin diagnstica de la funcin
tiroidea en ovinos. Rev Vet 8/9: 23-26.

(6.) Castillo V. 2001. Cambios de la funcin tiroidea en cachorros alimentados con dietas
comerciales con alto contenido de yodo. On line: http://www.idealibrary.com.

24.- A travs de un estudio de casos y controles se quiere conocer si existe asociacin


entre tabaquismo y cncer de lengua. La variable tabaquismo se medir como Leve (uno a
10 cigarrillo en 24 hrs), moderado (11 a 20 cigarrillos en 24hrs) severo (ms de 21
cigarrillos en 24 hrs). Como podemos clasificar esta variable:

a)
b)
c)
d)

Cuantitativa continua
Cuantitativa discreta
Cualitativa ordinal
Cuantitativa discontinua

En las variables de tipo ordinal las observaciones se clasifican y ordenan por categoras
segn el grado en que los objetos o eventos poseen una determinada caracterstica. Por
ejemplo, se puede clasificar a las personas con respecto al grado de una enfermedad en
leve, moderado o severo.

Moreno A. Principales medidas en epidemiologa. Rev Salud Pblica Mex, 2000;42(4): 338

25.- Femenino de 20 aos que acude a consulta externa, refiere que desde hace mas de un
ao presenta astenia, cansancio, prdida de apetito y dificultades para concentrarse en los
estudios. Al interrogatorio refiere que a perdido inters en los estudios, frecuenta menos
a sus amigos, con pesimismo en la mayora de sus actividades. Cul es el diagnstico ms
probable?

a) Anorexia nerviosa
b) Trastorno de ansiedad.
c) Trastorno Distmico
d) Depresin mayor

Criterios para el diagnstico de


F34.1 Trastorno distmico (300.4)

A. Estado de nimo crnicamente depresivo la mayor parte del da de la mayora de los das,
manifestado por el sujeto u observado por los dems, durante al menos 2 aos.
Nota: En los nios y adolescentes el estado de nimo puede ser irritable y la duracin debe
ser de al menos 1 ao.
B. Presencia, mientras est deprimido, de dos (o ms) de los siguientes sntomas:
1. Prdida o aumento de apetito
2. Insomnio o hipersomnia
3. Falta de energa o fatiga
4. Baja autoestima
5. Dificultades para concentrarse o para tomar decisiones
6. Sentimientos de desesperanza
C. Durante el perodo de 2 aos (1 ao en nios y adolescentes) de la alteracin, el sujeto no
ha estado sin sntomas de los Criterios A y B durante ms de 2 meses seguidos.
D. No ha habido ningn episodio depresivo mayor durante los primeros 2 aos de la
alteracin (1 ao para nios y adolescentes); por ejemplo, la alteracin no se explica mejor
por la presencia de un trastorno depresivo mayor crnico o un trastorno depresivo mayor,
en remisin parcial.
Nota: Antes de la aparicin del episodio distmico pudo haber un episodio depresivo mayor
previo que ha remitido totalmente (ningn signo o sntoma significativos durante 2 meses).
Adems, tras los primeros 2 aos (1 ao en nios y adolescentes) de trastorno distmico,
puede haber episodios de trastorno depresivo mayor superpuestos, en cuyo caso cabe
realizar ambos diagnsticos si se cumplen los criterios para un episodio depresivo mayor.
E. Nunca ha habido un episodio manaco, un episodio mixto o un episodio hipomanaco y
nunca se han cumplido los criterios para el trastorno ciclotmco.
F. La alteracin no aparece exclusivamente en el transcurso de un trastorno psictico
crnico, como son la esquizofrenia o el trastorno delirante.
G. Los sntomas no son debidos a los efectos fisiolgicos directos de una sustancia (p. ej.,
una droga, un medicamento) o a enfermedad mdica (p. ej., hipotiroidismo).
H. Los sntomas causan un malestar clnicamente significativo o deterioro social, laboral o
de otras reas importantes de la actividad del individuo.

Especificar si:
Inicio
temprano:
Si
el
inicio
es
antes
de
Inicio tardo: si el inicio se produce a los 21 aos o con posterioridad.

los

21

aos

Especificar (para los ltimos 2 aos del Trastorno distmico):


Con sntomas atpicos
DSM IV
MANUAL DIAGNSTICO Y ESTADSTICO DE LOS TRASTORNOS MENTALES
(American Psychiatric Association)

26.- Mujer de 22 aos que acude a consulta por prdida de peso, diarrea, palpitaciones,
temblor y labilidad emocional. No tiene antecedentes importantes. Refiere evacuaciones
diarreicas de 5 a 7 por da, sin moco o sangre.
EF: peso 52 kg, estatura 1.59, FC 108x, TA: 120/65, retraccin palpebral, hiperemia
conjuntival y de carncula, cuello con tiroides aumentada de tamao 3 veces de forma
difusa, extremidades superiores con hiperhidrosis palmar, temblor fino distal, reflejos
exaltados.
Los estudios que debemos solicitar para hacer el diagnstico es:
a)
b)
c)
d)

Coproparasitoscpico, coprolgico y coprocultivo


Ultrasonido tiroideo
Pruebas de funcin tiroidea
Electrocardiograma y ecocardiograma

EVALUACIN DE UN PACIENTE CON SOSPECHA DE


ENFERMEDAD TIROIDEA

27.- Femenino de 26 aos su padecimiento actual inici hace 4 aos, cuando sus ciclos
menstruales empezaron a ser irregulares. Su ritmo actual es de 40-90 x 3-4. FUR: hace 3
meses. Inici vida sexual a los 23 aos y no ha podido embarazarse. En la exploracin fsica
encontramos la piel ligeramente seca, hay salida de lquido blanquecino escaso a la
expresin del pezn izquierdo y tiene giordano positivo derecho.
1. El estudio que es de mayor utilidad para aclarar el diagnstico es:
a)
b)
c)
d)

Prueba de embarazo
LH, FSH y estrgenos
Tomografa de crneo
Prolactina

En toda paciente con galactorrea, trastornos menstruales, hirsutismo, disminucin de la


libido o infertilidad, deben determinarse los niveles de PRL plasmtica, entre el tercer y
quinto da del ciclo menstrual, si este es regular o en cualquier momento, si presenta
amenorrea u oligomenorrea.2 Si en la primera determinacin se obtienen cifras elevadas
debe repetirse y si se confirma nuevamente, se establece el diagnstico de
hiperprolactinemia. Una historia clnica com-pleta, con un interrogatorio y examen fsico
detallados, permitir orientarnos hacia la causa de la hiperprolactinemia. La primera causa
que se debe descartar antes de emprender otros estudios, es el embarazo. El uso de
estrgenos, anticonceptivos orales y drogas que aumentan la secrecin de PRL debe
precisarse en el interrogatorio, as como la presencia de quemaduras u otras lesiones en
trax que se buscan en el examen fsico.
Pocas mediciones hormonales tienen el significado clnico que se observa con la prolactina.
La tcnica est bien estandarizada y presenta bastante confiabilidad; la muestra de sangre
se puede obtener a cualquier hora del da y debido a las variaciones fisiolgicas (efecto del
ejercicio, alimentos, irritacin de la pared costal) cuando se obtiene una cifra de prolactina
por arriba de lo normal, es necesario repetir el anlisis.
Varios frmacos pueden producir una moderada elevacin en los niveles de prolactina, sin
alcanzar los valores que se encuentran en prolactinoma; las drogas ms comunes son
metoclopramida, fenotiazinas, risperidona, verapamil, metildopa, reserpina y los inhibidores
de MAO.
Exmenes: prolactina plasmtica (prolactinemia), tirotropina (hormona estimulante de la
glndula tiroides o TSH), test de embarazo.
Prolactina menor de 100 ng/ml: baja sospecha de prolactinoma.
Prolactina entre 100 y 300 ng/ml: mayor posibilidad de prolactinoma.
Prolactina superior a 300 ng/ml: alta sospecha de prolactinoma. Es precisa resonancia
magntica nuclear (RNM) de hipfisis (detecta prolactinoma mayor de 3mm).

En el 50% de los casos de hiperprolactinemia se detecta tumor: microadenomas o


macroadenomas; la prolactina suele encontrarse entre 100 y 200ng/ml; los macroadenomas
tienen efecto de masa y causan hipopituitarismo; los microadenomas no presentan efecto
de masa y slo producen hipogonadismo.
Referencias:
1. Schlechte JA. Prolactinoma. N Engl J Med 2003;349:2035-2041.
2. Zrate A, Canales ES, Jacobs LS, Soria J, Daughaday WH. Restoration of ovarian
function in patients with the amenorrhea-galactorrhea syndrome after long-term therapy
with L-Dopa. Fertil Steril 1973;24:340.
3. Tyson JE, Carter JN, Andreassen B, Huth J, Smith B. Nursing mediated
prolactin and luteinizing hormone secretion during puerperal lactation. Fertil
Steril 1978;30:154.
4. Schlechte JA, Sherman BM, Chapler FK, VanGilder J. Long-term followup of women
with surgically treated prolactin-secreting pituitary tumors. J
Clin Endocrinol Metab 1986;62:1296-301.
5. Losa M, Mortini P, Barzaghi R, Gioia L, Giovanelli M. Surgical treatment of
prolactin-secreting pituitary adenomas: early results and long-term outcome. J Clin
Endocrinol Metab 2002;87:3180-3186.

28.- Un hombre de 55 aos presenta dolor precordial que le apareci cuando estaban en
reposo; se irradio al cuello y al hombro izquierdo, refiere que tuvo 20 minutos de duracin
y cedi , tiene antecedentes de obesidad, Diabetes Mellitus, hipertensin y sedentarismo,
el diagnostico mas probable es:

a)
b)
c)
d)

Angina inestable.
Infarto agudo al miocardio.
Angina estable.
Pericarditis.

Entendemos por angina de pecho, o angor, un dolor torcico, generalmente retrosternal,


que puede ser descrito como opresivo, quemazn o simplemente una leve pesadez, y que
est motivado por la isquemia del miocardio. Este dolor o molestia puede irradiar o
presentarse nicamente en el cuello, mandbula, hombros, brazos, antebrazos, manos,
espalda o epigastrio. Rara vez se presenta por encima de la mandbula o debajo del
epigastrio. La isquemia miocrdica se presenta en ocasiones con sntomas distintos de la
angina como disnea, debilidad, fatiga o eructos. Estos sntomas son equivalentes anginosos
y suelen ser mas frecuentes en personas ancianas.

La situacin en que se presenta la angina inestable no parece relacionada con un mayor


trabajo cardaco. Es decir, la isquemia miocrdica no parece justificarse por un mayor
consumo miocrdico de oxgeno y, por lo tanto, la causa es una disminucin aguda del flujo
sanguneo coronario. Clsicamente se denomina angina inestable a la que se presenta en las
siguientes circunstancias:
1. Angina de reposo: ocurre en reposo o con un mnimo esfuerzo.
2. Angina de comienzo reciente: inicio de los sntomas en el ltimo mes, en un paciente
previamente asintomtico, y de aparicin con esfuerzos mnimos.
3. Angina progresiva: En un paciente con angina estable previa, los sntomas se
presentan con esfuerzos menores, son cada vez mas frecuentes o su duracin es
ms prolongada.
La presentacin clnica de la angina inestable puede ser idntica a la de un infarto agudo de
miocardio (IAM), la diferencia es conceptual: si hay necrosis miocrdica hablamos de
infarto agudo de miocardio. Los datos que podemos obtener de la historia clnica, la
exploracin fsica y el electrocardiograma, con frecuencia no permiten diferenciar entre
angina inestable e infarto agudo de miocardio. La elevacin de marcadores de dao
miocrdico como la creatnfosfoquinasa (CPK) o la troponina T o I, identifican una necrosis
miocrdica y, por lo tanto un IAM. Es por este motivo que actualmente tanto la angina
inestable como el IAM se agrupan bajo el trmino de sndrome coronario agudo. Segn la
presentacin electrocardiogrfica, el sndrome coronario agudo (SCA) se divide en SCA con
elevacin del segmento ST (frecuentemente evoluciona a un infarto con onda Q) y SCA sin
elevacin del segmento ST, que incluye a la angina inestable y la mayor parte de los casos
de IAM sin onda Q. Esta terminologa es la que actualmente se utiliza porque tiene la
ventaja de clasificar el cuadro clnico del paciente a partir de datos clnicos y
electrocardiogrficos que pueden obtenerse de modo rpido y sencillo.
LECTURA RECOMENDADA:
Guas clnicas para el manejo de la angina inestable e infartosin elevacin del ST.
Estratificacin del riesgo
Bibliografa Internacional
R. Marrn Tundidor*, P. Palazn Saura*, L. M. Claraco Vega*, C. Ascaso Martorell*,
J. Povar Marco*,
J. M. Franco Sorolla*, I. Calvo Cebollero**
*SERVICIO DE URGENCIAS Y **SERVICIO DE CARDIOLOGA-UNIDAD DE
HEMODINMICA Y CARDIOLOGA INTERVENCIONISTA.

29.- Acude a consulta un adolescente de 20 aos la cual refiere que desde hace varios
meses presenta astenia, cansancio, prdida de apetito y dificultades para concentrarse en
los estudios. Al interrogatorio refiere que a perdido inters en los estudios, frecuenta
menos a sus amigos, con pesimismo en la mayora de sus actividades. Cul es el diagnstico
ms probable?

a) Anorexia nerviosa
b) Trastorno de ansiedad.
c) Distimia.
d) Depresin mayor

LA DISTIMIA
La distimia es un estado de nimo crnicamente deprimido, menos grave que la depresin y
que no cumple los criterios para una depresin mayor, o lo hace slo en perodos muy
cortos. Su evolucin suele ser de ms de dos aos. Se caracteriza por un abatimiento
prolongado del estado de nimo en que el sujeto distmico se describe a s mismo como
triste o desanimado, perdiendo el inters por las cosas y vindose a menudo como intil
y poco interesante. Posee sntomas persistentes o intermitentes, de intensidad ms leve
comparacin a la depresin mayor. Aiskal (1983) la define como mal humor y se
caracteriza porque el individuo est habitualmente triste, introvertido, melanclico,
excesivamente consciente, incapaz de alegra y preocupado por su insuficiencia personal.
Los criterios de diagnstico de Distimia son los que a continuacin se detallan:
A. Estado de nimo crnicamente depresivo la mayor parte del da de la mayora de los das,
manifestado por el sujeto u observado por los dems, durante al menos 2 aos. Nota: En los
nios y adolescentes el estado de nimo puede ser irritable y la duracin debe ser de al
menos 1 ao.
B. Presencia, mientras est deprimido, de dos (o ms) de los siguientes sntomas:
1. Prdida o aumento de apetito
2. Insomnio o hipersomnia
3. Falta de energa o fatiga
4. Baja autoestima
5. Dificultades para concentrarse o para tomar decisiones
6. Sentimientos de desesperanza

C. Durante el perodo de 2 aos (1 ao en nios y adolescentes) de la alteracin, el sujeto


no ha estado sin sntomas de los Criterios A y B durante ms de 2 meses seguidos.
D. No ha habido ningn episodio depresivo mayor durante los primeros 2 aos de la
alteracin (1 ao para nios y adolescentes); por ejemplo, la alteracin no se explica mejor
por la presencia de un trastorno depresivo mayor crnico o un trastorno depresivo mayor,
en remisin parcial.
Nota: Antes de la aparicin del episodio distmico pudo haber un episodio depresivo mayor
previo que ha remitido totalmente (ningn signo o sntoma significativos durante 2 meses).
Adems, tras los primeros 2 aos (1 ao en nios y adolescentes) de trastorno distmico,
puede haber episodios de trastorno depresivo mayor superpuestos, en cuyo caso cabe
realizar ambos diagnsticos si se cumplen los criterios para un episodio depresivo mayor.
E. Nunca ha habido un episodio manaco, un episodio mixto o un episodio hipomanaco y
nunca se han cumplido los criterios para el trastorno ciclotmco.
F. La alteracin no aparece exclusivamente en el transcurso de un trastorno psictico
crnico, como son la esquizofrenia o el trastorno delirante.
G. Los sntomas no son debidos a los efectos fisiolgicos directos de una sustancia (p. ej.,
una droga, un medicamento) o a enfermedad mdica (p. ej., hipotiroidismo).
H. Los sntomas causan un malestar clnicamente significativo o deterioro social, laboral o
de otras reas importantes de la actividad del individuo.
Como vemos, la Distimia presenta sntomas ms o menos similares a la Depresin mayor,
pero se diferencian entre s respecto a:
Tipo de evolucin: el estado de nimo depresivo es crnico (no presenta intervalos libres
de sntomas o mejoras significativas) y dura por lo menos 2 aos.
Severidad de los sntomas: los mismos suelen ser leves o moderados, sin una alteracin
significativa de las relaciones familiares, sociales y laborales del individuo.

Referencias Bibliogrficas
- Diagnostic and Statistical Manual of Mental Disorders, 4 edition (DSM-IV TR).
American
Psychiatric
Association.
American
Psychiatric
Press,
2000.
- Kaplan and Sadock's Synopsis of Psychiatry, 9 edition. Lippincott Williams & Wilkins
Press, 2003.

30.- Recurren a consulta los padres de un menor, que cuenta con cinco aos de edad, lleva
varias noches despertndose agitado como si hubiera soado algo que le angustia. Cuando
acuden a su lado por la noche, el nio les mira y dice palabras que no tienen ningn
significado. Al cabo de un rato vuelve a dormirse y por la maana no recuerda nada de lo
ocurrido. El diagnstico sera:
a) Terrores nocturnos.
b) Pesadillas.
c) Disomnia.
d) Sonambulismo.
Criterios para el diagnstico de F51.5 Pesadillas (307.47)
A. Despertares repetidos durante el perodo de sueo mayor o en las siestas diurnas,
provocados por sueos extremadamente terrorficos y prolongados que dejan recuerdos
vividos, y cuyo contenido suele centrarse en amenazas para la propia supervivencia,
seguridad o autoestima. Los despertares suelen ocurrir durante la segunda mitad del
perodo de sueo.
B. Al despertarse del sueo terrorfico, la persona recupera rpidamente el estado
orientado y despierto (a diferencia de la confusin y desorientacin que caracterizan los
terrores nocturnos y algunas formas de epilepsia).
C. Las pesadillas, o la alteracin del sueo determinada por los continuos despertares,
provocan malestar clnicamente significativo o deterioro social, laboral o de otras reas
importantes de la actividad del individuo.
D. Las pesadillas no aparecen exclusivamente en el transcurso de otro trastorno mental (p.
ej., delirium, trastorno por estrs postraumtico) y no se deben a los efectos fisiolgicos
directos de una sustancia (p. ej., drogas, frmacos) o de una enfermedad mdica.
Criterios para el diagnstico de F51.4 Terrores nocturnos (307.46)
A. Episodios recurrentes de despertares bruscos, que se producen generalmente durante
el primer tercio del episodio de sueo mayor y que se inician con un grito de angustia.
B. Aparicin durante el episodio de miedo y signos de activacin vegetativa de carcter
intenso, por ejemplo, taquicardia, taquipnea y sudoracin.
C. El individuo muestra una falta relativa de respuesta a los esfuerzos de los dems por
tranquilizarle.
D. Existe amnesia del episodio: el individuo no puede describir recuerdo alguno detallado
de lo acontecido durante la noche.
E. Estos episodios provocan malestar clnicamente significativo o deterioro social, laboral,
o de otras reas importantes de la actividad del individuo.
F. La alteracin no se debe a los efectos fisiolgicos directos de una sustancia (p. ej.,
drogas, frmacos) o de una enfermedad mdica.

31.- Masculino de 43 aos que ingresa al servicio de urgencias por haber sufrido colisin
automovilstica de frente. Sufre fractura en tibia y peron, se practic ciruga ortopdica
sin complicaciones y, durante los dos primeros das del postoperatorio, el paciente estuvo
inquieto pero bien orientado. Al tercer da, el paciente se mostr de repente confuso y
temeroso y refiri visin de serpientes y araas que se desplazaban en su cama. En la exploracin se observ midriasis, temblor de oscilaciones amplias de las manos y los prpados,
sudoracin profusa, taquicardia con latido hipercintico y temperatura de 39C. Como
antecedente se consideraba un bebedor social moderado y negaba el abuso de drogas. El
diagnstico ms probable es:

a)
b)
c)
d)

Delirio por deprivacin alcohlica.


Delirio postanestesia.
Septicemia.
Hematoma subdural.

CRITERIOS DIAGNSTICOS DE LA ASOCIACIN AMERICANA DE PSIQUIATRA


PARA LA DEPRIVACIN ALCOHLICA Y EL DELIRIO POR DEPRIVACIN
ALCOHLICA (DSM-IV)
DEPRIVACIN ALCOHLICA
A.-Cese o disminucin del uso prolongado e intenso de alcohol
B.-Dos o ms de los siguientes criterios, entre varias horas y
varios das despus de A:
1-Hiperactividad autonmica (diaforesis, taquicardia)
2-Aumento de temblor en manos
3-Insomnio
4-Nuseas y vmitos
5-Ilusiones o alucinaciones transitorias visuales,
tctiles o auditivas
6-Agitacin psicomotriz
7-Ansiedad
Crisis epilpticas tipo gran mal
C.-Agotamiento y angustia significativas o deterioro social y
ocupacional en relacin con B
D.-Exclusin de otras condiciones mdicas generales y de otros
trastornos mentales
DELIRIO POR DEPRIVACIN ALCOHLICA
A.-Alteracin del nivel de consciencia con reduccin de
habilidad para fijar, mantener o cambiar la atencin
B.-Cambio en la cognicin o desarrollo de trastorno perceptual,
no explicado por demencia preexistente
C.-Se desarrolla en horas o das y tiende a fluctuar durante el
da y tras un sndrome de deprivacin

32.- Masculino de 50 aos que presenta sbitamente dolor intenso en primer ortejo de pie
derecho durante la noche posterior, tras ingesta de alcohol. Con los datos clnicos
anteriores usted pensara que el paciente cuenta con:

a)
b)
c)
d)

Hiperuricemia crnica sintomtica.


Sndrome de reiter.
Monoartritis infecciosa.
Artritis gotosa.

Las causas que generan hiperuricemia son mltiples, pero en general podemos dividirlo en
tres grupos:1 Hiperuricemia idioptica (10-15%) que representa los errores innatos del
metabolismo, padecimientos caracterizados por un incremento en la sntesis de purinas y
sobreproduccin de cido rico; 2 Por incremento del recambio metablico y que genera;
un exceso de cido rico srico, observado en los sndromes mieloproliferativos, neoplasias,
etctera, y 3 grupo conformado por padecimientos o condiciones que producen baja
excrecin renal de cido rico. La acidosis metablica, insuficiencia renal, enfermedades
metablicas endocrinolgicas y el uso de frmacos, son las causas ms comunes de esta
situacin.
Factores de Riesgo: Todas las causas de hiperuricemia son un factor de riesgo para la
gota. 1) La obesidad o el aumento o prdida repentinos de peso; 2) mayores de 40 aos; 3)
sexo masculino; 4) Miembros de la familia con gota;5) Diurticos, tales como el
hidroclorotiazido; 6) Algunas medicinas, como la aspirina; 7) Una diera rica en cristales de
nitrgeno; 8) Consumo de alcohol; 9) Algunos tipos de cncer o tratamientos contra el
cncer (por ejemplo, medicinas citotxicas); 10) Medicinas (tales como los que combaten la
apopleja y otros); 11) Deshidratacin;12) Hipercolesterolemia;13) Enfermedad renal; 12)
Desordenes endocrinos, como el hipotiroidismo y el hiperparatiroidismo

Como es una enfermedad inflamatoria tiene un comportamiento episdico, de distribucin


universal, predomina en el hombre con el 90% de los casos y 10% en mujeres, de
preferencia posmenopusicas. De inicio mas frecuente entra la 4 y 6 dcada. Con
diferentes estadios conocidos como hiperuricemia sintomtica, artritis gotosa aguda,
periodo intercrtico y gota crnica tofcea.
Entonces, de acuerdo a los estadios de la gota podemos mencionar:
A Hiperuricemia asintomtica
B Artritis Gotosa: Es la mas comn y se caracteriza por dolor agudo, intenso, que afecta a
una ms articulaciones de predominio monoarticularlas, de acuerdo a orden de frecuencia
son; el dedo gordo del pie tobillo, tarso, rodilla y mueca.
C Periodo Intercrtico: intervalo entre un ataque y otro. El paciente queda libre de
sntomas hasta la presentacin del siguiente ataque de artritis, la duracin del periodo
asintomtico es variable. Posteriormente los periodos asintomticos se van acortando y el
paciente evoluciona hacia la cronicidad.
Gota Tofcea Crnica: Caracterizada por periodos recidivantes de artritis con depsitos
de uratos conocidos como tofos, tumoraciones sobre la articulacin que pueden estar
excretando un material calcreo.

Rev Pacea Med Fam 2006; 3(3): 2-5


Dra. Nilsa Selaya C.
Dr. Cesar Rabaza M.
Dr. Rafael Castillo R.

33.- Un paciente con insuficiencia renal crnica debida a hipotensin prolongada grave es
atendido porque presenta dolor retroesternal. Se le indic hemodilisis dos veces por
semana en los ltimos dos aos y en fechas recientes ha experimentado episodios de
hipotensin al inicio del tratamiento. El dolor se localiza sobre el msculo trapecio. Se
reduce un poco al adoptar la posicin de pie y se exacerba con la respiracin profunda.
Cul de los siguientes trastornos es la causa ms probable del dolor retroesternal que
sufre este paciente?
a)
b)
c)
d)

Arteriopata coronaria
Espasmo esofgico difuso
Pericarditis
Embolias pulmonares

El dolor torcico que sufri este paciente es caracterstico de pericarditis e inflamacin


del pericardio, complicaciones comunes en personas con insuficiencia renal crnica en
hemodilisis. Estos enfermos tambin pueden tener inflamacin de varios recubrimientos
serosos, como peritoneo y pleura; no se conoce el mecanismo de esta complicacin. Aunque
es comn la arteriopata coronaria en pacientes en dilisis, las caractersticas de dolor en
este individuo sugieren que no es el diagnstico. La enfermedad esofgica tambin es
comn en sujetos en dilisis y debe descartarse especficamente como posible causa. La
relacin aparente con la dilisis, as como la frecuencia de los sntomas, van contra el
diagnstico de embolia pulmonar como causa del dolor torcico. Adems, el dolor
musculoesqueltico debido a diversos trastornos se observa en pacientes en dilisis y
puede deberse a anomalas en el metabolismo del calcio y el fsforo, que produce depsitos
de calcio en diversos componentes del sistema musculoesqueltico.

1. Alexander JS. A pericardial effusion of gold paint appearance due to the presence of
cholesterin. Br Med J 1919; 2: 463.
2. Brawley RK, Vasko JS, Morrrow AG. Cholesterol pericarditis: consideration of its
pathogenesis and treatment. Am J Med 1966; 41: 235-248.

34.- Se reportaron 45 casos de varicela entre los alumnos de primer grado de una escuela
secundaria durante la segunda quincena del mes de agosto de 2009. La tasa de ataque es
de 26.4%. Esta situacin nos indica que estamos ante un(a):
a)
b)
c)
d)

Epidemia
Endemia
Brote
Pandemia

De acuerdo a la NOM- 017 un brote se define como la ocurrencia de dos o ms casos


asociados epidemiolgicamente (tiempo, lugar y persona) entre s. La medida cuantitativa
de la extensin de un brote es la Tasa de Ataque (TA) que se calcula dividiendo el nmero
de casos nuevos entre el total de personas expuestas por 100.

Norma Oficial Mexicana NOM-017-SSA2 -1994, Para la vigilancia epidemiolgica.


Apartados 3.1.6 y 3.1.78.1.

35.- Femenino de 78 aos acude a consulta externa por referir debilidad general y apata,
aadindose en la ltima semana disnea progresiva hasta ser de pequeos esfuerzos.
Antecedentes: cardiopata hipertensiva con funcin sistlica conservada, en los ltimos 2
aos ha tenido 3 episodios de fibrilacin paroxstica cardiovertidos elctricamente.
Durante este tiempo ha recibido diversos tratamientos que incluan algunos de los
siguientes frmacos: propafenona, amiodarona, digoxina, diltiacem y captopril. El ECG
muestra
fibrilacin auricular con frecuencia ventricular a 130 lpm, RX de trax
cardiomegalia signos de congestin pulmonar y el estudio de funcin tiroidea una T4 libre
elevada con una TSH indetectable.
El medicamento responsable de la sintomatologa de esta paciente es:
a) Propafenona.
b) Amiodarona.
c) Digoxina.
d) Diltiacem.

Alteraciones de lab por amiodarona:


Hepticos (1,2%) Elevacin del ASAT y ALAT 2-3 veces valor normal
Elevaciones persistentes de LDH y Fosfatasa alcalina
Gstricos (4,2%) Naseas. Vmitos. Estreimiento
Pulmonares (1,9%) * Sntomas respiratorios y nuevas alteraciones en Rx de trax
> 30% de reduccin en difusin pulmonar.
(PFR)
Tiroides (3,7%) Hipertiroidismo o hipotiroidismo clnico
Cambios en la funcin tiroidea (TSH, T4, T3) que requiere medicacin
Neurolgicos (4,6%) Temblor. Ataxia. Disestesias Insomnio
Piel (2,3%)
Fotosensibilidad; coloracin azul/griscea

(piel)

Ojos (1,5%) Alteraciones visuales. Halos visuales


Visin nocturna borrosa. Depsitos corneales en la lmpara de hendidura.
Bradicardia/Conduccin (3,3%)
Bradicardia sintomtica. Bloqueo A-V de 3er grado
Bradicardia < 50 lpm asintomtica. Bloqueo
A-V de 1er y 2 grado
Supresin del frmaco (23%) Por efectos adversos no tolerados, decisin del paciente o
no cumplimentacin de prescripcin farmacolgica
(*) Se relaciona con la dosis (ms frecuente en dosis >400 mg/da) y duracin del
tratamiento.
Referencias:
Volumen 13, Nmero 4
Octubre - Diciembre 2002
pp 149 152
Cardiologa
Trabajo de investigacin
* Cardilogo, Ex-Residente Hospital de Especialidades, Centro
Mdico Nacional La Raza, IMSS. Hospital de la Fe, San Miguel
Allende, Gto.
** Jefe del Servicio de Cardiologa, Hospital de Especialidades
Centro Mdico Nacional La Raza, IMSS.

36.- En el caso de encontrar una diarrea con presencia de moco, sangre y protenas
procedentes de una mucosa intestinal inflamada constituye una diarrea:
a) Osmtica
b) Exudativa
c) Secretora
d) Motora

Diarrea exudativa. Es producto de la inflamacin, ulceracin de la mucosa intestinal y


alteracin de la permeabilidad para agua, electrolitos y solutos pequeos como la rea. . Se
trata de deposiciones que contienen moco, sangre, protenas y pus. Puede tener algunos
componentes de la diarrea secretora como consecuencia de la liberacin de prostaglandinas
por clulas inflamatorias. Es consecuencia de infecciones bacterianas (Salmonella),
clostridium difficile (frecuentemente inducidos por antibiticos) parsitos del colon
(Entamoeba histolytica), enfermedad de Crohn, enterocolitis por radiacin e isquemia
intestinal, proctocolitis ulcerativa y enfermedad intestinal inflamatoria idioptica.

LECTURAS RECOMENDADAS:
1.
2.
3.
4.
5.
6.

Avery ME. Snyder JD. Oral therapy for acute diarrhea. New Engl J Med 323:891,
1990.
Campos J. Tercer Curso Anual de Actualizaciones en Medicina Interna.
Fundacin Santa Fe de Bogot. Santaf de Bogot, 1990.
Diaz H, Campos J. Diarrea aguda. En: Medicina Interna. F Chalem, J Escandn, J
Campos,
R Esguerra, editores. Fundacin Instituto de Reumatologa e Inmunologa. Editorial
Presencia Ltda. Santaf de Bogota, 1992.
Donowitz M et al. Drug therapy for diarrheal diseases. A look ahead. Rev Infect
Dis 8:202, 1986.

37. - Which of the following drugs combination would be most appropriate in the patient
treatment of acute pelvic inflammatory disease?

a)
b)
c)
d)

Ampicillin / cefoxitin
Tetracycline / gentamicin
Cefoxitin / Doxycycline
Ampicillin / Amikacin

E.P.I.
Regmenes de tratamiento CDC 2002

Parenteral.
Cefotetan 2gr iv/12h Cefoxitina 2gr iv/6h + Doxiciclina 100 mgs iv/8h.
Clindamicina 900 mg iv/8h + Gentamicina iv/im (2mg/kg de carga, luego 1.5
mg/kg/8h.

Alternativas.
Ofloxacina 400 mg iv/12h Levofloxacino 500 mg iv diario con o sin
Metronidazol 500 mg iv/12 h.

Oral.

Ofloxacina 400 mg vo diario x 14 das Levofloxacina 500 mg vo diario x 14


das con o sin Metronidazol 500 mg vo diario por 14 das.
Ceftriaxona 250 mg im x 1 dosis Cefoxitina 2 gr im x 1 dosis y Probenecid
1 gr vo x 1 dosis u otra cefalosporina de 3 gen im + Doxiciclina 100 mgs
vo diario or 14 das con o sin Metronidazol 500 mgs vo diario x 14 das.

38.- Se trata de paciente de 32 aos con antecedente de cuadros catarrales y tabaquismo


crnico, inicia con molestias farngeas que progresan rpidamente a dolor intenso de
garganta e impide la deglucin acompaado de fiebre. Al acudir al servicio y ser asistido,
se niega a acostarse en camilla, permaneciendo sentado e inclinado hacia delante. El diagnstico ms probable ser:

a) Laringitis catarral aguda.


b) Angina de Ludwick.
c) Edema de Reinke.
d) Amigdalitis bacteriana.

Edema de Reinke

Son lesiones en las que se produce un acumulo de un liquido fluido, gelatinoso, bajo la
cubierta mucosa de las cuerdas vocales. Son generados por fenmenos de inflamacin
crnica debidas al abuso vocal y al consumo importante de tabaco que encontramos en ms
del 90% de los pacientes. Esta descrito que puede aparecer en casos de hipotiroidismo y
asociado al reflujo gastroesofgico.

El edema suele afectar a ambas cuerdas vocales y es de aparicin ms frecuente en los


varones. Se presenta como una disfona progresiva con voz ronca y con un tono ms bajo,
voces ms graves y con menos potencia vocal y tendencia a la fatiga en los casos ms
intensos.
El edema de Reinke puede mejorar claramente evitando el consumo de tabaco, el carraspeo
y modificando los malos hbitos vocales. Una correcta rehabilitacin vocal ser el
complemento adecuado en la gran mayora de los casos.
Se considerar el tratamiento con microciruga laringea en los casos crnicos, refractarios
al tratamiento conservador y en las que la calidad de voz es mala incapacitando al paciente
para su vida de relacin o de trabajo. En algunos profesionales de la voz encontramos
edemas de Reinke con voces ms graves, clidas que conforman una firma vocal
caracterstica que les identifica y que no les interesa cambiar por lo que no precisaran de
un tratamiento agresivo.

Clnica
Disfona de larga
evolucin con voz
grave sobre todo
matutina
Disnea ocasional
Relacin con abuso
del tabaco
Predominio en
varones de 50-60
aos, pero aumenta
la incidencia en
mujeres
Relacin con
reflujo gastroesofgico

Exploracin

Actitud y consejos

Laringoscopia
indirecta:

Medidas de higiene
vocal

- Irritacin y edema
organizado en
forma de bolsas
en los 2/3
anteriores de
ambos pliegues

Protectores
gstricos

- Defecto de cierre
posterior
- Aspecto
gelatinoso o rojizo

Microciruga
larngea con incisin
y aspirado del edema
en 1 2
intervenciones

Evolucin
Mejora
Recidivas si
persisten hbitos
tabquicos
Riesgo de cicatrices
vocales
No degeneran
aunque se asocian a
laringitis crnicas

Supresin absoluta
del tabaco
Rehabilitacin
logopdica
ocasionalmente

Bruch JM. Hoarseness in adults [Internet]. Waltham, MA: UpToDate, Rose, BD (Ed),
version 17.1 ; 2009 [acceso 6/4/2009]. Disponible en:
Chadha NK, James AL. Tratamiento antiviral adyuvante para la papilomatosis respiratoria
recurrente (Revisin Cochrane traducida). En: La Biblioteca Cochrane Plus, nmero 3,
2008. Oxford, Update Software Ltd. Disponible en: Colton R, Casper J, Leonard R.
Understanding Voice Problems. A Physiological Perspective for Diagnosis and
Treatment. 3rd ed. Baltimore Philadelphia: Lippincott Williams & Wilkins; 2006.
Gallagher TQ, Derkay CS. Recurrent respiratory papillomatosis: update 2008. Curr Opin
Otolaryngol Head Neck Surg. 2008;16(6):536-42.
Garca-Tapia R, Cobeta I. Clasificacin de las disfonas. En: Diagnstico y tratamiento de
los trastornos de la voz. Eds: Garca-Tapia R y Cobeta I. Editorial Garsi, S.A. 1996.
Madrid.
Ishizuka T, Hisada T, Aoki H, Yanagitani N, Kaira K, Utsugi M et al. Gender and age risks for
hoarseness and dysphonia with use of a dry powder fluticasone propionate inhaler in
asthma. Allergy Asthma Proc. 2007 Sep-Oct;28(5):550-6.

39.- Un nio de 5 aos no inmunizado acude a consulta con una historia de 2 semanas de
evolucin de tos paroxstica, fiebre de bajo grado, emesis post-tos y descarga nasal
viscosa. El EF revela otitis media bilateral y conjuntivitis hemorrgica. Se auscultan
estertores inspiratorios bilateralmente. BH con LT 45 000, con 95% de linfocitos. Cul de
los siguientes es el diagnstico ms probable?

a)
b)
c)
d)

Neumona por Chlamydia


Tosferina
Bronquiolitis
Neumonitis por VSR

La tos ferina es una enfermedad infecciosa altamente contagiosa causad por un bacilo gram
(-) llamado Bordetella pertussis.
El comienzo suele ser insidioso con una fase catarral, con tos irritante que poco a poco se
vuelve paroxstica, por lo regular en el trmino de una a dos semanas, y que dura de uno a
dos meses o ms. Los paroxismos se caracterizan por accesos repetidos y violentos de tos;
cada serie de ellos comprende innumerables toses sin inspiracin intermedia y puede ser
seguida por un estridor respiratorio de tono alto caracterstico. Los paroxismos con
frecuencia culminan con la expulsin de mucosidades claras y adherentes, a menudo seguida
de vmito. Los lactantes menores de 6 meses de edad, los adolescentes y los adultos
frecuentemente no tienen el cuadro tpico de estridores o tos paroxstica.
Resumen:
Etiologa: Bordetella pertussis.
Otros: Bordetella parapertussis, Bordetella bronchiseptica, Mycoplasma pneumoniae,
Chlamydia trachomatis, Chlamydia pnuemoniae y adenovirus.
Fuentes de contagio adolescentes y adultos.
Infeccin intradocimiciliaria en el 80 % de los no vacunados.
Contagio: Fase catarral y hasta 2 semanas de iniciada la tos.
Periodo de incubacin de 10 a 21 das.
Cultivo nasofarngeo Bordet-Gengou (Dacrn o alginato de calcio).
Negativo en fase temprana, > 4 semanas sin estaba vacunada y > 5 das si recibi
tratamiento.

Leucocitosis con linfocitosis absoluta.


Tratamiento:
Apoyo: Apnea, hipoxia y otras complicaciones.
Antibitico en fase catarral es efectivo.
Frenar la propagacin del germen.
Eritromicina 40-50 mgkgd cada 6 hrs. x 14 das.
Claritromicina 15-20 mgkgd cada 12hrs. x 7 das.
Azitromicina 10-12 mgkgd cada 24 hrs. x 5 das.
TMP SMZ 8 mgkgd cada 12 hrs. x 14 das.

Bibliografa:

Abul K. Abbas, Andrew H. Lichtman, Jordan S. Pober. Inmunologa Celular y


Molecular.
McGrall-Hill Interamericana. Cuarta edicin 2001.

Napolen Gonzlez Saldaa y Mercedes Macas Parra. Vacunas en Pediatra.


McGrall-Hill
Interamericana. Primera edicin 1999.
Report of the Committee on Infectious Diseases. American Academy of Pediatrics.
Red
Book 2000. 25 edicin 2000.
Stanley A. Plotkin, Walter A. Orenstein. Vaccines. W. S. Saunders Company.
Tercera edicin 1999.

40.- Se trata de paciente femenino de 52 aos de edad a la cual se le diagnostica


neumona por aspiracin, aislando en esputo bacilo Gram negativo anaerobio, en su historia
clnica niega alergia medicamentosa Cul es el tratamiento antimicrobiano de eleccin?

a) Metronidazol.
b) Eritromicina.
c) Ceftriaxona.
d) Ciprofloxacino

NEUMONIA POR ASPIRACION


La aspiracin de contenido gstrico ocurre con relativa frecuencia en pacientes
severamente enfermos. Los factores predisponentes incluyen alteraciones mentales,
anestesia general, traqueostoma e intubacin nasogstrica. Las consecuencias de la
aspiracin dependen de la naturaleza del contenido gstrico:
a. Entre mayor sea el volumen aspirado, mayor ser el dao causado
b. La acidez gstrica causa inflamacin con exudacin, disminucin de surfactante y
mayor riesgo de atelectasis
c. Las partculas alimenticias inician una reaccin granulomatosa
d. Las bacterias de las secreciones orofaringeas complican la respuesta inflamatoria
La aspiracin es causa de fiebre y de la aparicin de infiltrados pulmonares. La presencia
de broncoespasmo e hipoxemia sugieren el diagnstico que evoluciona hacia un cuadro de
neumonitis y una infeccin bacteriana. La consolidacin aparece rpidamente acompaada

de tos productiva y esputo purulento. Los cultivos de esputo muestran en el 63% de los
casos flora aerbica, en el 7% anaerobios y en el 30% flora mixta.

ANAEROBIOS
Los anaerobios son la flora predominante en las secreciones orafaringeas. Por lo tanto, la
va de infeccin por estos grmenes es la aspiracin. Existen cuatro patrones de infeccin:
1) neumonitis aguda; 2) neumona necrotizante; 3) absceso pulmonar y 4) empiema. Todos
estn relacionados entre s puesto que una neumona aguda si no se trata, evoluciona a un
proceso necrotizante que se puede localizar, dando origen a un absceso que al romperse
hacia el espacio pleural produce un empiema.
Hallazgos clnicos. Tos productiva con esputos de mal olor, dolor pleurtico; si existe un
absceso la respiracin es anfrica o cavernosa.
Hallazgos de laboratorio. Una muestra obtenida por puncin transtraqueal permite su
cultivo en un ambiente para anaerobiosis. Para obtener un resultado ptimo la muestra se
debe llevar al laboratorio en jeringas sin aire. Las infecciones pulmonares anaerbicas que
aparecen en pacientes hospitalizados generalmente son de tipo mixto y con frecuencia se
encuentran Staphylococcus o bacilos gram-negativos. Los microorganismos ms importantes
son el Bacteroides melaninogenicus y el Fusobacterium nucleatum y los estroptococos
anaerbicos propios de la flora existente en la orofaringe. En el caso de enfermedades
abdominales como la obstruccin abdominal o ciruga, una causa frecuente de infeccin es
el B. fragilis.
Tratamiento. La mayora de los microorganismos, con excepcin del B. fragilis son
sensibles a la penicilina G. y la carbenicilina. Casi todos, incluyendo el B. fragilis son
sensibles al cloranfenicol y a la clindamicina. El metronidazol es eficaz contra la mayora de
los anaerbicos incluyendo del B. fragilis. En la neumona aguda se recomienda la penicilina
cristalina 2 millones de unidades IV cada 4 horas; en la neumona necrotizante y el absceso
pulmonar se prescribe penicilina cristalina ms metronidazol. Es aconsejable practicar una
fibrobroncoscopia ante la presencia de un absceso para descartar un cuerpo extrao o una
lesin maligna. En el empiema anaerbico, la maniobra terapetica ms importante es el
drenaje adecuado por medio de un tubo torcico. Se utilizan dosis altas de penicilina
cristalina (3 millones de unidades cada 4 horas), ms clindamicina 600 mg IV cada 6 horas o
metronidazol, 15 mg/kg/6 horas, IV.

Referencias:
1.
2.
3.

4.
5.

6.

Des Jardins T. Neumona. En: Enfermedades Respiratorias. Editorial El Manual


Moderno. Mxico DF, 1993.
Glackman RA. Oral empirical treatment of pneumonia. The Challenge of Choosing
the Best Agent. Postgrad Med 95:165, 1994.
Jauregui A. Infecciones de vas respiratorias superiores y neumonas. En: Manual
de Teraputica Mdica. Instituto Nacional de la Nutricin. Interamericana. Mc
Graw Hill. Mjico DF, 1994.
La Force-FM. Antibacterial therapy for lower respiratory tract infections in adults:
a review. Clin Infect Dis. 14 Suppl 2:S233; discussion S244, 1992.
Londoo F. Neumonas. En: Neumologa. Editado por Jorge Restrepo M, Daro
Maldonado G. Fundamentos de Medicina, Corporacin para Investigaciones
Biolgicas. Medelln, 1986.
Tobin M. Diagnosis of pneumonia, techniques and problems. Clin Chest Med 8:513,
1997.

41.-Masculino de 40 aos que de manera inesperada y espontnea inicia cuadro


constituido por disnea, sensacin de ahogo o de paro respiratorio, sensacin de
inestabilidad, siente que se desmaya, palpitaciones o taquicardia, mareo, nauseas, dolor
abdominal. Cul de los siguientes el diagnstico mas probable?

a) Trastornos de ansiedad generalizada.


b) Trastorno fbico.
c) Trastorno de pnico.
d) Trastorno mixto ansioso-deperesivo.

La caracterstica fundamental del Trastorno de Pnico es la presencia de Crisis de Pnico


recurrentes, inesperadas y que no se encuentran relacionadas con ninguna circunstancia en
particular (es decir son espontneas), sin un factor externo que las desencadene, es decir
que no son desencadenadas por una exposicin a una situacin social (como es el caso de la
Fobia Social) o frente a un objeto temido (en cuyo caso se tratara de una Fobia
Especfica ). Otra caracterstica del Trastorno de Pnico es el miedo persistente a padecer
una nueva Crisis de Pnico, esto es miedo al miedo y se lo denomina Ansiedad Anticipatoria.
Esta ansiedad puede a llegar a ser tan importante que puede llevar a confundir el
diagnstico de Trastorno de Pnico con el de Trastorno de Ansiedad Generalizada (TAG).

Estas Crisis (o Ataques) de Pnico, inesperadas y recidivantes, suelen presentar cuatro o


ms de los siguientes sntomas:

Miedo intenso a morir o a estar sufriendo un ataque cardaco o alguna enfermedad


fsica grave que ponga en riesgo la vida
Miedo intenso a volverse loco o a perder el control de si mismo
Palpitaciones (percepcin del latido cardaco) o pulsaciones aceleradas (taquicardia)
Sudoracin
Palidez
Temblores o sacudidas musculares
Sensacin de ahogo o falta de aire
Opresin en la garganta (sensacin de no poder respirar) o en el pecho
Nuseas, vmitos o molestias y dolores abdominales
Inestabilidad, mareos o desmayos
Sensacin de irrealidad (sentir al mundo externo como algo extrao)
Sensacin de no ser uno mismo (despersonalizacin)
Hormigueos (parestesias)
Escalofros o sensacin de sufrir fro intenso

Las crisis se inician bruscamente, alcanzan su mxima intensidad en los primeros diez a 15
minutos y suelen durar menos de una hora. Dejan a quien las sufre en un estado de total
agotamiento psicofsico y con un gran temor (ansiedad anticipatoria) a volver a padecer una
nueva crisis (miedo al miedo). Tambin pueden aparecer sntomas de tipo depresivo, miedo a
salir o alejarse del hogar o necesidad de hacerlo acompaado por una ser muy cercano
(familiar). La persona suele sentirse muy hipersensible y vulnerable. Este tipo de trastorno
es tan traumtico de experimentar que quien lo padece suele cambiar en forma brusca y
desfavorable sus hbitos de vida: no querer salir solo de la casa o viajar, retraimiento
social, abandono de sus actividades laborales o acadmicas.
Muchas veces la persona que se encuentra padeciendo una Crisis de Pnico tiene la
necesidad de "salir corriendo" del lugar donde se encuentra o de consultar urgentemente a
un mdico en el caso que crea que se esta muriendo de verdad. Si esta crisis se
experimenta por primera vez en un lugar determinado, un tren o autobs por ejemplo, suele
quedar temor de volver a ese mismo sitio o medio de locomocin, desarrollndose de este
modo una fobia al mismo.
Un paciente que sufra de Crisis de Pnico describi su primera crisis de la siguiente forma:

"Sbitamente sent una oleada de miedo sin que hubiera razn alguna. El corazn me lata
apresuradamente, me faltaba el aire, senta que no poda respirar, el corazn lata tan
fuerte que pareca que iba a salirse por mi boca. Me dola el pecho, tena mareos, ganas de
vomitar, no poda parar de temblar. Senta que me mora. Era como una agona mortal que
nunca terminaba"

Este trastorno se lo clasifica dentro de las neurosis, no tiene nada que ver con la locura
(psicosis), ni desencadena con el tiempo en un cuadro de locura. Es importante recalcar
esto ya que muchos pacientes que sufren este trastorno, o los familiares del mismo,
piensan que se estn volviendo locos. Lo que s es frecuente es que las personas que
padecen de Trastorno de Pnico desarrollen, si no son tratados a tiempo y adecuadamente,
cuadros de tipo depresivos, fobias mltiples (especialmente agorafobia) o abuso de
sustancias (alcohol o drogas).

Referencias Bibliogrficas:
- Diagnostic and Statistical Manual of Mental Disorders, 4 edition (DSM-IV TR).
American
Psychiatric
Association.
American
Psychiatric
Press,
2000.
- Kaplan and Sadock's Synopsis of Psychiatry, 9 edition. Lippincott Williams & Wilkins
Press, 2003.

42.- Acude a consulta una mujer de 35 aos acompaada de su marido. Ella dice encontrarse muy bien, mejor que nunca, est pudiendo hacer ms cosas e incluso se siente
capaz de comprender cuestiones muy complicadas. El marido dice que ella lleva una semana
durmiendo poco y que no para; se levanta temprano, sale a la calle, vuelve, se cambia, vuelve
a salir, habla sin parar con cualquier persona que se encuentra y no controla lo que gasta.
Nunca le haba ocurrido nada parecido y ella no acepta tener ningn problema. El
diagnstico sera:

a)
b)
c)
d)

Episodio manaco.
Trastorno bipolar.
Trastorno psictico breve.
Trastorno de ansiedad generalizada.

Criterios para el episodio manaco


A. Un perodo diferenciado de un estado de nimo anormal y persistentemente elevado,
expansivo o irritable, que dura al menos 1 semana (o cualquier duracin si es necesaria la
hospitalizacin).
B. Durante el perodo de alteracin del estado de nimo han persistido tres (o ms) de los
siguientes sntomas (cuatro si el estado de nimo es slo irritable) y ha habido en un grado
significativo:
1. autoestima exagerada o grandiosidad.

2. Disminucin de la necesidad de dormir (p. ej., se siente descansado tras slo 3 horas de
sueo)

3. Ms hablador de lo habitual o verborreico.

4. Fuga de ideas o experiencia subjetiva de que el pensamiento est acelerado.

5. distraibilidad (p. ej., la atencin se desva demasiado fcilmente hacia estmulos


externos banales o irrelevantes)

6. Aumento de la actividad intencionada (ya sea socialmente, en el trabajo o los estudios, o


sexualmente) o agitacin psicomotora.

7. Implicacin excesiva en actividades placenteras que tienen un alto potencial para


producir consecuencias graves (p. ej., enzarzarse en compras irrefrenables, indiscreciones
sexuales o inversiones econmicas alocadas)
C. Los sntomas no cumplen los criterios para el episodio mixto.
D. La alteracin del estado de nimo es suficientemente grave como para provocar
deterioro laboral o de las actividades sociales habituales o de las relaciones con los dems,
o para necesitar hospitalizacin con el fin de prevenir los daos a uno mismo o a los dems,
o hay sntomas psicticos.
E. Los sntomas no son debidos a los efectos fisiolgicos directos de una sustancia (p.ej.
una droga, un medicamento u otro tratamiento) ni a una enfermedadmedica (p. ej.,
hipertiroidismo).
Nota: Los episodios parecidos a la mana que estn claramente causados por un tratamiento
somtico antidepresivo (p. ej., un medicamento, teraputica electroconvulsiva, teraputica
lumnica) no deben ser diagnosticados como trastorno bipolar I.

43.- Femenino De 17 aos refiere que lleva un tiempo con menos apetito y durmiendo
menos ya que le cuesta poder dormirse. Es una buena estudiante, pero en la ltima
evaluacin le han quedado cuatro asignaturas y le cuesta concentrarse en los estudios. Los
dos ltimos fines de semana no ha salido con sus amigas porque no le apeteca y se siente
irritada con su familia, aunque no entiende porqu. Tiene dolores de cabeza y a veces le
viene la idea de la muerte a la imaginacin aunque piensa que no lo hara por sus
sentimientos religiosos. Nunca le haba sucedido algo parecido. El diagnstico sera:
a) Trastorno depresivo mayor.
b) Trastorno ciclotmico.
c) Anorexia.
d) Episodio depresivo mayor.
Criterios para el diagnstico del episodio depresivo mayor (DSM-IV)

A. Presencia de cinco (o ms) de los siguientes sntomas durante un perodo de 2 semanas,


que representan un cambio respecto a la actividad previa; uno de los sntomas debe ser (1)
estado de nimo depresivo o (2) prdida de inters o de la capacidad para el placer.

Nota: No incluir los sntomas que son claramente debidos a enfermedad mdica o las ideas
delirantes o alucinaciones no congruentes con el estado de nimo.
1. estado de nimo depresivo la mayor parte del da, casi cada da segn lo indica el propio
sujeto (p. ej., se siente triste o vaco) o la observacin realizada por otros (p. ej., llanto).
Nota: En los nios y adolescentes el estado de nimo puede ser irritable
2. disminucin acusada del inters o de la capacidad para el placer en todas o casi todas las
actividades, la mayor parte del da, casi cada da (segn refiere el propio sujeto u observan
los dems)
3. prdida importante de peso sin hacer rgimen o aumento de peso (p. ej., un cambio de
ms del 5 % del peso corporal en 1 mes), o prdida o aumento del apetito casi cada da.
Nota: En nios hay que valorar el fracaso en lograr los aumentos de peso esperables
4. insomnio o hipersomnia casi cada da
5. agitacin o enlentecimiento psicomotores casi cada da (observable por los dems, no
meras sensaciones de inquietud o de estar enlentecido)
6. fatiga o prdida de energa casi cada da
7. sentimientos de inutilidad o de culpa excesivos o inapropiados (que pueden ser
delirantes) casi cada da (no los simples autorreproches o culpabilidad por el hecho de
estar enfermo)
8. disminucin de la capacidad para pensar o concentrarse, o indecisin, casi cada da (ya
sea una atribucin subjetiva o una observacin ajena)
9. pensamientos recurrentes de muerte (no slo temor a la muerte), ideacin suicida
recurrente sin un plan especfico o una tentativa de suicidio o un plan especfico para
suicidarse

B. Los sntomas no cumplen los criterios para un episodio mixto.


C. Los sntomas provocan malestar clnicamente significativo o deterioro social, laboral o de
otras reas importantes de la actividad del individuo.
D. Los sntomas no son debidos a los efectos fisiolgicos directos de una sustancia (p. ej.,
una droga, un medicamento) o una enfermedad mdica (p. ej., hipotiroidismo).
E. Los sntomas no se explican mejor por la presencia de un duelo (p. ej., despus de la
prdida de un ser querido), los sntomas persisten durante ms de 2 meses o se
caracterizan por una acusada incapacidad funcional, preocupaciones mrbidas de inutilidad,
ideacin suicida, sntomas psicticos o enlentecimiento psicomotor.

Bibliografa:

1. DSM-IV. American Psychiatric Association. . Diagnostic and Statistical Manual of Mental


Disorders (4th Ed.). Washington, DC.

44.- Se trata de femenino de 38 aos, que acude a intercosulta al servicio de medicina


interna por presentar obesidad troncular de reciente diagnstico, amenorrea y depresin.
En la exploracin fsica se aprecia facies redondeadas con hirsutismo moderado, TA de
160/100 mmHg y edemas en miembros inferiores. Se realizan examenes de laboratorio los
cuales arrojan los siuentes resultados: tirotropina (TSH) 0,7 mU/ml (N: 0,4-5,0), T4 libre
16.8 pmol/l (N: 9,0-23,0), cortisol libre en orina 11 mg/24h (N: 20-100), cortisol plasmtico
3 mg/dl (N: 5-25) y corticotropina (ACTH) 9 pmol/l (N: inferior a 52). Cul de las
siguientes situaciones estar dando lugar a este cuadro?

a) Sndrome de Cushing ACTH dependiente.


b) Administracin exgena de glucocorticoides.
c) Hipotiroidismo subclnico.
d) Enfermedad de Cushing.

Los corticosteroides (del lat. cortex, cis, corteza, y esteroide) o corticoides son una
variedad de hormonas del grupo de los esteroides (producida por la corteza de las
glndulas suprarrenales) y sus derivados.
Los corticosteroides estn implicados en una variedad de mecanismos fisiolgicos,
incluyendo aquellos que regulan la inflamacin, el sistema inmunitario, el metabolismo de
hidratos de carbono, el catabolismo de protenas, los niveles electrolticos en plasma y, por
ltimo, los que caracterizan la respuesta frente al estrs.
Estas sustancias pueden sintetizarse artificialmente y tienen aplicaciones teraputicas,
utilizndose principalmente debido a sus propiedades antiinflamatorias e inmunosupresoras
y a sus efectos sobre el metabolismo.

Los efectos metablicos del exceso de glucocorticoides son los siguientes:


1. Aumento de la neoglucognesis y resistencia a la insulina; esto puede
llevar a la diabetes mellitus
2. Aumento del catabolismo proteico; esto puede llevar a la emaciacin,
osteoporosis y adelgazamiento de la piel.
3. Aumento y redistribucin de la grasa corporal: se produce una
obesidad de predominio central, facie de luna, tungo o acmulo
dorsal de grasa, manteniendo extremidades relativamente delgadas.
4. Involucin del tejido linftico y disminucin de la respuesta
inflamatoria: se produce una disminucin de la inmunidad celular y
humoral con lo que aumenta la susceptibilidad a infecciones.
5. Aumento de la secrecin de cido por el estmago lo que lleva a una
predisposicin de lcera gastroduodenal.
6. Retencin de sodio y redistribucin de los fluidos corporales lo que
produce edema e hipertensin arterial.
7. Funcin gonadal: los glucocorticoides afectan la secrecin de
gonadotrofinas. En los hombres disminuye la concentracin de
testosterona. En las mujeres, suprime la respuesta de LH al GnRH,
lo que lleva a una supresin de la secrecin de estrgenos y
progestinas, con anovulacin y amenorrea.

Todos los efectos anteriormente enunciados pueden ocurrir independientes del origen de
los glucocorticoides. La causa ms comn de sndrome de Cushing se debe a la
administracin exgena de dosis farmacolgicas de ellos con fines generalmente
antiinflamatorios e inmunosupresores.

45.- En un estudio transversal se tienen 1000 pacientes con cncer mamario, 32 de ellas
estaban embarazadas. A partir de estos datos, se puede concluir que:
a) El embarazo es una complicacin rara del cncer mamario.
b) Si se hacen los ajustes de la edad, se puede determinar el riesgo de cncer de mama
durante el embarazo.
c) En este estudio el 3.2 % de las pacientes con cncer de mama estaban
embarazadas.
d) Existe asociacin causal entre estar embarazada y desarrollar cncer de mama.

Los estudios transversales solo permiten recoger informacin y describir la distribucin de


frecuencias de las caractersticas de salud de la poblacin y de las posibles asociaciones de
stas con otras variables. Solo permite calcular prevalencia (32/1000X100= 3.2)

Ruiz M. A. Epidemiologa Clnica, Panamericana, 1. Ed. 2004; pg: 198

46.- Masculino de 34 aos que acude por fiebre de comienzo sbito, tos con expectoracin
purulenta y dolor en el hemitrax que aumenta al respirar. La placa de trax muestra signos
de consolidacin pulmonar y un infiltrado lobular. Ud sospecha el siguiente agente causal e
indica tratamiento:

a) Chlamydia /azitromicina
b) Coxiella /vacomicina
c) S. pneumoniae / ceftriaxona
d) M. pneumoniae /penicilina

En ste caso lo mas probable es que el paciente se encuentre cursando con una
neumona adquirida en la comunidad originada en el 60-70% de los casos por estreptococo
pneumoniae el tratamiento ya no es con penicilina debido al alto grado de resistencia a
este tratamiento, en la actualidad se utiliza cefalosporinas es decir en el presente caso
seria la respuesta CEFTRIAXONA.
1.- Fishman AP, , Fishman JA, Grippi MA, Kaisser LR, Seor RM. Pulmonary Diseases and
disorder. 3a. Edicin McGraw-Hill, EUA, 2006.
2.- Fraser, R ; Neil, C; Par, P; Diseases of the Chest, Third Edition, Editorial Elsevier,
2005.
3.- Murray and Nadels; Textbook Respiratory Medicine, Vol 1-2, Elsevier editorial, 2005.

47.- Hombre de 73 aos. Llega al Servicio de urgencias por alteracin del estado de alerta,
a su ingreso se realiza glucosa capilar y tiene glucosa de 780 mg/dL. Tiene antecedente de
hipertensin arterial desde hace 12 aos. EF desorientado, deshidratado, TA 90/60, FC
110, peso 68 kg, estatura 1.72 cm, se coloca sonda Foley y se obtienen 40 ml de orina
turbia. Laboratorio: glucosa 810mg/dL, urea 44 mg/dL, creatinina 2 mg/dL, Na 155 mEq/L,
K 3.2 mEq/L.
El diagnstico del paciente es:

a)
b)
c)
d)

Cetoacidosis
Hipoglucemia
Estado hiperosmolar hiperglucmico
Estado hiperosmolar hipernatrmico

El paciente prototpico en estado hiperosmolar hiperglucmico (HHS) es un anciano con DM


de tipo 2 que tiene antecedentes de varias semanas de duracin con poliuria, prdida de
peso y decremento de la ingestin oral que culminan en confusin mental, letargo o coma.
Los datos de la exploracin fsica reflejan deshidratacin profunda e hiperosmolalidad y
revelan hipotensin, taquicardia y trastorno del estado mental. Es notable la ausencia de
sntomas como nuseas, vmitos y dolor abdominal, as como de la respiracin de Kussmaul
caracterstica de la DKA. Con frecuencia el HHS es precipitado por una enfermedad
concurrente grave, como infarto del miocardio o accidente vascular cerebral. Otros
factores precipitantes frecuentes son sepsis, neumona y otras infecciones, y es
indispensable investigar su presencia. Asimismo, pueden contribuir tambin al desarrollo de
este trastorno padecimientos debilitantes (accidente vascular cerebral previo o demencia)
y situaciones sociales que obstaculizan la ingestin de agua.
Fisiopatologa
El dficit relativo de insulina y el aporte insuficiente de lquidos son las causas que
subyacen al HHS. El dficit de insulina aumenta la produccin heptica de glucosa por el
msculo esqueltico (vase lo tratado anteriormente en la DKA). La hiperglucemia induce
una diuresis osmtica que provoca disminucin del volumen intravascular, que se exacerba
todava ms por el aporte insuficiente de lquidos. No se comprende por completo la
ausencia de cetosis en el HHS. Probablemente, el dficit insulnico es slo relativo y menos
grave que en el caso de la DKA. En algunos estudios se han encontrado concentraciones ms
bajas de hormonas contrarreguladoras y de cidos grasos libres en el HHS que en la DKA.
Tambin es posible que el hgado sea menos capaz de sintetizar cuerpos cetnicos, o que el
cociente insulina/glucagon no favorezca la cetognesis.

Bibliografa:

1. Lobesio C: Sndrome hiperglucmico hiperosmolar. Texto de medicina


Intensiva. 5ta Ed. 2000; 95:1050-54.
2. Jimnez Murillo J, Barca C de la, Romero M, Montero F J. Coma hiperosmolar
en Medicina de urgencia. En: Jimnez Murillo J, Montero F J. Gua diagnstica y
protocolo de actuacin.2da. Ed. Madrid: Harcourt, 1999: 403-5.
3. Remuan C, lvarez JL: Coma Hiperosmolar. Rev Cubana Med 2001;
40(3):189-94.
4. Shoemaker LW: Critical Care Medicine. 3era Ed. Text Book of Critical Care.
Philadelphia Saunders. 2000; 71:783-85.
5. Soler Morejn C: Coma hiperosmolar en: Temas Actualizados en: Rev Cubana
Med 1999; 38(3):183-7.
Villanueva V. Complicaciones agudas de la diabetes mellitas. Rev.

48.- Mujer de 27 aos. Acude a consulta por hirsutismo e irregularidades menstruales.


Antecedente familiar de diabetes mellitus en madre y una hermana. Su padecimiento
empez desde la adolescencia con ritmo menstrual de 40 60 x 5 -7 das. EF: peso 87 kg,
estatura 1.54 m, TA 130/80, FC 72x, acantosis nigricans en cuello y axilas, no galactorrea,
permetro abdominal de 112 cm. Vello supralabial, en patilla, en lnea media infraumbilical y
en cara interna de muslos. Laboratorio: glucosa 104 mg/dL, resto de qumica sangunea
normal, Bh normal. Prolactina 16 ng/dL, testosterona: 78 ng/dL (25 a 50 ng/dl). US
El diagnstico probable es:

a)
b)
c)
d)

Hiperprolactinemia por microprolactinoma


Sndrome de ovarios poliqusticos por resistencia a la insulina
Virilizacin por tumor productor de testosterona
Diabetes mellitus secundaria a la obesidad

El sndrome de ovarios poliqusticos (SOPQ) afecta aproximadamente a un 4% de mujeres


en edad reproductiva y se caracteriza por anovulacin crnica e hiperandrogenismo. Es la
causa ms comn de infertilidad en mujeres.
Se caracteriza clnicamente por acn, alopecia, hirsutismo, irregularidades menstruales e
infertilidad.
Los hallazgos de laboratorio ms frecuentes son: aumento de la hormona luteinizante (LH),
aumento de la relacin LH/FSH (hormona folculoestimulante), aumento de andrgenos
(tanto ovricos como adrenales) y de estrgenos circulantes. Otros hallazgos de
laboratorio habituales son una prueba tolerancia oral a la glucosa anormal y alteraciones en
el perfil lipdico.
Todo esto junto con las imgenes ecocardiogrficas caractersticas definen al sndrome.
La teraputica permite dos grandes enfoques que pueden superponerse: la correccin de
las manifestaciones de hiperandrogenismo y el tratamiento de las alteraciones del eje
reproductivo (anovulacin, esterilidad).
Los antiandrgenos estn fundamentalmente indicados para tratar los sntomas virilizantes.

Las alternativas para inducir la ovulacin son numerosas: al citrato de clomifeno y a la


antigua reseccin en cua se agregan las gonadotrofinas humanas, pulsos de GnRH (hormona
liberadora de gonadotrofinas), medidas o frmacos para modificar los niveles de insulina, y
finalmente tcnicas quirrgicas endoscpicas para reducir la masa ovrica.

Revista de Posgrado de la VIa Ctedra de Medicina - N 125 Marzo 2003


Pg. 37-40
SINDROME DE OVARIOS POLIQUISTICOS
Dra. Sandra Beneyto, Dra. Mara Andrea Ferreyra, Dr. Andrs Galfrascoli,
Dr. Andrs Gonzlez, Dra. Susana Sosa

49.- Masculino de 45 aos con hipercalcemia asintomtica. El resto de sus exmenes de


laboratorio muestran elevacin de parathormona, baja en fsforo, elevacin de cloro y BUN
y creatinina normales. El calcio urinario est elevado. Cul es la etiologa ms probable?
a)
b)
c)
d)

Mileoma mltiple
Hiperparatiroidismo primario
Hipervitaminosis
Sarcoidosis

Al grupo de sndromes que tienen como caracterstica comn la secrecin excesiva y no


controlada de hormona para-tohormona (HPT) por una o ms paratiroides funcionantes se
le conoce como hiperparatiroidismo primario (HPP). La causa ms frecuente es el adenoma y
le sigue en frecuencia la hiperplasia.
El cuadro clnico del HPP afecta principalmente al sistema seo, al rin y al sistema
gastrointestinal. El cuadro clnico se caracteriza por debilidad muscular, fatigabilidad fcil,
manifestaciones gastrointestinales y depresin. Cuando hay afeccin renal el paciente
presenta hipercalciuria (25%), litiasis renal recidivante (25%) y/o nefrocalcinosis (20%).
Cuando hay resorcin subperistica, tumores pardos en huesos largos o en el maxilar
inferior y a nivel del crneo, las lesiones osteolticas le dan el aspecto de sal y pimienta.
En los exmenes de laboratorio hay hipercalcemia, hipofosfatemia, fosfatasa alcalina
elevada (fraccin sea), elevacin de HPT (90%), osteocalcina y desoxipiridinolinas; as
como calciuria e hiperfosfaturia. Cuando hay afeccin del sistema gastrointestinal hay
lceras y hemorragias.
Por lo tanto, como podemos observar, se trata de una paciente con HPP clsico con afeccin
del sistema seo y del rin, sin manifestaciones gastrointestinales aparentemente, el cual
al someterse a tratamiento quirrgico y mdico adecuados hubo mejora inmediata que ha
continuado hasta la fecha actual.

Hiperparatiroidismo primario
Ma. Esther Gutirrez Daz Ceballos,1 Hctor A. Rodrguez Martnez,1 Evelyn M. Torres
Acosta,1 Humberto Cruz Ortiz1
1 Unidad de Patologa y Servicio de Endocrinologa del Hospital General de Mxico y de la
Facultad de Medicina, UNAM

50.- Un hombre de 58 aos de edad con una historia de fumar 20 cajetillas/ao, est
siendo evaluado para la colocacin de un bypass coronario. Cul de las siguientes pruebas
preoperatorias es la mejor para valorar funcin respiratoria en este paciente?

a)
b)
c)
d)

FVC
PaCO2
PaO2
FEV1/FVC

La relacin FEV1/ FVC le proporciona la capacidad de las funciones pulmonares del paciente
y la FEV1 le indica si hay una obstruccin de la va area, la FVC Y VR aisladas le indican si
hay una obstruccin nicamente, la PaCO2 Y PaO2 solo le indican si hay retencin de Co2 y
la oxemia del paciente.

1.- Fishman AP, , Fishman JA, Grippi MA, Kaisser LR, Seor RM. Pulmonary Diseases and
disorder. 3a. Edicin McGraw-Hill, EUA, 2006.
2.- Fraser, R ; Neil, C; Par, P; Diseases of the Chest, Third Edition, Editorial Elsevier,
2005.
3.- Murray and Nadels; Textbook Respiratory Medicine, Vol 1-2, Elsevier editorial, 2005.

51.- Un varn de 65 aos de edad con antecedente de bronquiectasia acude al


departamento de urgencias con hemoptisis. Refiere tos acentuada y produccin de esputo
en la ltima semana acompaados de febrculas. A menudo su esputo se tie de estras de
sangre, pero el ltimo da observ que est tosiendo sangre coagulada en cantidades
equivalentes a una cucharada, con un total aproximado de una taza en 24hrs. La exploracin
fsica muestra signos vitales normales, con saturacin de oxgeno de 98% en aire ambiente.
El paciente manifiesta disnea leve y sibilancias exhalatorias difusas. La radiografa de
trax, adems de mostrar bronquiectasias, es normal.
Cul es el tratamiento inmediato ms apropiado para esta hemoptisis?
a)
b)
c)
d)

Broncoscopia
Embolizacin de la arteria bronquial
TAC de trax
Reseccin quirrgica

En este caso se debe realizarle una broncoscopia con la finalidad de localizar el sitio de
sangrado y detener el mismo por compresin con un cateter de fogarty, la TAC no tiene
indicacion como tratamiento.
TABLA 1. Indicaciones de la broncoscopia diagnstica
Sntomas clnicos no explicados
Tos persistente
Disnea / sibilantes no justificados
Hemoptisis
Disfona
Asma unilateral, de comienzo sbito
Sndrome de vena cava
Parlisis diafragmtica
Aclaracin de una sospecha diagnstica
Alteraciones radiolgicas
Investigar el origen de una citologa de esputo positiva
Estadiaje del carcinoma broncognico
Sospecha de fstula traqueoesofgica
Evaluacin de la va area tras un traumatismo torcico
Evaluacin de la va area en quemados
Toma de muestras en una enfermedad intersticial
Estudio microbiolgico de infecciones respiratorias
Evaluacin de la respuesta al tratamiento del cncer de pulmn
BRONCOSCOPIA DIAGNSTICA Y TERAPUTICA
Monografas NEUMOMADRID
Prudencio Daz-Agero lvarez
Javier Flandes Aldeyturriaga
VOLUMEN X / 2007

52.- Una mujer de 52 aos de edad presenta neumona adquirida en la comunidad


complicada con derrame pleural. Se lleva a cabo toracocentesis con los siguientes
resultados:
Aspecto
Viscoso, opaco
Ph
7.11
Protenas
5.8g/100ml
LDH
285UI/l
Glucosa66mg/dl
Leucocitos
3 800/mm3
Eritrocitos
24 000/mm3
PMN
93%
Muchos PMN; no se observan microorganismos.
Se envan muestras para cultivos bacterianos pero todava no se cuenta con los resultados.
Cul caracterstica del lquido pleural es ms indicativa de que la paciente requerir
pleurostoma con sonda?

a)
b)
c)
d)

La presencia de ms de 90% de PMN


Glucosa de menos de 100mg/dl
pH menor de 7.2
LDH con cifras de ms de 2/3 del lmite superior normal para suero

En ste caso lo mas probable es que el paciente se encuentre cursando con empiema el cual
en el inicio del mismo se valora con una gasometra del lquido pleural el cual si indica un ph
menor a 7.2 es necesario colocar sonda endotorcica para drenarlo, en algunas
publicaciones se menciona que la glucosa puede ser de utilidad pero cuando es menor a 40
mg, por lo tanto la nica respuesta es la del pH menor de 7.2.
El pH del LP sirve para el diagnstico etiolgico de los exudados. Hay que medir el pH y la
PaCO2 en sangre para descartar acidosis sangunea. En general el LP con pH bajo tiene
glucosa baja y LDH alta. El pH est bajo, incluso por debajo de 7.20 en derrames pleurales
paraneumnicos complicados y cuando es menor de 7,10 es indicacin de drenaje de la
cavidad pleural. Una excepcin son las infecciones por Proteus mirabilis que producen
aumento de la amoniemia. El pH bajo en las neoplasias est relacionado con el nmero de
clulas y con el pronstico (menor supervivencia y peor respuesta a la pleurodesis).
Tambin est disminuido en la artritis reumatoide donde puede ser menor de 7.20 mientras
que en el LES puede estar normal. En el hemotrax, tambin est bajo debido al consumo
de glucosa por los hemates con la consiguiente produccin de CO2 y disminucin del pH. Los
urinotrax tambin pueden tener pH bajo. En los trasudados, el pH del lquido pleural puede
estar ms alto que en la sangre debido al transporte activo del CO3H de la sangre al
espacio pleural.

Causas de derrame pleural con pH inferior a 7'30


Derrame paraneumnico complicado y empiema
Tuberculosis
Neoplasias
Rotura esofgica
Artritis reumatoide

Referencia:

1.- Fishman AP, , Fishman JA, Grippi MA, Kaisser LR, Seor RM. Pulmonary Diseases and
disorder. 3a. Edicin McGraw-Hill, EUA, 2006.

53.- Masculino de 49 aos de edad que se queja de dolor y rigidez en las articulaciones de
3 meses de evolucin. Ambos pies y ambas manos estn calientes y las articulaciones
edematizadas. Cul de los siguientes sugiere el diagnstico de artritis reumatoide?

a)
b)
c)
d)

Rigidez matutina que dura 1 hr


Adormecimiento y palidez de los dedos al ser expuestos al fro
Afeccin simtrica de las articulaciones distales interfalngicas
Lquido sinovial con alta viscosidad y 30 mil linfocitos /mm3

Los criterios diagnsticos establecidos por el Colegio Americano de Reumatologa, incluyen


alguno de los siguientes:
Presencia de artritis de ms de 6 semanas de duracin
Rigidez articular matutina prolongada (+ de una hora)
Presencia de ndulos caractersticos en la piel
Erosiones articulares visibles por radiologa
Positividad analtica de un anticuerpo que se conoce como factor reumatoide, si bien
el 25% de los pacientes con AR nunca desarrollarn este factor y, dicho anticuerpo, puede
aparecer en sujetos que no tienen AR.

BIBLIOGRAFA RECOMENDADA:
1) Klippel JH, Stone JH, Crofford LJ, White PH, editors. Primer on the rheumatic
diseases. 13th ed. New York: Springer-The Arthritis Foundation; 2008.

2) Martnez-Elizondo P, editor. Introduccin a la Reumatologa. 4a ed. Mxico: Colegio


Mexicano de Reumatologa A.C./Intersistemas S.A. de C.V.; 2008.
3) Firestein GS, Budd RC, Harris ED Jr, McInnes IB, Ruddy S, Sergent JS, editors.
Kelleys Textbook of Rheumatology. 8th ed. Philadelphia: Saunders Elsevier; 2009.

54.- Un estudiante universitario de 20 aos de edad acude a consulta debido a tos seca,
fiebre, cefalea y dolor muscular durante las ltimas 2 semanas. Refiere que sus
compaeros de casa han desarrollado sntomas similares. Niega el uso de drogas ilcitas y
no es homosexual. Su temperatura es de 38.2C, FC 90lpm, FR 18x. Se auscultan murmullo
vesicular bilateral. Una RX de trax muestra opacidades intersticiales multifocales.
Presenta leucocitosis y un test de aglutinina fro +. Cual de los siguientes es el patgeno
ms probable?

a)
b)
c)
d)

Bacterias anaerbicas
Mycoplasma pneumoniae
Pneumocystis carinii
Streptococcus pneumoniae

En el caso antes mencionado, se trata de una neumona llamada atpica que se presenta por
norma en pacientes jvenes y que han estado en lugares de convivencia con otras personas,
la imagen radiogrfica de esta neumona es de una neuropata intersticial como se indica.

Referencias:
1.- Fishman AP, , Fishman JA, Grippi MA, Kaisser LR, Seor RM. Pulmonary Diseases and
disorder. 3a. Edicin McGraw-Hill, EUA, 2006.
2.- Fraser, R ; Neil, C; Par, P; Diseases of the Chest, Third Edition, Editorial Elsevier,
2005.
3.- Murray and Nadels; Textbook Respiratory Medicine, Vol 1-2, Elsevier editorial, 2005.

55.- Un hombre de 55 aos de edad presenta una historia de 2 das de hemoptisis. Reporta
un inicio agudo de 8 episodios de tos con sangre fresca (aprox una cucharadita de sangre
por cada episodio). No reporta otros sntomas, excepto por una tos productiva de 5-10ml
de esputo cada maana. Tiene una historia de EPOC, para lo cual toma broncodilatadores.
Ha fumado 30 cigarrillos diariamente por los ltimos 30 aos. El examen fsico es normal, y
una RX limpia. Cual de los siguientes es la causa ms probable de la hemoptisis?

a)
b)
c)
d)

Bronquiectasias
Carcinoma broncognico
Bronquitis crnica
Tb pulmonar

La EPOC puede presentar episodios de hemoptisis con placa radiogrfica normal, aunque no
es muy frecuente; las otras causas como TB pulmonar y Cncer pulmonar as como las
bronquiectasias presentan lesiones en la radiografa que hace sospechar en estos
diagnsticos y en el presente caso no se mencionan lesiones en la Rx, la deficiencia de alfa 1
provoca la presencia de enfisema pulmonar la cual igualmente debera tener una placa con
alteraciones.
1.- Fishman AP, , Fishman JA, Grippi MA, Kaisser LR, Seor RM. Pulmonary Diseases and
disorder. 3a. Edicin McGraw-Hill, EUA, 2006.
2.- Fraser, R ; Neil, C; Par, P; Diseases of the Chest, Third Edition, Editorial Elsevier,
2005.
3.- Murray and Nadels; Textbook Respiratory Medicine, Vol 1-2, Elsevier editorial, 2005.

56. - A 38-year-old man comes to the physician because of slowly progressive visual
problems that make him bump into objects on both sides. He also reports that, while
driving, he has trouble switching lanes because he needs to turn his head all the way
backward to look for other cars. Ocular examination shows bitemporal field loss with
preserved visual acuity.Examination of the fundus is unremarkable. Which of the following
is the most likely diagnosis?

a)
b)
c)
d)

Pituitary adenoma
Occipital lobe meningioma
Optic neuritis
Retinal detachment

PRESENTACIN CLNICA
Los tumores hipofisarios se reconocen clnicamente por uno o ms de tres patrones de
presentacin muy constantes:
- Sntomas de hipersecrecin hipofisiaria
- Sntomas de hiposecrecin hipofisaria
- Sntomas neurolgicos
El tercer patrn de presentacin es el dominado por los sntomas neurolgicos, aislados o
coexistentes con una o varias de las alteraciones endocrinolgicas antes descritas. Como se
ha dicho, una masa hipofisaria progresivamente creciente generar una constelacin de
signos y sntomas neurolgicos que dependern de la trayectoria del crecimiento y de las
estructuras nerviosas vecinas que resulten alteradas. Los sntomas y signos a encontrar
son:
a. Cefaleas: Pueden ser un signo precoz y se atribuyen al estiramiento de la duramadre
que recubre la hipfisis o del diafragma de la silla turca. Est presente en el 75% de los
casos y se localiza ms frecuentemente en la regin frontal y orbital.
b. Prdida de visin: Debido a la compresin de los nervios pticos o del quiasma ptico. El
patrn clsico de prdida visual es una hemianopsia bitemporal a menudo asociada con
disminucin de la agudeza visual. Posteriormente puede haber una ceguera completa de uno
o ambos ojos.
c. Manifestaciones hipotalmicas: Debido a la compresin del hipotlamo por grandes
adenomas hipofisarios que provocan:
- Alteraciones del sueo
- Alteraciones de la atencin
- Alteraciones de la conducta
- Alteracin de la alimentacin
- Alteracin de las emociones
d. Hidrocefalia obstructiva: Debido a la infiltracin de las lminas terminales que hace
que el tumor penetre en la regin del III ventrculo, ocasionando obstruccin a la salida de
LCR.
e. Compromiso de nervios craneales. Debido a la extensin lateral del tumor hacia la
regin del seno cavernoso, por donde transcurren los nervios craneanos. As las
manifestaciones ms caractersticas son:
- Ptosis: Por compromiso del III nervio craneal (motor ocular comn).

- Dolor o alteraciones sensitivas faciales. Por compromisos de las ramas oftlmica y maxilar
superior del Nervio Trigmino.
- Diplopia: Por compromiso del III, IV y VI nervios craneales.

Bibliografa:
1.

Kovacs K, Horvath E, Vidal S: Classification of pituitary adenomas. J Neurooncol 54


(2): 121-7, 2001.

2. Ironside JW: Best Practice No 172: pituitary gland pathology. J Clin Pathol 56 (8):
561-8, 2003.
3. Scheithauer BW, Kovacs KT, Laws ER Jr, et al.: Pathology of invasive pituitary
tumors with special reference to functional classification. J Neurosurg 65 (6):
733-44, 1986.
4. Ezzat S, Asa SL, Couldwell WT, et al.: The prevalence of pituitary adenomas: a
systematic review. Cancer 101 (3): 613-9, 2004.

57. - A 27-year-old woman has been sad for the last two weeks. She is fatigued a has a
hard time concentrating at work. Just a few weeks earlier she was energetic an
enthusiastic, and was able to work 10-12 hours a day with little sleep and go dancing at
night. Her husband wants a divorce because he is tired of these constant ups and downs.
The most accurate diagnosis is:

a)
b)
c)
d)

Borderline personality disorder


Seasonal mood disorder
Dissociative identity disorder
Cyclothymic disorder

Criterios para el diagnstico de


F34.0 Trastorno ciclotmico (301.13)
A. Presencia, durante al menos 2 aos, de numerosos perodos de sntomas hipomanacos y
numerosos perodos de sntomas depresivo que no cumplen los criterios para un episodio
depresivo mayor.
Nota: En los nios y adolescentes la duracin debe ser de al menos 1 ao.
B. Durante el perodo de ms de 2 aos (1 ao en nios y adolescentes) la persona no ha
dejado de presentar los sntomas del Criterio A durante un tiempo superior a los 2 meses.
C. Durante los primeros 2 aos de la alteracin no se ha presentado ningn episodio
depresivo mayor, episodio manaco o episodio mixto.
Nota: Despus de los 2 aos iniciales del trastorno ciclotmico (1 ao en los nios y
adolescentes), puede haber episodios manacos o mixtos superpuestos al trastorno
ciclotmico (en cuyo caso se diagnostican ambos trastornos, el ciclotmico y el trastorno
bipolar I) o episodios depresivos mayores (en cuyo caso se diagnostican ambos trastornos,
el ciclotmico y el trastorno bipolar II).
D. Los sntomas del Criterio A no se explican mejor por la presencia de un trastorno
esquizoafectivo y no estn superpuestos a una esquizofrenia, un trastorno
esquizofreniforme, un trastorno delirante o un trastorno psictico no especificado.
E. Los sntomas no son debidos a los efectos fisiolgicos directos de una sustancia (p. ej.,
una droga, un medicamento) o a una enfermedad mdica (p. ej., hipertiroidismo).
F. Los sntomas provocan malestar clnicamente significativo o deterioro social, laboral o de
otras reas importantes de la actividad del individuo.

58.- Masculino de 32 aos que inici con disfona y disfagia, es referido al servicio de
endocrinologa donde es confirmado el diagnostico de ndulo tiroideo nico, El estudio
recomendado es?
a) Biopsia por aspiracin con aguja fina
b) Gammagrama tiroideo
c) Tomografa computada de cuello
d) Repetir Pruebas de funcin tiroidea

El ndulo tiroideo solitario se define como el crecimiento localizado de la glndula tiroides,


usualmente es benigno, la prevalencia es del 4 al 7% en la poblacin general. Aunque el
cncer tiroideo es el tumor endocrino ms comn slo representa el 1% de todos los
cnceres y 5% de todos los ndulos tiroideos. El estudio clnico diagnstico y teraputico
debe iniciarse con historia clnica completa, exploracin fsica y exmenes de laboratorio
que incluyan un perfil tiroideo para evaluar funcin de la glndula. La citologa tiroidea
por aspiracin (CTA) es el principal procedimiento diagnstico en los pacientes con
ndulo tiroideo solitario, por ser capaz de diferenciar lesiones benignas de las
malignas, sus principales ventajas son: segura, reduce costos de atencin mdica,
selecciona mejor los pacientes que sern sometidos a tratamiento quirrgico y se
realiza en pacientes ambulatorios.
Referencias:
1.

Torres AP, Hernndez SE, Caracas PN, Serrano GI et al Diagnstico y tratamiento


del ndulo tiroideo. Rev Edocrinol Nutr 2000; 8 (3): 87-93.

59.- Se ingresa al hospital a masculino de 66 aos con diagnstico de Neumona


Neumoccica. Es tratado con Ceftriaxona
durante su estancia
evoluciona
satisfactoriamente. A los 8 das de su estancia inicia con fiebre, diarrea lquida maloliente,
dolor abdominal tipo clico y leucocitosis. Cul, de los siguientes, le parece el diagnstico
ms probable?:

a) Sabmonellosis.
b) Infeccin por Clostridium Difficile.
c) Amebiasis intestinal
d) Giardiasis.

El Clostridium difficile es un bacilo anaerobio formador de esporas subterminales grandes,


de forma oval, localizadas en el intestino. Posee dos enterotoxinas que producen amplios
daos en las clulas del intestino y as ocasionar diarrea. Los pacientes pueden presentar un
amplio espectro de la enfermedad, que va de la diarrea simple no complicada asociada al uso
de antibiticos, hasta la colitis seudomembranosa asociada al uso de stos, potencialmente
mortal. El C. difficile no tiene serotipos. Existen cepas patgenas y no patgenas. Las
ltimas producen cantidades variables de toxina A (enterotoxina) y B (citotoxina). Los
tratamientos con antibiticos de amplio espectro eliminan gran cantidad de la flora normal,
permitiendo sobrecrecimiento del C. difficile patgeno. No existen defensas inmunolgicas
definidas del husped. El estndar de oro para el diagnstico de laboratorio es la prueba de
citotoxina en heces. El tratamiento puede basarse en metronidazol o vancomicina, aunque
suelen ocurrir recidivas.

Cuadro II. Opciones de tratamiento con


antimicrobianos, tasa de respuesta y recidiva para la diarrea y colitis aguda
por Clostridium difficile.
Medicamento*
Tasa de respuesta
Tasa de recidiva
Metronidazol:
98%
7%
Adultos: 250 mg c/6horas
Nios: 40 mg / kg / da
Vancomicina:
96%
18%
Adultos:125 mg c/6 horas
Nios: 10 mg/kg/ c/6 horas
Bacitracina: 25,000 U c/6 horas
83%
34%
Colestiramina (4 g c/6 horas)
68%
Desconocida
Tratamiento de 10 das.

Indicaciones de ciruga
La ms importante indicacin de la ciruga es el agravamiento de la situacin clnica del
paciente, a pesar de estar recibiendo el tratamiento indicado.
Las bases clnicas son la insuficiencia orgnica, peritonitis (posible perforacin) y colitis
progresiva.
La enfermedad progresiva puede documentarse por medio de tomografa computarizada; la
pared colnica muy engrosada y la presencia de ascitis son indicativos de un pobre
pronstico.

Gaceta Mdica de Mxico


Volumen Volume 138
Nmero Number 1 Enero-Febrero
January-February 2002

Artculo:

Infeccin por Clostridium difficile

60.- Femenino fumadora de 2 cajetillas diarias de tabaco, con antecedentes de DM tipo II


descontrolada, as como HTAS, actualmente portadora de lcera gstrica Cul de las
siguientes es la indicacin quirrgica?
a) Sangrado de tubo digestivo alto con inestabilidad hemodinmica.
b) Menor de 60 aos.
c) Biopsia negativa a tumor maligno.
d) La lcera no cicatriza despus de 12 semanas de tratamiento bien llevado por el
paciente.

lcera Pptica
50-75 % de casos.
La hemorragia por lcera duodenal es 4 veces ms frecuente que la lcera gstrica, pero
ambas tienen la misma tendencia a la hemorragia.
La Hemorragia Masiva tiene una frecuencia del 10-15% y generalmente es producida por
horadacin por el proceso inflamatorio de la arteria regional.
*
a)
b)
c)

Localizacin:
Bulbo duodenal.
Curvatura menor.
Zona prepilrica.

De ellas, la curvatura menor hace las hemorragias masivas ms frecuentes, pero la


duodenal, en general, es la ms frecuente.
La relacin entre la localizacin de la lcera y la gravedad de la hemorragia se debe a la
lesin de las arterias principales, de la curvatura menor, la coronaria estomquica, la
pilrica y la gastroduodenal.
TRATAMIENTO DE LA HEMORRAGIA DIGESTIVA ALTA

Es una emergencia Mdico-Quirrgica, de gran importancia y frecuencia, cuya morbilidad y


mortalidad estn influenciadas por un manejo oportuno, coherente, en el que
necesariamente concurren varios especialistas, en lo posible organizados en equipo.

Es evidente que el manejo en equipo y la identificacin de los pacientes de alto riesgo se


convierten en las armas ms importantes en el manejo actual de esta patologa.

TRATAMIENTO QUIRURGICO

El momento de la indicacin y la tcnica a realizar son los pilares principales de una desicin
correcta. Las tcnicas a realizar sern de acuerdo a la experiencia del cirujano y el riesgo
quirrgico del enfermo. Tales como: Vagotoma troncular piloroplasta, es ms rpida y
menos traumtica, elimina factor vagal, baja mortalidad postoperatoria, pero alto ndice de
recidiva del sangrado.

Vagotoma ms Gastrectoma 4/5 elimina todo el factor sangrante, usada en fracaso de


otras tcnicas.

Gastrectoma total: en casos especiales. Estado general del paciente, edad, obesidad,
repeticin de sangrado indican la tcnica a usar.

En lcera Duodenal la Vagotoma ms piloroplasta y sutura de la lcera es lo ms adecuado.


En lcera Gstrica, Gastrectoma y Sutura de la lcera: Gastrectoma subtotal Bilroht I,
reseccin en cua de lcera.

Vrices esofgicas: sonda de Sengstaken o de Linton.

Tcnica de desconeccin portoacigos o Tanner modificada.

Lectura recomendada:
Yamada Manual de gastroenterologia pagina 337.

61.- Masculino de 70 aos de edad, que se queja de debilidad muscular. Se aprecia


heliotropo con enema y eritema en su trax superior, cuello y cara. Qu es lo ms probable
que encuentre a la exploracin?
a)
b)
c)
d)

Debilidad muscular proximal


Ataxia
Hiperreflexia de tendones
Inflamacin de pequeas articulaciones

Son un grupo heterogneo de enfermedades que se caracterizan por debilidad muscular


secundaria a la inflamacin del msculo estriado.

EPIDEMIOLOGA

INCIDENCIA: 5 casos nuevos por milln / ao


PREVALENCIA: 10 - 60 casos por milln de habitantes.
2 PICOS DE EDAD: 5-15 aos
40-60 aos
FRECUENCIA MUJER : HOMBRE
2:1
RAZA NEGRA > ORIENTALES (4:1)

Frecuencia baja.

La polimiositis respeta la piel, mientras que la dermatomiositis presentar alteraciones


cutneas caractersticas acompaando a la afectacin muscular
Cuadro Clnico:
- Alteraciones musculares. Vienen marcadas por la presencia de debilidad muscular aguda
o subaguda (generalmente de inicio insidioso), simtrica y difusa, con preferencia por
musculatura proximal de extremidades (cintura plvica y escapular), tronco y cuello. En la
mayora de los casos es indoloro. Con el tiempo, desarrollan atrofia, contracturas y
disminucin de los reflejos.
- Alteraciones cutneas. La ms frecuente en la DM es una erupcin cutnea
eritematoviolcea que afecta a cuello, cara y trax. Es caracterstico tambin, el eritema
heliotropo (en prpados), que puede extenderse a otras zonas fotoexpuestas), las ppulas
de Gottron (localizadas en los nudillos), telangiectasias periungueales, a veces ulceracin
drmica y calcinosis (fundamentalmente en la DM infantil).
- Articulares. Artralgias, artritis transitorias, no erosivas, con tendencia a la simetra.

- Otras. Afectacin cardiaca variable (alteracin ECG, arritmia, miocarditis), pulmonar


(fibrosis intersticial asociada con anti Jo-1), renal (muy rara), fenmeno de Raynaud.

Diagnstico:
- Analtica: aumento de VSG y de enzimas musculares, (CPK, aldolasa, GOT, GPT, LDH). La
CPK es la ms sensible y la que guarda una mejor correlacin clnica con la actividad de la
enfermedad y la valoracin de recadas. El FR es + en 20% y
ANA es + en 10-30%. Si la destruccin muscular es intensa, puede producir mioglobinuria.
- Destacan anticuerpos: anti-Jo1: en casos de PM asociado a neumonitis intersticial
(sndrome antisintetasa-miosistis, fibrosis pulmonar, artritis no erosiva y fenmeno de
Raynaud).
anti-PM1 o PM-Scl: asociacin con esclerodermia.
anti-Mi, en DM.
antimioglobina.

BIBLIOGRAFA RECOMENDADA:

Klippel JH, Stone JH, Crofford LJ, White PH, editors. Primer on the rheumatic
diseases. 13th ed. New York: Springer-The Arthritis Foundation; 2008.

Martnez-Elizondo P, editor. Introduccin a la Reumatologa. 4a ed. Mxico: Colegio


Mexicano de Reumatologa A.C./Intersistemas S.A. de C.V.; 2008.

Firestein GS, Budd RC, Harris ED Jr, McInnes IB, Ruddy S, Sergent JS, editors.
Kelleys Textbook of Rheumatology. 8th ed. Philadelphia: Saunders Elsevier; 2009.

62.- Una mujer de 26 aos consulta por un episodio de hemiparesia izquierda sugerente de
ictus. Entre sus antecedentes refiere un hbito tabquico, no se ha documentado
hipertensin ni hiperglucemia, ha tenido un episodio previo de amaurosis fugax y dos
episodios de tromboflebitis en extremidades inferiores. No refiere antecedentes
quirrgicos ni ingesta de medicacin. Ha tenido tres abortos espontneos. El hemograma y
el estudio de coagulacin son normales. La funcin heptica y renal y los electrlitos son
normales. El colesterol total es de 260 mg/dl (normal <240) y los triglicridos de 160 mg/dl
(normal <150). La TC en la fase aguda no aporta datos significativos. Cul sera el
diagnstico ms probable y conducta a seguir?

a) El cuadro corresponde a sndrome antifosfolpido. Solicitara una determinacin de


anticuerpos anticardiolipina.
b) Se trata de una endocarditis infecciosa a partir de una tromboflebitis sptica.
Iniciara tratamiento antibitico emprico en espera de los cultivos.
c) El cuadro corresponde a un accidente aterotrombtico en una paciente con una
hiperlipemia familiar. El origen ms probable es la cartida. Solicitara un estudio de
troncos supraarticos. Iniciara tratamiento hipolipemiante.
d) Dada la edad, se trata de una enfermedad desmielinizante. Solicitara una resonancia
magntica cerebral.

El Sndrome Antifosfolipdico es una entidad adquirida y multisistmica, caracterizada por


hipercoagulacin, en la que las manifestaciones cutneas, en un alto porcentaje, permiten
sospechar su diagnstico; stas se caracterizan por livedo reticularis y los diferentes
grados de necrosis cutnea dependiendo de los vasos afectados, adems de otros hallazgos
importantes como son la trombocitopenia y los antecedentes de abortos. Los marcadores
serolgicos son los anticuerpos antifosfolipdicos que corresponden al anticoagulante lpico
y a la anticardiolipina.

SNDROME ANTIFOSFOLIPDICO
CRITERIO MAYOR
- Trombosis arterial
- Trombosis venosa
- Aborto recurrente-muerte intrauterina
- Trombocitopenia
El diagnstico se establece con cualquiera de los criterios mayores y el hallazgo serolgico
de anticoagulante lpico o ttulos altos de anticuerpos anticardiolipina (1,4,9).
Peridicamente se realizan reuniones de consenso para unificar criterios de diagnstico y
tratamiento en el SAF.
El SAF puede clasificarse como primario o idioptico cuando no se demuestra una causa
subyacente y secundario a mltiples causas, de las cuales la ms importante es el LES en el
grupo de enfermedades inmunolgicas.

Referencias:
1. Gibson GE, Su D, Pittelkow MR. Antiphospholipid syndrome and the skin. J Am Acad
Dermatol. 1997; 36:970-82.
2. Griffiths MH, Papadaki L, Neild GH. The renal pathology of primary antiphospholipid
syndrome: a distinctive form of endothelial injury. QJ Med. 2000; 91:457-67.
3. Quintero del Ro AI. Antiphospholipid antibodies in pediatrics. Current Reumatol Rep.
2002; 4(5):387-91.
4. Nahass GT. Antiphospholipid antibodies and the antiphospholipid antibody Syndrome. J
Am Acad Dermatol. 1997; 36:149-68.
5. Piette WW. Antiphospholipid syndrome: the problems and the promise. Br J Dermatol.
2000: 142:1079-83.
6. Gezer S. Antiphopholipid syndrome. Dis Mon. 2003; 49(12):696-741.
7. Nash MJ, Camilleri RS, Kunka S, McKie IJ, Machin SJ, Cohen H. The anticardio- lipin
assay is required for sensitive screening for antiphopholipid antibodies. J Thromb
Haemost. 2004; 2(7):1071-3.

63.- Masculino de 25 aos, que 10 das despus de acudir a una despedida de soltero,
comienza con inflamacin de rodilla derecha y de ambos tobillos, conjuntivitis bilateral,
aftas orales y erosiones superficiales no dolorosas en el glande.
El diagnstico ms probable es:

a)
b)
c)
d)

Enfermedad de Still.
Infeccin gonoccica.
Infeccin por Staphylococcus Aureus.
Enfermedad de Reiter.

Proceso inflamatorio estril de la membrana sinovial, precedido o precipitado por una


infeccin que ocurre fuera de la articulacin.

DIAGNOSTICO:
HISTORIA
SNTOMAS GENERALES
MANIFESTACIONES MSCULO-ESQUELTICAS
Artralgias, artritis aditiva o migratoria.
Monoartritis u oligoartritis asimtrica.
Articulaciones grandes que sostienen peso: Rodillas, tobillos y caderas.
Dactilitis o dedos en salchicha
Afeccin axial: Articulaciones S-I y columna lumbar.
Entesopata, tenosinovitis.

MANIFESTACIONES GENITO-URINARIAS: Uretritis, balanitis circinada


(es importante sealar que las lesiones son indoloras) , prostatitis
Cervicitis, cistitis, enfermedad plvica inflamatoria.

MANIFESTACIONES CUTNEAS Y DE MEMBRANAS MUCOSAS


Queratodermia blenorrgica.
Eritema nodoso.
Distrofia ungueal.
lceras orales.

MANIFESTACIONES OCULARES
Conjuntivitis y uvetis.
MANIFESTACIONES G-I

BIBLIOGRAFA RECOMENDADA:

Klippel JH, Stone JH, Crofford LJ, White PH, editors. Primer on the rheumatic
diseases. 13th ed. New York: Springer-The Arthritis Foundation; 2008.

Martnez-Elizondo P, editor. Introduccin a la Reumatologa. 4a ed. Mxico: Colegio


Mexicano de Reumatologa A.C./Intersistemas S.A. de C.V.; 2008.

Firestein GS, Budd RC, Harris ED Jr, McInnes IB, Ruddy S, Sergent JS, editors.
Kelleys Textbook of Rheumatology. 8th ed. Philadelphia: Saunders Elsevier; 2009.

64.- Se trata de masculino de 26 aos, con antecedentes de episodios recurrentes de


dolor ocular, fotofobia y lagrimeo, que desarrolla dolor insidioso y progresivo en regin
lumbar con exacerbacin nocturna en cama. A la exploracin fsica: test de Schber
positivo y soplo de regurgitacin artica grado II-III/VI. Rx de trax: sugerente de
retraccin fibrosa apical derecha. El diagnstico ms probable es:

a)
b)
c)
d)

Sndrome de Reiter.
Espondilitis anquilosante.
Osteoartrosis.
Enfermedad de Whipple.

La espondilitis anquilosante es un padecimiento reumtico inflamatorio,


generalizado y crnico, que afecta primordialmente el esqueleto axial, con la
presencia de dao de las articulaciones S-I (sacroiliitis) como su hallazgo
fundamental.
PREVALENCIA: Africanos y esquimales: 0.1 %. Blancos: 0.5 1 %. Indios Haida
(Norte de Canad): 6 %
PREVALENCIA DEL HLA-B27: 6 8 %
PREVALENCIA EN FAMILIARES DE ENFERMOS, HLA-B27 +: 10 20 %
INCIDENCIA: 6.3 6.9/100 habitantes por ao.
CONCORDANCIA en gemelos: Monocigotos, 63 %; dicigotos, 12.5 %
EDAD: Se inicia en la adolescencia o la edad adulta temprana. Rara despus de los
40 aos.
GNERO: 3 5:1 a favor del hombre.
SACROILIITIS:
Infiltrado de linfocitos T CD4+ y CD8+, as como macrfagos.
RNAm de TNF abundante cerca de los infiltrados.
RNAm de TGF- cerca de las reas de formacin de hueso nuevo.

Tejido de granulacin en mdula sea subcondral.


Hallazgos precoces: Sinovitis, inflamacin MO subcondral.
Hallazgos avanzados: destruccin extensa del cartlago y el hueso
subcondral.

COLUMNA VERTEBRAL: Osteitis, sindesmofitos, acuadramiento.


ENTESITIS
ARTICULACIONES PERIFRICAS: Hiperplasia sinovial, infiltrado linfoide, fibrosis
y lesiones vasculares.

OCULARES: uvetis anterior. En la mayora de los enfermos se produce durante los


primeros 10 aos de evolucin.
En general es unilateral, con tendencia a recidivar. Cursa con dolor, fotofobia y
lagrimeo. No suele dejar secuelas.
CARDIO-VASCULARES: aortitis ascendente, insuficiencia artica, anormalidades de
la conduccin.
La ms caracterstica es la insuficiencia artica por inflamacin de la aorta y de la
vlvula artica. Es ms frecuente en la espondilitis anquilosante de larga duracin,
especialmente en las que cursan con artritis perifrica importante y con
manifestaciones generales (fiebre, adelgazamiento y anemia). Otras manifestaciones
son la insuficiencia cardiaca, la cardiomegalia y los defectos de conduccin.

PULMONARES: fibrosis apical


GASTRO-INTESTINALES: EII
NEUROLGICAS: Complicaciones por fracturas y luxaciones, sndrome de la cola
de caballo

RENALES: Amiloidosis, nefritis por IgA, uso de AINE, prostatitis inespecfica


OTRAS.

Referencias:
Braun J, et al. Arthritis Rheum 1995;38:499-505.

65.- Femenino de 43 aos que desde hace 1 mes inicia con debilidad a nivel de cintura
escapular y pelviana. En la piel se aprecia edema palpebral y coloracin eritematoviolcea
periorbitaria as como lesiones eritematodescamativas sobre prominencias sea de dorso
de manos. El diagnstico ms probable en esta paciente es:

a)
b)
c)
d)

Dermatomiositis.
Eritema polimorfo.
Lupus eritematoso sistmico.
Artritis reumatoide.

DERMATOMOSISTIS
Cuadro Clnico:
- Alteraciones musculares. Vienen marcadas por la presencia de debilidad muscular aguda
o subaguda (generalmente de inicio insidioso), simtrica y difusa, con preferencia por
musculatura proximal de extremidades (cintura plvica y escapular), tronco y cuello. En la
mayora de los casos es indoloro. Con el tiempo, desarrollan atrofia, contracturas y
disminucin de los reflejos.
- Alteraciones cutneas. La ms frecuente en la DM es una erupcin cutnea
eritematoviolcea que afecta a cuello, cara y trax. Es caracterstico tambin, el eritema
heliotropo (en prpados), que puede extenderse a otras zonas fotoexpuestas), las ppulas
de Gottron (localizadas en los nudillos),
telangiectasias periungueales, a veces ulceracin drmica y calcinosis (fundamentalmente
en la DM infantil).

- Articulares. Artralgias, artritis transitorias, no erosivas, con tendencia a la simetra.


- Otras. Afectacin cardiaca variable (alteracin ECG, arritmia, miocarditis), pulmonar
(fibrosis intersticial asociada con anti Jo-1), renal (muy rara), fenmeno de Raynaud.

Diagnstico:
- Analtica: aumento de VSG y de enzimas musculares, (CPK, aldolasa, GOT, GPT, LDH). La
CPK es la ms sensible y la que guarda una mejor correlacin clnica con la actividad de la
enfermedad y la valoracin de recadas. El FR es + en 20% y
ANA es + en 10-30%. Si la destruccin muscular es intensa, puede producir mioglobinuria.
- Destacan anticuerpos: anti-Jo1: en casos de PM asociado a neumonitis intersticial
(sndrome antisintetasa-miosistis, fibrosis pulmonar, artritis no erosiva y fenmeno de
Raynaud).

anti-PM1 o PM-Scl: asociacin con esclerodermia.


anti-Mi, en DM.
antimioglobina.

BIBLIOGRAFA RECOMENDADA:

Klippel JH, Stone JH, Crofford LJ, White PH, editors. Primer on the rheumatic
diseases. 13th ed. New York: Springer-The Arthritis Foundation; 2008.

Martnez-Elizondo P, editor. Introduccin a la Reumatologa. 4a ed. Mxico: Colegio


Mexicano de Reumatologa A.C./Intersistemas S.A. de C.V.; 2008.

Firestein GS, Budd RC, Harris ED Jr, McInnes IB, Ruddy S, Sergent JS, editors.
Kelleys Textbook of Rheumatology. 8th ed. Philadelphia: Saunders Elsevier; 2009.

66. - Masculino de 56 aos que inicia con dolor precordial sbito irradiado a brazo
izquierdo, cuello y madbula, sudoracin profusa y ansiedad, con antecedentes de
tabaquismo postivo de 20 aos de evolucin consumiendo 1 cajetilla al da, obesidad grado I,
niega otras patologas. Se sospecha de IAM por lo que se procede a toma de ECG, el
siguiente trazo electrocardiogrfco es compatible con:

a)
b)
c)
d)

IAM Postero-lateral
Angina inestable
IAM lateral Alto
IAM cara anterior

Electrocardiograma (EKG) de 12 derivaciones de un paciente con un infarto agudo del


miocardio de cara anterior. Ntese el supradesnivel convexo del segmento ST desde V1V5, con elevacin del punto J que alcanza los 6 mm en V3.

Diagnstico topogrfico del infarto agudo de miocardio (IMA): 5, 6


Infarto anterior extenso: DI, aVL, V1, V2, V3, V4, V5, V6
Infarto anteroseptal: VI, V2, V3, V4
Infarto anterolateral: DI, aVL, V4, V5, V6
Lateral alto: DI, aVL
Lateral bajo: V5, V6
Infarto inferior: Se observa en DII, DIII, aVF
Infarto posterolateral: DII, DIII, aVF, V5 y V6

67.-El dolor abdominal intermitente, la anemia, la proteinuria, la necrosis tubular, la


esterilidad, los trastornos de la conducta, la polineuropata perifrica se encuentran en la
intoxicacin por:

a) Organoclorados.
b) Sulfuro de hidrogeno.
c) Plomo.
d) Etilenglicol.

La intoxicacin por plomo es la ms comn de las exposiciones a metales, el cual tiene


muchos usos, las fuentes ms frecuentes vienen de las minas y del reciclado de materiales
conteniendo plomo. Este metal es absorbido por pulmones y del tracto gastrointestinal. El
mecanismo de accin es por unin a los grupos sulfhidrilo y txico para las enzimas
dependientes de zinc. Diagnstico: La toxicidad aguda se presenta luego de una exposicin
respiratoria a altas concentraciones, con encefalopata, insuficiencia renal y sntomas
gastrointestinales. La toxicidad crnica es la ms frecuente y se manifiesta con
compromiso multisistmico: hematopoytico, del sistema nervioso, gastrointestinal, rin y
sistema reproductor. (Astenia, dolor abdominal, irritabilidad, nusea, vmitos, prdida de
peso, cefalea, anemia, neuropata perifrica, ribete de Burton, IRC, proteinuria, Nefritis
intersticial,etc). En los exmenes auxiliares podemos encontrar anemia, punteado basfilo,
aumento del cido rico, etc.

REFERENCIA BIBLIOGRFICAS:

1. Keogh JP y Boyer LV. Lead en Sullivan y Krieger editores: Clinical Environmental


Health and Toxic Exposures. Lippincott Williams & Wilkins, 2da edicin, 2001.
2. Decreto Supremo N 019-98-MTC. Dispone eliminar del mercado la oferta de gasolina 95
RON con plomo y reducir el lmite mximo de contenido de plomo en la gasolina 84 RON.
(14/07/98)
3. Shannon Michael. Lead en Haddad, Shanon y Winchester editores: Clinical
Management of Poisoning and Drug Overdose. WB Saunders, 3ra edicin, 1998.
4. Krantz A, Dorevitch S. Metal exposure and common chronic diseases: A guide for the
clinician. Dis Mon 2004; 50:215- 262.
5. Nogu S. Burton s Line. N Engl J Med 2006; 354:e21.
6. Rempel D. The lead-exposed worker. JAMA 1989; 262:532-4.

68.- Usted es un cirujano general y descubre que haciendo algunas modificaciones a una
tcnica quirrgica disminuye el tiempo de uso de quirfano, pero quiere comparar si tiene
los mismos resultados clnicos que cuando aplica la tcnica clsica , Qu tipo de estudio
debe realizar?
a)
b)
c)
d)

Casos y controles.
Cohorte.
Ensayo clnico controlado.
Transversal.

Los ensayos clnicos controlados son estrategias diseadas para evaluar la eficacia de un
tratamiento en el ser humano mediante la comparacin de la frecuencia de un determinado
evento de inters clnico o desenlace en un grupo de enfermos tratados con la terapia en
prueba con la de otro grupo de enfermos que reciben un tratamiento control.

Referencias:
Calva M.J.J. Estudios Clnicos Experimentales. Salud Pblica de Mxico vol.42, nm. 4,
2000 (349).

69.- Se siguieron por un periodo de 10 aos a 500 hombre hipertensos y 500 hombres sin
hipertensin para detectar evento vascular cerebral (EVC). Durante el estudio 80 de los
hipertensos tuvieron EVC (Incidencia de 160 por 1000) y 30 sin hipertensin presentaron el
evento (Incidencia 60 por 1000), el RR fue de 2.66. Este es un ejemplo de un estudio:

a) Cohorte.
b) Transversal.
c) Ensayo clnico controlado.

d) De casos y controles.

En los estudios de cohorte se eligen dos grupos uno de expuesto y otro de no expuestos los
cuales son seguidos a travs del tiempo para detectar las posibles consecuencias.
Posteriormente se analiza la informacin calculado la incidencia en el grupo de expuestos y
en el grupo de no expuestos; y una vez obtenidos se calcula el Riesgo Relativo.

Referencia:
Ruiz M. A. Epidemiologa Clnica, Panamericana, 1. Ed. 2004; pgs: 287-289.

70.- Masculino de 65 aos es trasladado por ambulancia al servicio de urgencias,


encontrado en la va pblica una madrugada del reciente invierno, con fuerte aliento
alcohlico, slo responde a estmulos dolorosos, hemipleja braquiocrural derecha.
Frecuencia cardiaca irregular. Se ingresa y se realiza un ECG. Segn los siguientes trazos,
el diagnstico cardiolgico es:

a)
b)
c)
d)

Fibrilacin auricular.
IAM cara posterior.
Fibrilacin ventricular.
Angina estable.

Electrocardiograma:
- Frecuencia cardiaca 65 lpm.
- AQRS -20, aunque cuesta definirlo categricamente, son ejes llamados indeterminados.
- Ausencia de onda P.
- Lnea de base con oscilaciones irregulares:
- en DII y aVR son a muy alta frecuencia (corresponden a temblor muscular).
- en las otras derivaciones, la frecuencia y configuracin son compatibles con fibrilacin
auricular.
- Los complejos QRS en V4-6 tienen voltaje aumentado, aunque no llegan a cubrir los
criterios de crecimiento ventricular izquierdo.
- En la porcin final del QRS estn encerradas en crculo rojo las ondas J de
Osborn.
- Segmento ST y onda T de difcil definicin en DI-II, aVL, aVF (alteraciones difusas en la
repolarizacin ventricular).
La hipotermia produce enlentecimiento en la despolarizacin ventricular prolongacin en la
fase 2 del potencial de accin-, por lo que se prolonga el periodo refractario; induciendo la
aparicin de las ondas J de Osborn. Las fibras auriculares pueden responder con fibrilacin
cuando son expuestas al fro.
El solo tomar helado y sentir el dolor retroesternal est asociado a inversin en la onda T
en DII-III.
Las ondas J de Osborn o simil- tambin se observan en la hipercalcemia y la angina
vasoespstica.

71.- Femenino de 39 aos de edad la cual inicia con un cuadro de exoftalmos axial de varias
semanas de evolucin, con predominio en su ojo derecho. En la exploracin se aprecia una
conjuntiva con sntomas discretos de hiperemia y edema, una queratitis de carcter
punteado en tercio inferior corneal y se sospecha una retraccinpalpebral al observar cmo
el borde del prpado superior se encuentra por encima del limbo, permitindonos visualizar
la esclertica. La paciente no refiere diminucin de visin ni alteraciones tipo visin doble
y toma presin intraocular con parmetros dentro de los normales. Cul de los siguientes
diagnsticos le parece ms compatible con el cuadro?

a) Tumor intraorbitario.
b) Enfermedad de Graves-Basedow.
c) Queratoconjuntivitis epidmica.
d) Tumor intraocular.

OFTALMOPATA TIROIDEA.
Es la causa ms frecuente de exoftalmos tanto bilateral como unilateral en adultos. La
forma tpica aparece en pacientes con enfermedad de Graves-Basedow, en los que podemos
encontrar exoftalmos y sndrome palpebro-retrctil. Este sndrome ocular puede aparecer
en pacientes eutiroideos o hipotiroideos, pudiendo constituir el signo ms precoz de una
tirotoxicosis incipiente.
CLNICA.
Se distinguen dos formas clnicas:
a) Tirotxica (hipersensibilidad a las catecolaminas y habitualmente hay hipertiroidismo):
exoftalmos moderado depresible.
Hay edema del contenido orbitario, pero no fibrosis ni oftalmoparesia.
b) Maligna (inflamacin orbitaria autoinmune y puede haber normo o hipotirodismo):
exoftalmos irreductible severo con oftalmopejia progresiva y queratitis por exposicin.
Puede afectar al nervio ptico por compresin y producir prdida visual. Hay fibrosis de la
grasa y vientres musculares.
Manifestaciones oculares asociadas: retraccin palpebral bilateral que permite ver la
esclera por encima del limbo, disminucin de la motilidad palpebral, alteracin de la
pigmentacin palpebral, hiperemia conjuntival.

DIAGNSTICO.
Por los signos clnicos descritos y exploraciones complementarias, como la exoftalmometra
(medida de la protrusin ocular), la radiologa (aumento de densidad de los tejidos blandos),
el engrosamiento del vientre de algunos msculos extraoculares (apreciados en la TC, la RM
y la ecografa orbitaria) y la analtica sistmica

Referencias:
AACE Thyroid Task Force. American Association of Clinical Endocrinologists medical
guidelines for clinical practice fo rthe evaluation and treatment of hyperthyroidism and
hypothyroidism. Endocr Pract. 2002;8(6).
Davies TF, Larsen PR. Thyrotoxicosis. In: Kronenberg HM, Melmed S, Polonsky KS, Larsen
PR, eds. Williams Textbook of Endocrinology. 11th ed. Philadelphia, Pa: Saunders Elsevier;
2008:chap 11.

72.- Mujer de 25 aos de edad que presenta en la cara diseminado comedones, ppulas y
pstulas, crecimiento excesivo de vello en mejillas mentn y cuello, acompaada de
seborrea, refiere cada de pelo exagerada. Inici desde la adolescencia. El diagnstico
clnico es?

a)
b)
c)
d)

Acn e Hirsutismo.
Sndrome de masculinizacin.
Lupus eritematoso discoide.
Alopecia androgentica.

El hirsutismo es el crecimiento excesivo de vello terminal en mujeres siguiendo un patrn


masculino de distribucin, en zonas andrgeno-dependientes: patillas, barbilla, cuello,
areolas mamarias, trax, en rea inmediatamente superior o inferior al ombligo, as como en
muslos, espalda. Frecuentemente se asocia a acn, calvicie con patrn masculino (alopecia
andrognica) e irregularidades menstruales.
Es un trastorno que afecta aproximadamente al 10% de las mujeres en edad frtil, y puede
ser leve, lo que representa una variacin del patrn de crecimiento normal, y en raras
ocasiones es signo de un trastorno subyacente grave.
Por lo general, es idioptico, pero puede estar relacionado al exceso de andrgenos, como el
sndrome de ovario poliqustico o la hiperplasia suprarrenal congnita.
Los pacientes con acn presentan de un modo ms o menos frecuente seborrea de cuero
cabelludo, con o sin pitiriasis.
Otras alteraciones andrgeno-dependientes pueden asociarse, pero no de un modo habitual,
como sucede con el hirsutismo y la alopeca, que pueden presentarse en menos del 10% de
las pacientes.

En el Sndrome de Cushing Hipofisario, la ACTH estimula la produccin suprarrenal de


hormonas andro-gnicas, resultando esto en acn e hirsutismo.
En el sndrome de poliquistosis ovrica (Stein-Leventhal), hay un incremento de la secrecin
gonadal de andrgenos a nivel del hilio y de la teca interna. La hipersecrecin de
andrgenos dotados de bioactividad puede inducir acn e hirsutismo. La presencia de acn
en una mujer -adolescente o adulta - con niveles de testosterona srica superiores a 300
ng/dl sugiere la presencia de un tumor ovrico.
En la Anorexia Nerviosa, el acn se desencadena generalmente en la fase de recuperacin,
donde puede coexistir con un ovario poliqustico. En fase de amenorrea LHRH, LH, FSH,
estrgenos y progesterona estn disminudos. El cortisol plasmtico est elevado con vida
media prolongada y prdida de la variacin diurna, otro factor que puede producir acn.
No hay que olvidar aquellos sndromes donde el acn coexiste con importantes trastornos
osteoarti-culares, como ser el Sndrome de Apert y el Sndrome SAPHO.
El acn se halla generalmente presente en los pacientes epilpticos - tal vez provocado en
gran medida por la medicacin - y en pacientes psiquitricos (manaco-depresivos y
obsesivo-compulsivos). Es importante tener en cuenta la asociacin de una neurosis de
angustia inespecfica y el acn.

Referencia:
1. Azziz R, Carmina E, Sawaya ME. Idiopathic hirsutism. Endocr Rev 2000;21:347-62.
Hirsutismo. Gonzalez Guerra. Servicio de dermatologa.Fundacin Jimnez Daz (Madrid)

73.- Femenino de 56 aos con antecedentes de prtesis de cadera; no utiliza sus vendas
elsticas. Inicia sbitamente con disnea, taquipnea, dolor torcico, ansiedad y taquicardia.
Ud ordena una ECG que revela que el eje de QRS es mayor de 90 ms taquicardia sinusal.
Ud sospecha:

a)
b)
c)
d)

IAM
Cor pulmonare
TEP
Infarto pulmonar

TEP
Los factores de riesgo ms importantes son:
Infarto de miocardio, insuficiencia cardiaca congestiva e insuficiencia venosa
crnica.
Lesin por traumatismo, sobre todo fractura de miembros inferiores y huesos
largos.
Anticonceptivos y terapia estrognica.
Edad > 40 aos.
Inmovilizacin prolongada > 4 das aproximadamente.
Neoplasias con compresin del sistema venoso profundo y liberacin de sustancias
procoagulantes.
Estados de hipercoagulabilidad primaria.
Accidentes cerebrovasculares.
Parto y puerperio.
Antecedentes de TEP y TVP.
Ciruga ortopdica, abdominal y neurolgica.

Sntomas del TEP

(%)

Disnea de aparicin sbita


inexplicable

84

Dolor torcico de tipo pleurtico

76

Tos

50

Dolor en pantorrilla

39

Sudoracin - ansiedad

36

Hemoptisis

28

Infartos pulmonares: dolor


pleurtico, roce pleural,
hemoptisis y fiebre.

10

Dolor no pleurtico

17

Sncope

13

Palpitaciones

10

Dolor anginoso

Asintomtico

--

Sgnos del TEP

(%)

Taquipnea (> 20 r.p.m)

85

Taquicardia (> 100 l.p.m)

58

Aumento del 2 tono pulmonar

57

Estertores pulmonares

55

Fiebre > 37.5c

50

Signos de TVP en extremidades


inferiores

41

Roce pleural

18

Cianosis

18

Hepatomegalia

10

Reflujo hepatoyugular

ECG. En el que podemos encontrar:


Inversin de la onda T en derivaciones precordiales derechas.
Taquicardia sinusal.
Bloqueo de rama derecha.
Arritmias supraventriculares.
Trastornos de la conduccin.
Clsico patrn S1 Q3 T3 que es raro e indicativo de hipertensin pulmonar.
Eje de QRS es mayor de 90

Bibliografa:

Barranco Ruiz F. Martos Lpez J. Simon Martos B. Tromboembolismo pulmonar


masivo. En: Principios de Urgencias, Emergencias y Cuidados Crticos. [Internet].
Universidad de Burgos. [Fecha de consulta 28-7-2003]. Disponible en:
http://www.uninet.edu/tratado/c0208i.html.
Belle L, Martin M, Brunier S, Brunet N, Bosson JL, Gros C et al. Evaluation d'un
algorithme diagnostique de l'embolie pulmonaire non grave avec successivement: DDimeres, echodoppler veineux des membres inferieurs et tomodensitometrie
helicoidale au centre hospitalier d'annecy.Ann Cardiol Angeiol (Paris). 2002
Nov;51(5):243-7. [Medline]
Cereza G, Dans I. Eficacia y seguridad de Heparinas de bajo peso molecular en el
tratamiento de la tromboembolia pulmonar. Med Clin (Barc) 1999; 113: 115-116.
[Medline]
Conthe Gutierrez P, Lobos Bejaranoa JM, Alonso Garcia A. Tromboembolismo
pulmonar. FMC 1998; 5 (2): 89.
Dalen JE. Trombolytic therapy in patients with submassive pulmonary embolism. N
Engl Med 2003 Jan 23; 348(4): 357-359.
Farrell S, Hayes T, Shaw M A negative SimpliRED D-dimer assay result does not
exclude the diagnosis of deep vein thrombosis or pulmonary embolus in emergency
department patients. Ann Emerg Med 2000; 35:121-5.

74.- Masculino de 66 aos, con Insuficiencia Cardiaca por cardiopata hipertensiva, en


situacin estable (en clase funcional I segn grado de disnea), presenta en el Ecocardiograma, Disfuncin Sistlica (Fraccin de Eyeccin < 35%). Qu grupo de frmacos
estara ms indicado como tratamiento inicial?

a)
b)
c)
d)

Digitalicos.
Inhibidores de la ECA.
Antagonistas de Calcio.
Betabloqueantes.

IC Izquierda:
Disnea al ejercicio, tos, fatiga, ortopnea, DPN, cardiomegalia, estertores,
ritmo de galope, congestin venosa pulmonar.
IC Derecha:
Presin venosa elevada, hepatomegalia, edema, usualmente asociado a IC
Izq.

TIPOS:
Insuficiencia Cardaca Aguda

Edema pulmonar agudo


Shock Cardiognico

Insuficiencia Cardaca Crnica

Estado Fisiopatolgico Caracterizado por la incapacidad del corazn para bombear la


cantidad de sangre necesaria para abastecer el metabolismo celular.
La ICC representa un sndrome clnico complejo caracterizado por alteraciones de funcin
ventricular y de regulacin neurohormonal que se acompaa de:
-intolerancia al esfuerzo.
-Reduccin en la calidad de vida.
-Reduccin de la esperanza de vida.
SINTOMAS:
Disnea
Disminucin capacidad funcional
Sntomas urinarios
Sntomas cerebrales
Sntomas insuficiencia cardaca derecha

PROPUESTADEUNESQUEMADETRATAMIENTO
FibrilacinA.

Diurtico
IECA

contraindicacinoefecto
adverso

ARA
Siguensntomas

Espironolactona12,5a25mg/d
o

Digoxina0,125mga0,25mg/d

BB

Preferibleingresohospitalario
inicialconseguimiento
mensual

Se ha demostrado claramente la importancia del control neurohumoral en el


paciente con disfuncin ventricular.

Manejo inicial del paciente con ICA


= Diurtico + Vasodilatador.

Solo el paciente con ICA + BGC = Inotrpico.

Nuevos estudios clnicos e investigacin bsica se requiere para buscar nuevas


estrategias de manejo.

REFERENCIAS:
Los inhibidores de la enzima conversora de angiotensina Rev Cubana Cardiol Cir Cardiovasc
1997:11;29-47.

75.- Se trata de femenino de 39 aos la cual presenta amenorrea secundaria de 2 aos y


medio de evolucin. Los niveles reportados de prolactina son de 150ng/ml (normal hasta
20 ng/ml). La resonacia magntica detecta macrotumor de 2,8 cm. de dimetro con
expansin lateral izquierda. No alteraciones visuales. Cul sera el tratamiento de eleccin?

a) Ciruga por tratarse de un macrotumor.


b) Tratamiento mdico con agonistas dopaminrgicos.
c) Somatostatina previa a ciruga.
d) Radioterapia hipofisaria previa a ciruga.

Tratamiento

Los dopaminrgicos han revolucionado el tratamiento del prolactinoma y virtualmente han


dejado fuera a la ciruga; as independientemente del tamao del adenoma la primera opcin
teraputica es la farmacolgica.1,2 Con los dopaminrgicos se consigue en poco tiempo
restaurar el funcionamiento ovrico y corregir la esterilidad, incluso antes de que se
normalice la concentracin de prolactina; asimismo se consigue reducir el tamao del
adenoma.
Los dopaminrgicos actan sobre los receptores localizados en las clulas mamotrpicas de
la hipfisis anterior y suprimen la sntesis y secrecin de prolactina con la consecuente
normalizacin del eje hipotlamogonadotropico hipotlamogonadotropico- ovrico. La accin
dopaminrgica puede ocasionar efectos colaterales indeseables como nusea, hipotensin
arterial, constipacin nasal, mareo y estreimiento, los cuales no necesariamente
corresponden con la dosis utilizada, pero s se relacionan con el tipo de dopaminrgico. Est
ampliamente documentada la superioridad de la farmacoterapia para el tratamiento de los
prolactinomas; adems la ciruga es raramente curativa, incluso en el caso de
microadenoma.4,5.

Cuadro I. Agentes dopaminrgicos que se usan como tratamiento de la


hiperprolactinemia y el prolactinoma.
Genrico
Comercial Dosis (mg)
Bromocriptina
Parlodel 2.5-5 diaria
Lisurida
Dopergin 0.2 diaria
Quinagolida
Norprolac 25-50 diaria
Cabergolina
Dostinex 0.5 c/4 das

Los adenomas hipofisarios representan el 10% de todos los tumores intracraneales


diagnosticados y 25% de los tumores cerebrales que son intervenidos quirrgicamente. Los
objetivos del tratamiento de un paciente con un adenoma de la pituitaria son: eliminar el
efecto de la masa tumoral (compresin sobre estructuras vecinas) disminuir la produccin
excesiva de hormonas, restaurar la funcin normal de la pituitaria y evitar la recurrencia.
El tratamiento de eleccin para todos los prolactinomas es con un agonista de la dopamina.
La bromocriptina y la cabergolina son efectivas para reducir el tamao del tumor y para
restaurar la funcin gonadal. El tratamiento quirrgico debe recomendarse slo cuando
falla el tratamiento mdico.
Los tumores de la pituitaria productores de hormona de crecimiento son tratados
preferentemente mediante adenomectoma transesfenoidal, pero la normalizacin de los
niveles de HC y de IGF-1 ocurre en menos de la mitad de los pacientes con macroadenomas;
por lo tanto, un importante nmero de pacientes acromeglicos requiere un tratamiento
adicional. Los anlogos de la somatostatina son en la actualidad los medicamentos que ms
usados para el control de la acromegalia. En grupos especiales de pacientes, el tratamiento
con agonistas de la dopamina y somatostatina parece que suprimen mejor los niveles de HC
que cuando se administran esos frmacos en forma separada.

Figura 1. Paciente de 20 aos quien consult por amenorrea primaria. El estudio de


RMN muestra un macroprolactinoma que invade el seno cavernoso izquierdo y envuelve
la cartida del mismo lado.

RMN de control 10 meses despus de tratamiento con un agonista dopaminrgico.


Corte coronal en T1. No se observa tumor. Tallo hipofisario central y quiasma ptico
libre.

REFERENCIAS BIBLIOGRFICAS
1.-Gac Md Mx Vol. 140 No. 5, 2004
2.- Schlechte JA. Prolactinoma. N Engl J Med 2003;349:2035-2041.
3.- Zrate A, Canales ES, Jacobs LS, Soria J, Daughaday WH. Restoration of ovarian
function in patients with the amenorrhea-galactorrhea syndrome after long-term therapy
with L-Dopa. Fertil Steril 1973;24:340.
4.- Tyson JE, Carter JN, Andreassen B, Huth J, Smith B. Nursing mediated.

76. - A 4-year-old boy presents with a history of constipation since the age of 6 months.
His stools, produced every 3-4 days, are described as large and hard. Physical examination
is normal; rectal examination reveals a large ampulla, poor sphincter tone, and stool in
rectal vault. The next step in the management of this infant would be.

a)
b)
c)
d)

Lower GI barium study.


Parental reassurance an counseling in Hirschprungs disease.
Serum electrolyte measurement.
Upper GI barium study.

Expresin Clnica:
Dentro de los sntomas que permiten una sospecha precoz, se encuentra el estreimiento o
constipacin, definida en el recin nacido como el retraso en la eliminacin de meconio
mayor a 48 horas asociada a distensin abdominal, y en los nios mayores como
deposiciones infrecuentes de consistencia aumentada (. El 98% de los lactantes elimina el
meconio en las primeras 48 horas de vida. Los prematuros eliminan ms tardamente el
meconio, pero la EH es rara en prematuros. De los pacientes con EH, slo el 60% elimina el
meconio despus de las 48 horas, por lo que este signo no es patognomnico de la
enfermedad.
La mayora de los nios que presentan aganglionosis congnita, son sintomticos los
primeros das o las primeras semanas luego del nacimiento (2). Alrededor de dos tercios de
los pacientes presenta sntomas dentro de los tres primeros meses de vida y 80%
desarrolla sntomas dentro del primer ao de vida. Slo un 10% de los pacientes inicia
sntomas entre los 3 y 14 aos de edad y en general se trata de pacientes con enfermedad
de segmento ultracorto. (13)
Los recin nacidos y lactantes pequeos presentan con frecuencia signos de obstruccin
intestinal, distensin abdominal, vmitos biliosos e intolerancia a la alimentacin. La
inspeccin anal y la radiografa pueden orientarnos hacia una causa mecnica de obstruccin,
pero no descarta EH. Si la obstruccin no tiene una causa mecnica, adems de pensar en
una EH, debe plantearse el diagnstico diferencial con hipotiroidismo, insuficiencia
suprarrenal, hipokalemia, hipercalcemia, hipomagnesemia, y en casos excepcionales
alteraciones neuromusculares.
Cuando la sintomatologa es poco evidente,
Puede presentarse como un cuadro de constipacin crnica, con historia de dificultad en la
eliminacin de deposiciones, masas fecales palpables en fosa ilaca izquierda y un tacto
rectal en que no se encuentran deposiciones en la ampolla rectal y esfnter anal hipertnico.
En muchas ocasiones la estimulacin rectal provoca salida explosiva de heces lquidas de
olor ftido (17). Por lo tanto, frente a pacientes con constipacin crnica, en los cuales se
ha descartado causa mecnica de obstruccin intestinal, que no cede a las medidas
dietticas ni farmacolgicas, debe plantearse el diagnstico de:

EH. Tambin puede encontrarse dilatacin de asas intestinales, adelgazamiento de la pared


abdominal, alteraciones de la nutricin y el crecimiento.
En nios mayores, los sntomas ms comunes incluyen constipacin crnica progresiva,
impactacin fecal recurrente, mal incremento ponderal y malnutricin.

Rev. Ped. Elec. 2008, Vol 5, N 1. ISSN 0718-0918.


Servicio Salud Metropolitano Norte.
Facultad de Medicina Hospital Clnico de Nios.
Departamento de Pediatra y Ciruga Infantil Roberto Del Ro.

Referencias:
1. De Manueles J. Enfermedad de Hirschsprung.
Protocolos diagnsticos y teraputicos en pediatra. Sociedad Espaola de Pediatra. Pag.
56-60.
2. Feldmon T., Wershil B. Hirschsprung Disease. Pediatrics in review. Vol 23. N 11, August
2003.
3. Luis L.A., Encinas J.L., Avila L.F., et cols.
Enfermedad de Hirschsprung: enseanzas de los ltimos 100 casos. Cir Pediatr 2006;
19:177- 181.
4. J.M. Gil-Vener y cols. Diagnstico dieferncial de Hirschsprung-neurodisplasia intestinal.
Fiabilidad de las pruebas diagnsticas. Cir Pediatr 2006; 19: 91-94.
5. M. Lpez, y cols. ndices de fiabilidad de la manometra anorrectal para el diagnstico de
la enfermedad de Hirschsprung en cualquier edad. Cir Pediatr 2005; 18:13-16.
6. Hernndez F., Rivas S., vila L.F., Daz M., ET cols. Aganglionismos extensos.
Tratamiento y resultados a largo plazo. Cir Pediatr 2003; 16: 54-57.
7. Goulet O. y cols. Intestinal transplantation in children: preliminary experience en Paris.
JPEN.
J Parenter Enteral Nutr 1999; 23 (5 Suppl)
8. Pea A. Enfermedad de Hirschsprung. Los avances y las preguntas no contestadas. Cir
Pediatr 2002: 15:46-47.
9. Polliotto S, Heinen F, Anduna G, Korman R. Evaluacin de resultado a tres aos de
nuestra primera experiencia en el tratamiento laparoscpico de la enfermedad de
Hirschsprung. Cir Pediatr 2001; 14: 85-87.

77.- Un hombre de 50 aos acude al servicio de urgencias por presentar fiebre de 39.0c,
exantema mculo-papuloso generalizado, incluyendo palmas y plantas. El paciente labora en
el campo
ordeando vacas frecuentemente parasitadas por garrapatas. Seale la
enfermedad a la que se refiere, el germen causante y el tratamiento adecuado:

a) Kala-azar, Leishmaniae Donovani: Antimoniales.


b) Fiebre Q, Coxiella Burnetti, Doxiciclina.
c) Fiebre botonosa, Ricckettsia Conori: Doxiciclina.
d) Fiebre de Malta, Brucella Mellitensis: Cotrimoxazol.

Las rickettsias son organismos coco-bacilares, Gram negativos, de 2-3 micras de dimetro,
intracelulares, incapaces de crecer en ausencia de clulas vivas del husped.
El gnero Rickettsia es ubicado taxonmicamente en la familia Rickettsiaceae, junto a
otros dos gneros:
Coxiella, con la especie C.burnetii, responsable de la fiebre.
Q, y Ehrlichia con las especies E.chaffeensis, agente de la ehrlichiasis monoctica, y
E.phagocytophila, productora de la ehrlichiasis granuloctica humana.
Junto a la familia Rickettsiaceae est la familia Bartonellaceae con tres especies
principales: Bartonella henselae, agente de la enfermedad por araazo de gato (cat
scratch disease); B.quintana, responsable de la angiomatosis bacilar(1), y B.bacilliformis,
productora de la bartonelosis o enfermedad de Carrin (verruga peruana).
Las rickettsiosis son zoonosis transmitidas desde los huspedes o reservorios animales al
hombre a travs de picaduras de artrpodos diversos, que varan con cada enfermedad
(piojos, pulgas, garrapatas, otros caros, esencialmente).
La rickettsiosis por R.conorii es conocida con el nombre de fiebre botonosa o manchada del
Mediterrneo o fiebre de Marsella, siendo transmitida al hombre desde el perro que
constituye su reservorio por garrapatas de los gneros Amblyomma y Riphicephalus
principalmente. En ellas el germen cumple un ciclo que incluye el pasaje transovrico a los
descendientes por lo cual representan tambin otro verdadero reservorio del parsito.
Clnicamente la enfermedad se caracteriza por la aparicin en el sitio de la picadura de la
garrapata de una lesin inicial indurada con centro necrtico muchas veces (mancha negra
o tache noir) rodeada de aureola inflamatoria, seguida de adenopatas regionales de
carcter inflamatorio en los das subsiguientes. Concomitantemente, fiebre
frecuentemente alta de 39-40C, malestar general, cefaleas a veces intensas, dolores
musculares y articulares.
Es relativamente frecuente la observacin de un exantema mculo-papuloso que explica el
nombre de fiebre botonosa y que puede afectar varios territorios. Es una afeccin
endmica en Sudfrica, Europa del Sur y Medio Este (2). El diagnstico se confirma
esencialmente por la tcnica de inmunofluorescencia indirecta (IFI) empleando lminas que
contienen antgenos de R.conorii y utilizando, siempre que sea posible, sueros pareados

obtenidos al inicio del cuadro y 20-30 das despus para investigar la seroconversin. La
histopatologa de las lesiones iniciales o taches noires fue estudiada en detalle por

Montenegro y colaboradores en 1983(3). En otro trabajo posterior, Montenegro y


colaboradores (4) demuestran en ratones inoculados con R.conorii la importancia crucial de
la inmunidad celular con respecto a la humoral en el control de la infeccin experimental y
reduccin del ndice de mortalidad. El ratamiento de eleccin es dicloxacilina, alternativas
macrlidos y quinolonas (ciprofloxacino).
Bibliografa:

1 . Sampaio SAP, Rivitti EA. Dermatologia. So Paulo: Artes Mdicas, 1998: 1155.
2 . Harris RL, Kaplan SL, Bradshaw MW, Williams Jr, Temple W. Boutonneuse fever in
american travelers. J Infect
Dis 1986; 153:126-8.
3 . Montenegro MR, Mansueto S, Hegarty BC, Walker DH. The histology of taches
noires of boutonneuse fever and demonstration of Rickettsia conorii in them by
immunofluorescence. Virchows Arch (Pathol Anat) 1983; 400:309-17.
4 . Montenegro MR, Walker DH, Hegarty BC. Infection of genetically immunodeficient
mice with Rickettsia conorii . Acta
Virol 1984; 28:508-14.
5. Conti Daz IA, Rubio I, Somma Moreira RE, Prez Bormida G. Rickettsiiosis
cutneo-ganglionar por Rickettsiaconorii en el Uruguay. Rev Inst Med Trop (So Paulo),
1990.

78.- Femenino de 62 aos con diagnstico de miastenia gravis. Cul de los siguientes
medicamentos est dirigido al manejo de esta patologa?
a)
b)
c)
d)

Neostigmina.
Quinidina.
Sumatriptn.
Succinilcolina.

Manejo farmacolgico: Existen diferentes pautas teraputicas dirigidas a contrarrestar


los sntomas de la enfermedad o el mecanismo inmunolgico. Los frmacos utilizados son:

- Inhibidores de la Acetilcolinesterasa (Neostigmina, Piridostigmina). Dirigidos al manejo


sintomtico de la MG, mejorando la fuerza motora pero no la progresin de la enfermedad.
Su mecanismo de accin es la inhibicin reversible de la acetilcolinesterasa, lo cual genera
un aumento de ACh en la placa motora. La dosis a utilizar es variable y debe modificarse en
distintas etapas de la enfermedad, incluso siendo frecuente no lograr un efecto uniforme
en los diferentes grupos musculares en un mismo paciente. El objetivo por lo tanto ser

utilizar la dosis mnima con la que se genere la mejor respuesta clnica. El efecto se obtiene
de 30 minutos a 2 horas de la administracin y tiene una duracin de hasta 6 horas. Las
dosis recomendadas de Piridostigmina son de 15-60 mg cada 4-6 horas va oral y de
Neostigmina 0,5-2 mg/kg cada 4-6 horas intramuscular. Las reacciones adversas asociadas
son: dolor abdominal, hipersalivacin, aumento de las secreciones respiratorias y
bradicardia y se relacionan con el efecto colinrgico generado, por lo que es necesario
administrar concomitantemente atropina2,3.

Referencias:
1.- Ponsetia JM: Miastenia Gravis. Manual Teraputico. Barcelona; Springer Verlag Ibrica,

1995.

2.- Ponsetia JM, Espina E, Armengola M: Diagnstico y Tratamiento de la Miastenia grave.


Med Clin (Barc) 2000; 115: 264-70.
3.- Drachman DB: Myasthenia gravis. N Engl J Med 1994; 330: 1797-810.
4.- Andrews PI: Autoimmune myasthenia gravis in childhood. Semin Neurol 2004; 24: 10110
5. - Anlar B: Juvenile myasthenia: diagnosis and treatment. Paediatr Drugs 2000; 2: 1619.
6. - Gajdos P: Myasthenic syndrome. Diagnosis trends. Rev Prat 2000; 50: 419-23 7.

79.- El examen microscpico de una biopsia pulmonar de un paciente con mucormicosis


pulmonar mostrar:
a)
b)
c)
d)

Micelio Dicotomizado Septado Con Conidias


Micelio Dicotomizado Hialino Cenocitico
Micelio Con Clamidoconidios Y Blastoporas
Micelio Septado Y Esclerotes De Mediar

Examen directo. Se realiza a partir de exudados o secreciones nasales, expectoracin,


lavados bronquiales y heces, inclusive se puede hacer a partir de biopsias. La muestra se
debe aclarar con KOH al 10 %. Al microscopio se observan numerosas hifas no tabicadas,
hialinas, dicotmicas, de aproximadamente 5F de ancho por 20-25 de largo, esta imagen es
patognomnica.

Referencia:
Harada M, Manabe T, Yamashita K, Okamoto N. Pulmonary mucormycosis with fatal massive
hemoptysis. Acta Pathol 1992;42(1):49-54.

80.- Al realizar la exploracin clnica y colocar un diapasn que est vibrando frente al
conducto auditivo del odo que queremos explorar (conduccin area) y apoyando despus
sobre la mastoides (conduccin sea), podemos de modo sencillo y en la consulta, distinguir
entre sordera nerviosa (alteracin en la cclea o nervio auditivo) y sordera de conduccin
(trastorno en el sistema de transmisin tmpano-osicular). Cul de estas afirmaciones es
correcta para un paciente que presenta una sordera de conduccin?:

a) La percepcin del sonido es igual por va area que por va sea.


b) La percepcin del sonido es mejor por va area que por va sea.
c) La percepcin del sonido es mejor por va sea que por va area.
d) La percepcin del sonido es indistinguible tanto por va area como sea.

MEDICIN CLNICA DE LA AUDICIN


La valoracin audiolgica mnima debe incluir la determinacin de los umbrales de
conduccin area y sea, el umbral de recepcin y la discriminacin del lenguaje, una
timpanometra y pruebas de reflejos acsticos, que incluyan la prueba de deterioro de los
reflejos. La informacin obtenida por medio de estas tcnicas permite determinar si hace
falta una mayor diferenciacin entre la sordera neural y la sensorial.
La audicin por conduccin area se valora presentando un estmulo acstico mediante
auricular o altavoces. Una sordera o elevacin del umbral de audicin detectada por esta
prueba se puede deber a defectos en cualquier parte del aparato auditivo: pabelln
auricular, conducto auditivo, odo medio o interno, VIII par craneal o vas auditivas
centrales.
La audicin por conduccin sea se valora colocando una fuente sonora (el vibrador de un
audimetro o un diapasn) en contacto con la cabeza. El sonido produce una vibracin a
travs del crneo, que alcanza las paredes seas de la cclea y estimula directamente el
odo interno. La audicin por conduccin sea no atraviesa los odos externos y medio y
permite valorar la integridad del odo interno, del octavo nervio craneal y de las vas
auditivas centrales.
Si aumenta el umbral de conduccin del aire y el umbral de conduccin sea es normal, la
sordera es de conduccin, mientras que si ambos umbrales aumentan por igual es de tipo
neurosensorial. Existen algunas formas de sordera mixtas con componente neurosensorial y
de conduccin, en las que aumentan ambos umbrales, aunque el de la conduccin area es
ms significativo.

Referencia:

Manual Merck 10. Edicin en Espaol

Editors of The Merck Manual.

Robert S. Porter, MD, Editor-in-chief.

Justin L. Kaplan, MD, Senior Assistant Editor.

Editorial Board of The Merck Manual.

Vous aimerez peut-être aussi